Pulmonary

Réussis tes devoirs et examens dès maintenant avec Quizwiz!

A student nurse asks the RN what can be measured by arterial blood gases (ABGs). The RN tells the student that the ABGs can measure (select all that apply) a. acid-base balance. b. oxygenation status. c. acidity of the blood. d. glucose bound to hemoglobin. e. bicarbonate (HCO3−) in arterial blood.

A, B, C, E

For which patients with pneumonia would the nurse suspect aspiration as the likely cause of pneumonia (select all that apply)? a. Patient with seizures b. Patient with head injury c. Patient who had thoracic surgery d. Patient who had a myocardial infarction e. Patient who is receiving nasogastric tube feeding

A, B, E

A patient is concerned that he may have asthma. Of the symptoms that he relates to the nurse, which ones suggest asthma or risk factors for asthma (select all that apply)? a. Allergic rhinitis b. Prolonged inhalation c. History of skin allergies d. Cough, especially at night e. Gastric reflux or heartburn

A, C, D, E

A plan of care for the patient with COPD could include (select all that apply) a. exercise such as walking. b. high flow rate of O2 administration. c. low-dose chronic oral corticosteroid therapy. d. use of peak flow meter to monitor the progression of COPD. e. breathing exercises such as pursed-lip breathing that focus on exhalation.

A, E

A patient is seen at the clinic with fever, muscle aches, sore throat with yellowish exudate, and headache. The nurse anticipates that the collaborative management will include (select all that apply) a. antiviral agents to treat influenza. b. treatment with antibiotics starting ASAP. c. a throat culture or rapid strep antigen test. d. supportive care, including cool, bland liquids. e. comprehensive history to determine possible etiology.

C, D, E

A patient with TB has been admitted to the hospital and is placed in an airborne infection isolation room. What should the patient be taught (select all that apply)? a. Expect routine TST to evaluate infection. b. Visitors will not be allowed while in airborne isolation. c. Take all medications for full length of time to prevent multidrug-resistant TB. d. Wear a standard isolation mask if leaving the airborne infection isolation room. e. Maintain precautions in airborne infection isolation room by coughing into a paper tissue.

C, D, E

During the respiratory assessment of the older adult, the nurse would expect to find (select all that apply) a. a vigorous cough. b. increased chest expansion. c. increased residual volume. d. increased breath sounds in the lung apices. e. increased anteroposterior (AP) chest diameter

C, E

A patient with chronic obstructive pulmonary disease (COPD) has a nursing diagnosis of impaired breathing pattern related to anxiety. Which nursing action is most appropriate to include in the plan of care? A. Titrate oxygen to keep saturation at least 90%. B. Discuss a high-protein, high-calorie diet with the patient. C. Suggest the use of over-the-counter sedative medications. D. Teach the patient how to effectively use pursed lip breathing.

D

ANY patient with a respiratory condition asks "How does air get into my lungs?" The nurse bases her answer on her knowledge that air moves into the lungs because of a. contraction of the accessory abdominal muscles. b. increased carbon dioxide and decreased oxygen in the blood. c. stimulation of the respiratory muscles by the chemoreceptors. d. decrease in intrathoracic pressure relative to pressure at the airway

D

An appropriate nursing intervention for a patient with pneumonia with the nursing diagnosis of ineffective airway clearance related to thick secretions and fatigue would be to a. perform postural drainage every hour. b. provide analgesics as ordered to promote patient comfort. c. administer O2 as prescribed to maintain optimal oxygen levels. d. teach the patient how to cough effectively to bring secretions to the mouth.

D

Which guideline would be a part of teaching patients how to use a metered-dose inhaler (MDI)? a. After activating the MDI, breathe in as quickly as you can. b. Estimate the amount of remaining medicine in the MDI by floating the canister in water. c. Disassemble the plastic canister from the inhaler and rinse both pieces under running water every week. d. To determine how long the canister will last, divide the total number of puffs in the canister by the puffs needed per day

D

A patient with a possible pulmonary embolism complains of chest pain and difficulty breathing. The nurse finds a heart rate of 142 beats/minute, blood pressure of 100/60 mmHg, and respirations of 42 breaths/minute. Which action should the nurse take first? Prepare patient for a spiral computed tomography (CT). Administer anticoagulant drug therapy. Notify the patient's health care provider. Elevate the head of the bed to a semi-Fowler's position.

Elevate the head of the bed to a semi-Fowler's position.

A patient with chronic obstructive pulmonary disease (COPD) has poor gas exchange. Which action by the nurse would be most appropriate? Ask the patient to rest in bed in a high-Fowler's position with the knees flexed. Encourage the patient to sit up at the bedside in a chair and lean slightly forward. Have the patient rest in bed with the head elevated to 15 to 20 degrees. Place the patient in the Trendelenburg position with several pillows behind the head.

Encourage the patient to sit up at the bedside in a chair and lean slightly forward.

A patient who is taking rifampin (Rifadin) for tuberculosis calls the clinic and reports having orange discolored urine and tears. Which is the best response by the nurse? Ask the patient about any visual abnormalities such as red-green color discrimination. Explain that orange discolored urine and tears are normal while taking this medication. Advise the patient to stop the drug and report the symptoms to the health care provider. Ask if the patient is experiencing shortness of breath, hives, or itching.

Explain that orange discolored urine and tears are normal while taking this medication.

Which assessment finding in a patient who has received omalizumab (Xolair) is most important to report immediately to the health care provider? Flushing and dizziness Respiratory rate 22 breaths/minute Peak flow reading 75% of normal Pain at injection sit

Flushing and dizziness

The nurse provides preoperative instruction for a patient scheduled for a left pneumonectomy for cancer of the lung. Which information should the nurse include about the patient's postoperative care? Frequent use of an incentive spirometer Bed rest for the first 24 hours Positioning on the right side Chest tube placement with continuous drainage

Frequent use of an incentive spirometer

A patient with pneumonia has a fever of 101.4° F (38.6° C), a nonproductive cough, and an oxygen saturation of 88%. The patient complains of weakness, fatigue, and needs assistance to get out of bed. Which nursing diagnosis should the nurse assign as the highest priority? Impaired transfer ability related to weakness Impaired gas exchange related to respiratory congestion Ineffective airway clearance related to thick secretions Hyperthermia related to infectious illness

Impaired gas exchange related to respiratory congestion

When assessing a patient who has just arrived after an automobile accident, the emergency department nurse notes tachycardia and absent breath sounds over the right lung. For which intervention will the nurse prepare the patient? Emergency pericardiocentesis Stabilization of the chest wall with tape Insertion of a chest tube with a chest drainage system Administration of an inhaled bronchodilator

Insertion of a chest tube with a chest drainage system

The clinic nurse makes a follow-up telephone call to a patient with asthma. The patient reports having a baseline peak flow reading of 600 L/minute and the current peak flow is 420 L/minute. Which action should the nurse take first? Ask about recent exposure to any new allergens or asthma triggers. Question the patient about use of the prescribed inhaled corticosteroids. Tell the patient to go to the hospital emergency department. Instruct the patient to use the prescribed albuterol (Proventil).

Instruct the patient to use the prescribed albuterol (Proventil).

A patient who is experiencing an acute asthma attack is admitted to the emergency department. Which assessment should the nurse complete first? Determine when the dyspnea started. Listen to the patient's breath sounds. Obtain the forced expiratory volume (FEV) flow rate. Ask about inhaled corticosteroid use.

Listen to the patient's breath sounds.

The nurse is admitting a patient diagnosed with an acute exacerbation of chronic obstructive pulmonary disease (COPD).What is the best way for the nurse to determine the appropriate oxygen flow rate? Avoid administration of oxygen at a rate of more than 2 L/minute Maintain the pulse oximetry level at 90% or greater. Minimize oxygen use to avoid oxygen dependency. Administer oxygen according to the patient's level of dyspnea.

Maintain the pulse oximetry level at 90% or greater.

A patient who is experiencing an asthma attack develops bradycardia and a decrease in wheezing. Which action should the nurse take first? Notify the health care provider. Document changes in respiratory status. Administer IV methylprednisolone (Solu-Medrol). Encourage the patient to cough and deep breathe.

Notify the health care provider.

Which nursing action for a patient with chronic obstructive pulmonary disease (COPD) could the nurse delegate to experienced unlicensed assistive personnel (UAP)? Adjust oxygen to keep saturation in prescribed parameters. Obtain oxygen saturation using pulse oximetry. Teach the patient about safe use of oxygen at home. Monitor for increased oxygen need with exercise.

Obtain oxygen saturation using pulse oximetry.

A patient with chronic obstructive pulmonary disease (COPD) has a nursing diagnosis of imbalanced nutrition: less than body requirements. Which intervention would be most appropriate for the nurse to include in the plan of care? Offer high-calorie snacks between meals and at bedtime. Increase the patient's intake of fruits and fruit juices. Encourage increased intake of whole grains. Assist the patient in choosing foods with high vegetable and mineral content.

Offer high-calorie snacks between meals and at bedtime.

The clinic nurse teaches a patient with a 42 pack-year history of cigarette smoking about lung disease. Which information will be most important for the nurse to include? Computed tomography (CT) screening for lung cancer Reasons for annual sputum cytology testing Options for smoking cessation Erlotinib (Tarceva) therapy to prevent tumor risk

Options for smoking cessation

The nurse cares for a patient who has just had a thoracentesis. Which assessment information obtained by the nurse is a priority to communicate to the health care provider? Blood pressure is 145/90 mm Hg. Respiratory rate is 22 breaths/minute when lying flat. Oxygen saturation is 88%. Pain level is 5 (on 0 to 10 scale) with a deep breath.

Oxygen saturation is 88%.

The emergency department nurse is evaluating the effectiveness of therapy for a patient who has received treatment during an asthma attack. Which assessment finding is the best indicator that the therapy has been effective? Respiratory rate is 16 breaths/minute. Oxygen saturation is >90%. Accessory muscle use has decreased. No wheezes are audible.

Oxygen saturation is >90%.

A patient experiences a chest wall contusion as a result of being struck in the chest with a baseball bat. The emergency department nurse would be most concerned if which finding is observed during the initial assessment? Complaint of chest wall pain Heart rate of 110 beats/minute Large bruised area on the chest Paradoxic chest movement

Paradoxic chest movement

A patient with idiopathic pulmonary arterial hypertension (IPAH) is receiving nifedipine (Procardia). Which assessment would best indicate to the nurse that the patient's condition is improving? Patient's chest x-ray indicates clear lung fields. Patient reports decreased exertional dyspnea. Blood pressure (BP) is less than 140/90 mm Hg. Heart rate is between 60 and 100 beats/minute.

Patient reports decreased exertional dyspnea.

A young adult patient with cystic fibrosis (CF) is admitted to the hospital with increased dyspnea. Which intervention should the nurse include in the plan of care? Arrange for a hospice nurse visit. Perform chest physiotherapy every 4 hours. Place the patient on a low-sodium diet. Schedule a sweat chloride test.

Perform chest physiotherapy every 4 hours.

The nurse is caring for a patient with cor pulmonale. The nurse should monitor the patient for which expected finding? Complaints of chest pain Elevated temperature Peripheral edema Clubbing of the fingers

Peripheral edema

The nurse develops a plan of care to prevent aspiration in a high-risk patient. Which nursing action will be most effective? Insert nasogastric tube for feedings for patients with swallowing problems. Turn and reposition immobile patients at least every 2 hours. Place patients with altered consciousness in side-lying positions. Monitor for respiratory symptoms in patients who are immunosuppressed.

Place patients with altered consciousness in side-lying positions.

Which finding by the nurse for a patient with a nursing diagnosis of impaired gas exchange will be most useful in evaluating the effectiveness of treatment? Absence of wheezes, rhonchi, or crackles Respiratory rate of 18 breaths/minute Even, unlabored respirations Pulse oximetry reading of 92%

Pulse oximetry reading of 92%

A patient newly diagnosed with asthma is being discharged. The nurse anticipates including which topic in the discharge teaching? Self-administration of inhaled corticosteroids Side effects of sustained-release theophylline Use of long-acting b-adrenergic medications Complications associated wit oxygen therapy

Self-administration of inhaled corticosteroids

A patient with chronic obstructive pulmonary disease (COPD) has a nursing diagnosis of impaired breathing pattern related to anxiety. Which nursing action is most appropriate to include in the plan of care? Titrate oxygen to keep saturation at least 90%. Teach the patient how to effectively use pursed lip breathing. Suggest the use of over-the-counter sedative medications. Discuss a high-protein, high-calorie diet with the patient.

Teach the patient how to effectively use pursed lip breathing.

Which action by the nurse will be most effective in decreasing the spread of pertussis in a community setting? Teaching family members about the need for careful hand washing Teaching patients about the need for adult pertussis immunizations Encouraging patients to complete the prescribed course of antibiotics Providing supportive care to patients diagnosed with pertussis

Teaching patients about the need for adult pertussis immunizations

The nurse educator is evaluating the care that a new registered nurse (RN) provides to a patient receiving mechanical ventilation. Which action by the new RN indicates the need for more education? The RN increases the FIO2 to 100% before suctioning. The RN positions the patient with the head of bed at 10 degrees. The RN secures a bite block in place using adhesive tape. The RN asks for assistance to reposition the endotracheal tube.

The RN positions the patient with the head of bed at 10 degrees.

The nurse educator is evaluating the performance of a new registered nurse (RN) who is providing care to a patient who is receiving mechanical ventilation with 15 cm H2O of peak end-expiratory pressure (PEEP). Which action indicates that the new RN is safe? The RN changes the ventilator circuit tubing routinely every 48 hours. The RN tapes connection between the ventilator tubing and the ET. The RN plans to suction the patient every 1 to 2 hours. The RN uses a closed-suction technique to suction the patient.

The RN uses a closed-suction technique to suction the patient.

The nurse interviews a patient with a new diagnosis of chronic obstructive pulmonary disease (COPD). Which information is most helpful in confirming a diagnosis of chronic bronchitis? The patient's history indicates a 30 pack-year cigarette history. The patient denies having any respiratory problems until the last 12 months. The patient tells the nurse about a family history of bronchitis. The patient complains about a productive cough every winter for 3 months.

The patient complains about a productive cough every winter for 3 months.

The nurse teaches a patient about pursed lip breathing. Which action by the patient would indicate to the nurse that further teaching is needed? The patient practices by blowing through a straw. The patient inhales slowly through the nose. The patient puffs up the cheeks while exhaling. The patient's ratio of inhalation to exhalation is 1:3.

The patient puffs up the cheeks while exhaling.

The nurse teaches a patient with chronic bronchitis about a new prescription for Advair Diskus (combined fluticasone and salmeterol). Which action by the patient would indicate to the nurse that teaching about medication administration has been successful? The patient shakes the device before use. The patient attaches a spacer to the Diskus. The patient performs huff coughing after inhalation. The patient rapidly inhales the medication.

The patient rapidly inhales the medication.

The nurse teaches a patient how to administer formoterol (Perforomist) through a nebulizer. Which action by the patient indicates good understanding of the teaching? The patient coughs vigorously after using the inhaler. The patient attaches a spacer before using the inhaler. The patient activates the inhaler at the onset of expiration. The patient removes the facial mask when misting has ceased.

The patient removes the facial mask when misting has ceased.

The nurse takes an admission history on a patient with possible asthma who has new-onset wheezing and shortness of breath. Which information may indicate a need for a change in therapy? The patient takes propranolol (Inderal) for hypertension. The patient has a history of pneumonia 6 months ago. The patient uses acetaminophen (Tylenol) for headaches. The patient has chronic inflammatory bowel disease.

The patient takes propranolol (Inderal) for hypertension.

The nurse teaches a patient who has asthma about peak flow meter use. Which action by the patient indicates that teaching was successful? The patient takes montelukast (Singulair) for peak flows in the red zone. The patient uses albuterol (Proventil) metered dose inhaler (MDI) for peak flows in the yellow zone. The patient inhales rapidly through the peak flow meter mouthpiece. The patient calls the health care provider when the peak flow is in the green zone.

The patient uses albuterol (Proventil) metered dose inhaler (MDI) for peak flows in the yellow zone.

Which assessment finding obtained by the nurse when caring for a patient receiving mechanical ventilation indicates the need for suctioning? The patient's oxygen saturation is 93%. The patient's respiratory rate is 32 breaths/minute. The patient was last suctioned 6 hours ago. The patient has occasional audible expiratory wheezes.

The patient's respiratory rate is 32 breaths/minute.

A patient with right lower-lobe pneumonia has been treated with IV antibiotics for 3 days. Which assessment data obtained by the nurse indicates that the treatment has been effective? Selected Answer: The patient coughs up small amounts of green mucus. The patient's white blood cell (WBC) count is 9000/µL. Bronchial breath sounds are heard at the right base. Increased tactile fremitus is palpable over the right chest.

The patient's white blood cell (WBC) count is 9000/µL.

Which information will the nurse include in the asthma teaching plan for a patient being discharged? Hold your breath for 5 seconds after using the bronchodilator inhaler. Inhale slowly and deeply when using the dry powder inhaler (DPI). Use the inhaled corticosteroid when shortness of breath occurs. Tremors are an expected side effect of rapidly acting bronchodilator

Tremors are an expected side effect of rapidly acting bronchodilator

The nurse assesses a patient with a history of asthma. Which assessment finding indicates that the nurse should take immediate action? Use of accessory muscles in breathing Pulse oximetry reading of 91% Peak expiratory flow rate of 240 L/minute Respiratory rate of 26 breaths/minute

Use of accessory muscles in breathing

Following assessment of a patient with pneumonia, the nurse identifies a nursing diagnosis of ineffective airway clearance. Which assessment data best supports this diagnosis? Respiratory rate of 28 breaths/minute Resting pulse oximetry (SpO2) of 85% Weak, nonproductive cough effort Large amounts of greenish sputum

Weak, nonproductive cough effort

A patient is scheduled for pulmonary function testing. Which action should the nurse take to prepare the patient for this procedure? Administer oral corticosteroids 2 hours before the procedure. Give the rescue medication immediately before testing. Withhold bronchodilators for 6 to 12 hours before the examination. Ensure that the patient has been NPO for several hours before the test.

Withhold bronchodilators for 6 to 12 hours before the examination.

The nurse needs to take the blood pressure of a small child. Of the cuffs available, one is too large, and one is too small. The BEST nursing action is to: a. use the small cuff. b. use the large cuff. c. use either cuff, using palpation method. d. locate the proper size cuff before taking the blood pressure.

ANSWER: D The smaller cuff gives a falsely increased blood pressure and is not the method of choice. The larger cuff, which may give a falsely lowered blood pressure, is preferable to the smaller cuff, which gives a falsely increased blood pressure, but neither is the method of choice. Auscultation is preferred to palpation. To obtain an accurate blood pressure reading, it is preferable to use the proper-size cuff. Thus locating one before taking the blood pressure is the best nursing action.

The nurse notes thick, white secretions in the endotracheal tube (ET) of a patient who is receiving mechanical ventilation. Which intervention will be most effective in addressing this problem? Increase suctioning to every hour. Instill 5 mL of sterile saline into the ET before suctioning. Reposition the patient every 1 to 2 hours. . Add additional water to the patient's enteral feedings.

Add additional water to the patient's enteral feedings.

A patient seen in the asthma clinic has recorded daily peak flows that are 75% of the baseline. Which action will the nurse plan to take next? Instruct the patient to keep the next scheduled follow-up appointment. Increase the dose of the leukotriene inhibitor. Teach the patient about the use of oral corticosteroids. Administer a bronchodilator and recheck the peak flow.

Administer a bronchodilator and recheck the peak flow.

The nurse reviews the medication administration record (MAR) for a patient having an acute asthma attack. Which medication should the nurse administer first? Salmeterol (Serevent) 50 mcg per dry-powder inhaler (DPI) Methylprednisolone (Solu-Medrol) 60 mg IV Albuterol (Ventolin) 2.5 mg per nebulizer Triamcinolone (Azmacort) 2 puffs per metered-dose inhaler (MDI)

Albuterol (Ventolin) 2.5 mg per nebulizer

A patient has acute bronchitis with a nonproductive cough and wheezes. Which topic should the nurse plan to include in the teaching plan? Appropriate use of cough suppressants Safety concerns with home oxygen therapy Purpose of antibiotic therapy Ways to limit oral fluid intake

Appropriate use of cough suppressants

An alcoholic and homeless patient is diagnosed with active tuberculosis (TB). Which intervention by the nurse will be most effective in ensuring adherence with the treatment regimen? Arrange for a friend to administer the medication on schedule. Teach the patient about the high risk for infecting others unless treatment is followed. Arrange for a daily noon meal at a community center where the drug will be administered. Give the patient written instructions about how to take the medications.

Arrange for a daily noon meal at a community center where the drug will be administered.

While family members are visiting, a patient has a respiratory arrest and is being resuscitated. Which action by the nurse is best? Tell the family members that watching the resuscitation will be very stressful. Take the family members quickly out of the patient room and remain with them. Ask family members if they wish to remain in the room during the resuscitation. Assign a staff member to wait with family members just outside the patient room.

Ask family members if they wish to remain in the room during the resuscitation.

A patient has just been admitted with probable bacterial pneumonia and sepsis. Which order should the nurse implement first? Blood cultures from two sites Acetaminophen (Tylenol) rectal suppository Chest x-ray via stretcher Ciprofloxacin (Cipro) 400 mg IV

Blood cultures from two sites

The major advantage of a Venturi mask is that it can a. deliver up to 80% O2. b. provide continuous 100% humidity. c. deliver a precise concentration of O2. d. be used while a patient eats and sleeps.

C

The nurse can best determine adequate arterial oxygenation of the blood by assessing a. heart rate. b. hemoglobin level. c. arterial oxygen tension. d. arterial carbon dioxide tension.

C

Bed rest is prescribed for a client with pneumonia during the acute phase of the illness. The nurse should determine the effectiveness of bed rest by assessing the client's: 1. Decreased cellular demand for oxygen. 2. Reduced episodes of coughing. 3. Diminished pain when breathing deeply. 4. Ability to expectorate secretions more easily.

1. Exudate in the alveoli interferes with ventilation and the diffusion of gases in clients with pneumonia. During the acute phase of the illness, it is essential to reduce the body's need for oxygen at the cellular level; bed rest is the most effective method for doing so. Bed rest does not decrease coughing or promote clearance of secretions, and it does not reduce pain when taking deep breaths.

The nurse reviews an arterial blood gas report for a client with chronic obstructive pulmonary disease (COPD). pH 7.35; PC02 62; PO2 70; HCO3 34 The nurse should: 1. Apply a 100% non-rebreather mask. 2. Assess the vital signs. 3. Reposition the client. 4. Prepare for intubation.

2. Clients with chronic COPD have CO2 retention and the respiratory drive is stimulated when the PO2 decreases. The heart rate, respiratory rate, and blood pressure should be evaluated to determine if the client is hemodynamically stable. Symptoms, such as dyspnea, should also be assessed. Oxygen supplementation, if indicated, should be titrated upward in small increments. There is no indication that the client is experiencing respiratory distress requiring intubation.

When teaching a client with chronic obstructive pulmonary disease to conserve energy, the nurse should teach the client to lift objects: 1. While inhaling through an open mouth. 2. While exhaling through pursed lips. 3. After exhaling but before inhaling. 4. While taking a deep breath and holding it.

2. Exhaling requires less energy than inhaling. Therefore, lifting while exhaling saves energy and reduces perceived dyspnea. Pursing the lips prolongs exhalation and provides the client with more control over breathing. Lifting after exhaling but before inhaling is similar to lifting with the breath held. This should not be recommended because it is similar to the Valsalva maneuver, which can stimulate cardiac arrhythmias.

A client's arterial blood gas values are as follows: pH, 7.31; PaO2, 80 mm Hg; PaCO2, 65 mm Hg; HCO3 −, 36 mEq/ L. The nurse should assess the client for? 1. Cyanosis. 2. Flushed skin. 3. Irritability. 4. Anxiety.

2. The high PaCO2 level causes flushing due to vasodilation. The client also becomes drowsy and lethargic because carbon dioxide has a depressant effect on the central nervous system. Cyanosis is a sign of hypoxia. Irritability and anxiety are not common with a PaCO2 level of 65 mm Hg but are associated with hypoxia.

A client with acute asthma is prescribed short-term corticosteroid therapy. Which is the expected outcome for the use of steroids in clients with asthma? 1. Promote bronchodilation. 2. Act as an expectorant. 3. Have an anti-inflammatory effect. 4. Prevent development of respiratory infections.

3. Corticosteroids have an anti-inflammatory effect and act to decrease edema in the bronchial airways and decrease mucus secretion. Corticosteroids do not have a bronchodilator effect, act as expectorants, or prevent respiratory infections.

A nurse notes that a client has kyphosis and generalized muscle atrophy. Which of the following problems is a priority when the nurse develops a nursing plan of care? 1. Infection. 2. Confusion. 3. Ineffective coughing and deep breathing. 4. Difficulty chewing solid foods.

3. In kyphosis, the thoracic spine bends forward with convexity of the curve in a posterior direction, making effective coughing and deep breathing difficult. Although the client may develop other problems because respiratory status deteriorates when pulmonary secretions are not adequately cleared from airways, ineffective coughing and deep breathing should receive priority attention.

The nurse administers theophylline (Theo-Dur) to a client. To evaluate the effectiveness of this medication, which of the following drug actions should the nurse anticipate? 1. Suppression of the client's respiratory infection. 2. Decrease in bronchial secretions. 3. Relaxation of bronchial smooth muscle. 4. Thinning of tenacious, purulent sputum.

3. Theophylline (Theo-Dur) is a bronchodilator that is administered to relax airways and decrease dyspnea. Theophylline is not used to treat infections and does not decrease or thin secretions.

Which of the following measures would most likely be successful in reducing pleuritic chest pain in a client with pneumonia? 1. Encourage the client to breathe shallowly. 2. Have the client practice abdominal breathing. 3. Offer the client incentive spirometry. 4. Teach the client to splint the rib cage when coughing.

4. The pleuritic pain is triggered by chest movement and is particularly severe during coughing. Splinting the chest wall will help reduce the discomfort of coughing. Deep breathing is essential to prevent further atelectasis. Abdominal breathing is not as effective in decreasing pleuritic chest pain as is splinting of the rib cage. Incentive spirometry facilitates effective deep breathing but does not decrease pleuritic chest pain.

In evaluating an asthmatic patient's knowledge of self-care, the nurse recognizes that additional instruction is needed when the patient says, a. "I use my corticosteroid inhaler when I feel short of breath." b. "I get a flu shot every year and see my health care provider if I have an upper respiratory tract infection." c. "I use my inhaler before I visit my aunt who has a cat, but I only visit for a few minutes because of my allergies." d. "I walk 30 minutes every day but sometimes I have to use my bronchodilator inhaler before walking to prevent me from getting short of breath."

A

The nurse is scheduled to administer seasonal influenza vaccinations to the residents of a long-term care facility. Which of the following criteria would contraindicate the administration of the vaccine to a resident? A) Hypersensitivity to eggs B) Age greater than 80 years C) History of upper respiratory infections D) Chronic obstructive pulmonary disease (COPD)

A

To promote the release of surfactant, the nurse encourages the patient to a. take deep breaths. b. cough five times per hour to prevent alveolar collapse. c. decrease fluid intake to reduce fluid accumulation in the alveoli. d. sit with head of bed elevated to promote air movement through the pores of Kohn

A

When assessing activity-exercise patterns related to respiratory health, the nurse inquires about a. dyspnea during rest or exercise. b. recent weight loss or weight gain. c. ability to sleep through the entire night. d. willingness to wear oxygen equipment in public.

A

When teaching a patient about the most important respiratory defense mechanism distal to the respiratory bronchioles, which topic would the nurse discuss? a. Alveolar macrophages b. Impaction of particles c. Reflex bronchoconstriction d. Mucociliary clearance mechanism

A

A patient with bacterial pneumonia has rhonchi and thick sputum. What is the nurse's most appropriate action to promote airway clearance? a. Assist the patient to splint the chest when coughing. b. Teach the patient about the need for fluid restrictions. c. Encourage the patient to wear the nasal oxygen cannula. d. Instruct the patient on the pursed lip breathing technique.

ANS: A Coughing is less painful and more likely to be effective when the patient splints the chest during coughing. Fluids should be encouraged to help liquefy secretions. Nasal oxygen will improve gas exchange, but will not improve airway clearance. Pursed lip breathing is used to improve gas exchange in patients with COPD, but will not improve airway clearance.

On auscultation of a patient's lungs, the nurse hears low-pitched, bubbling sounds during inhalation in the lower third of both lungs. How should the nurse document this finding? a. Inspiratory crackles at the bases b. Expiratory wheezes in both lungs c. Abnormal lung sounds in the apices of both lungs d. Pleural friction rub in the right and left lower lobes

ANS: A Crackles are low-pitched, bubbling sounds usually heard on inspiration. Wheezes are high-pitched sounds. They can be heard during the expiratory or inspiratory phase of the respiratory cycle. The lower third of both lungs are the bases, not apices. Pleural friction rubs are grating sounds that are usually heard during both inspiration and expiration

The nurse assesses the chest of a patient with pneumococcal pneumonia. Which finding would the nurse expect? a. Increased tactile fremitus b. Dry, nonproductive cough c. Hyperresonance to percussion d. A grating sound on auscultation

ANS: A Increased tactile fremitus over the area of pulmonary consolidation is expected with bacterial pneumonias. Dullness to percussion would be expected. Pneumococcal pneumonia typically presents with a loose, productive cough. Adventitious breath sounds such as crackles and wheezes are typical. A grating sound is more representative of a pleural friction rub rather than pneumonia.

15. Which patient in the ear, nose, and throat (ENT) clinic should the nurse assess first? a. A 23-year-old who is complaining of a sore throat and has a muffled voice b. A 34-year-old who has a "scratchy throat" and a positive rapid strep antigen test c. A 55-year-old who is receiving radiation for throat cancer and has severe fatigue d. A 72-year-old with a history of a total laryngectomy whose stoma is red and inflamed

ANS: A The patient's clinical manifestation of a muffled voice suggests a possible peritonsillar abscess that could lead to an airway obstruction requiring rapid assessment and potential treatment. The other patients do not have diagnoses or symptoms that indicate any life-threatening problems.

A patient who was admitted the previous day with pneumonia complains of a sharp pain of 7 (based on 0 to 10 scale) "whenever I take a deep breath." Which action will the nurse take next? a. Auscultate breath sounds. b. Administer the PRN morphine. c. Have the patient cough forcefully. d. Notify the patient's health care provider.

ANS: A The patient's statement indicates that pleurisy or a pleural effusion may have developed and the nurse will need to listen for a pleural friction rub and/or decreased breath sounds. Assessment should occur before administration of pain medications. The patient is unlikely to be able to cough forcefully until pain medication has been administered. The nurse will want to obtain more assessment data before calling the health care provider

11. The nurse completes discharge instructions for a patient with a total laryngectomy. Which statement by the patient indicates that additional instruction is needed? a. "I must keep the stoma covered with an occlusive dressing at all times." b. "I can participate in most of my prior fitness activities except swimming." c. "I should wear a Medic-Alert bracelet that identifies me as a neck breather." d. "I need to be sure that I have smoke and carbon monoxide detectors installed."

ANS: A The stoma may be covered with clothing or a loose dressing, but this is not essential. An occlusive dressing will completely block the patient's airway. The other patient comments are all accurate and indicate that the teaching has been effective.

Following assessment of a patient with pneumonia, the nurse identifies a nursing diagnosis of ineffective airway clearance. Which assessment data best supports this diagnosis? a. Weak, nonproductive cough effort b. Large amounts of greenish sputum c. Respiratory rate of 28 breaths/minute d. Resting pulse oximetry (SpO2) of 85%

ANS: A The weak, nonproductive cough indicates that the patient is unable to clear the airway effectively. The other data would be used to support diagnoses such as impaired gas exchange and ineffective breathing pattern.

The nurse is caring for a patient who has a right-sided chest tube after a right lower lobectomy. Which nursing action can the nurse delegate to the unlicensed assistive personnel (UAP)? a. Document the amount of drainage every eight hours. b. Obtain samples of drainage for culture from the system. c. Assess patient pain level associated with the chest tube. d. Check the water-seal chamber for the correct fluid level.

ANS: A UAP education includes documentation of intake and output. The other actions are within the scope of practice and education of licensed nursing personnel.

7. Which statement by the patient indicates that the teaching has been effective for a patient scheduled for radiation therapy of the larynx? a. "I will need to buy a water bottle to carry with me." b. "I should not use any lotions on my neck and throat." c. "Until the radiation is complete, I may have diarrhea." d. "Alcohol-based mouthwashes will help clean oral ulcers."

ANS: A Xerostomia can be partially alleviated by drinking fluids at frequent intervals. Radiation will damage tissues at the site being radiated but should not affect the abdominal organs, so loose stools are not a usual complication of head and neck radiation therapy. Frequent oral rinsing with non-alcohol-based rinses is recommended. Prescribed lotions and sunscreen may be used on radiated skin, although they should not be used just before the radiation therapy.

A patient with bacterial pneumonia has rhonchi and thick sputum. Which action will the nurse use to promote airway clearance? a. Assist the patient to splint the chest when coughing. b. Educate the patient about the need for fluid restrictions. c. Encourage the patient to wear the nasal oxygen cannula. d. Instruct the patient on the pursed lip breathing technique.

ANS: A Coughing is less painful and more likely to be effective when the patient splints the chest during coughing. Fluids should be encouraged to help liquefy secretions. Nasal oxygen will improve gas exchange, but will not improve airway clearance. Pursed lip breathing is used to improve gas exchange in patients with COPD, but will not improve airway clearance. DIF: Cognitive Level: Application REF: 552-553 TOP: Nursing Process: Implementation MSC: NCLEX: Physiological Integrity

The nurse is caring for a patient with primary pulmonary hypertension who is receiving epoprostenol (Flolan). Which assessment information requires the most immediate action? a. The BP is 98/56 mm Hg. b. The oxygen saturation is 94%. c. The patient's central intravenous line is disconnected. d. The international normalized ratio (INR) is prolonged.

ANS: C The half-life of this drug is 6 minutes, so the nurse will need to restart the infusion as soon as possible to prevent rapid clinical deterioration. The other data also indicate a need for ongoing monitoring or intervention, but the priority action is to reconnect the infusion. DIF: Cognitive Level: Application REF: 581 OBJ: Special Questions: Prioritization TOP: Nursing Process: Assessment MSC: NCLEX: Physiological Integrity

After a patient with right lower-lobe pneumonia has been treated with intravenous (IV) antibiotics for 2 days, which assessment data obtained by the nurse indicates that the treatment has been effective? a. Bronchial breath sounds are heard at the right base. b. The patient coughs up small amounts of green mucus. c. The patient's white blood cell (WBC) count is 9000/µl. d. Increased tactile fremitus is palpable over the right chest.

ANS: C The normal WBC count indicates that the antibiotics have been effective. All the other data suggest that a change in treatment is needed. DIF: Cognitive Level: Application REF: 549 TOP: Nursing Process: Evaluation MSC: NCLEX: Physiological Integrity

After change-of-shift report, which patient should the nurse assess first? a. 72-year-old with cor pulmonale who has 4+ bilateral edema in his legs and feet b. 28-year-old with a history of a lung transplant and a temperature of 101° F (38.3° C) c. 40-year-old with a pleural effusion who is complaining of severe stabbing chest pain d. 64-year-old with lung cancer and tracheal deviation after subclavian catheter insertion

ANS: D The patient's history and symptoms suggest possible tension pneumothorax, a medical emergency. The other patients also require assessment as soon as possible, but tension pneumothorax will require immediate treatment to avoid death from inadequate cardiac output or hypoxemia.

4. The nurse carefully assesses the preterm infant for respiratory distress syndrome because of a deficiency of: a. protein. b. estrogen. c. hyaline. d. surfactant.

ANS: D The production of surfactant, necessary for the absorption of oxygen by the lungs, is deficient in the preterm infant.

The nurse prepares a patient with a left-sided pleural effusion for a thoracentesis. How should the nurse position the patient? a. Supine with the head of the bed elevated 30 degrees b. In a high-Fowler's position with the left arm extended c. On the right side with the left arm extended above the head d. Sitting upright with the arms supported on an over bed table

ANS: D The upright position with the arms supported increases lung expansion, allows fluid to collect at the lung bases, and expands the intercostal space so that access to the pleural space is easier. The other positions would increase the work of breathing for the patient and make it more difficult for the health care provider performing the thoracentesis

The laboratory has just called with the arterial blood gas (ABG) results on four patients. Which result is most important for the nurse to report immediately to the health care provider? a. pH 7.34, PaO2 82 mm Hg, PaCO2 40 mm Hg, and O2 sat 97% b. pH 7.35, PaO2 85 mm Hg, PaCO2 45 mm Hg, and O2 sat 95% c. pH 7.46, PaO2 90 mm Hg, PaCO2 32 mm Hg, and O2 sat 98% d. pH 7.31, PaO2 91 mm Hg, PaCO2 50 mm Hg, and O2 sat 96%

ANS: D These ABGs indicate uncompensated respiratory acidosis and should be reported to the health care provider. The other values are normal or close to normal.

The nurse palpates the posterior chest while the patient says "99" and notes absent fremitus. Which action should the nurse take next? a. Palpate the anterior chest and observe for barrel chest. b. Encourage the patient to turn, cough, and deep breathe. c. Review the chest x-ray report for evidence of pneumonia. d. Auscultate anterior and posterior breath sounds bilaterally.

ANS: D To assess for tactile fremitus, the nurse should use the palms of the hands to assess for vibration when the patient repeats a word or phrase such as "99." After noting absent fremitus, the nurse should then auscultate the lungs to assess for the presence or absence of breath sounds. Absent fremitus may be noted with pneumothorax or atelectasis. The vibration is increased in conditions such as pneumonia, lung tumors, thick bronchial secretions, and pleural effusion. Turning, coughing, and deep breathing is an appropriate intervention for atelectasis, but the nurse needs to first assess breath sounds. Fremitus is decreased if the hand is farther from the lung or the lung is hyperinflated (barrel chest).The anterior of the chest is more difficult to palpate for fremitus because of the presence of large muscles and breast tissue.

The health care provider writes an order for bacteriologic testing for a patient who has a positive tuberculosis skin test. Which action will the nurse take? a. Repeat the tuberculin skin testing. b. Teach about the reason for the blood tests. c. Obtain consecutive sputum specimens from the patient for 3 days. d. Instruct the patient to expectorate three specimens as soon as possible.

ANS: C Three consecutive sputum specimens are obtained on different days for bacteriologic testing for M. tuberculosis. The patient should not provide all the specimens at once. Blood cultures are not used for tuberculosis testing. Once skin testing is positive, it is not repeated. DIF: Cognitive Level: Application REF: 555 TOP: Nursing Process: Implementation MSC: NCLEX: Physiological Integrity

What explains the importance of detecting strabismus in young children? a. Color vision deficit may result. b. Amblyopia, a type of blindness, may result. c. Epicanthal folds may develop in affected eye. d. Ptosis may develop secondarily.

ANSWER: B Color vision depends on rods and cones in the retina, not muscle coordination. Amblyopia may develop if the eyes do not work together. The brain may ignore the visual cues from one eye, resulting in blindness. Epicanthal folds are present at birth. Ptosis, or drooping eyelids, is not related to strabismus (or cross-eyes).

After 2 months of tuberculosis (TB) treatment with isoniazid (INH), rifampin (Rifadin), pyrazinamide (PZA), and ethambutol, a patient continues to have positive sputum smears for acid-fast bacilli (AFB). Which action should the nurse take next? Schedule the patient for directly observed therapy three times weekly. Teach about treatment for drug-resistant TB treatment. Ask the patient whether medications have been taken as directed. Discuss with the health care provider the need for the patient to use an injectable antibiotic.

Ask the patient whether medications have been taken as directed.

A patient with bacterial pneumonia has rhonchi and thick sputum. What is the nurse's most appropriate action to promote airway clearance? Instruct the patient on the pursed lip breathing technique. Assist the patient to splint the chest when coughing. Teach the patient about the need for fluid restrictions. Encourage the patient to wear the nasal oxygen cannula.

Assist the patient to splint the chest when coughing.

The patient had abdominal surgery yesterday. Today the lung sounds in the lower lobes have decreased. The nurse knows this could be due to what occurring?

Atelectasis RATIONALE: Postoperatively there is an increased risk for atelectasis from anesthesia as well as restricted breathing from pain. Without deep breathing to stretch the alveoli, surfactant secretion to hold the alveoli open is not promoted. Pneumonia will occur later after surgery. Pleural effusion occurs because of blockage of lymphatic drainage or an imbalance between intravascular and oncotic fluid pressures, which is not expected in this case.

The effects of cigarette smoking on the respiratory system include a. hypertrophy of capillaries causing hemoptysis. b. hyperplasia of goblet cells and increased production of mucus. c. increased proliferation of cilia and decreased clearance of mucus. d. proliferation of alveolar macrophages to decrease the risk for infection.

B

To detect early signs or symptoms of inadequate oxygenation, the nurse would examine the patient for a. dyspnea and hypotension. b. apprehension and restlessness. c. cyanosis and cool, clammy skin. d. increased urine output and diaphoresis.

B

When auscultating the chest of an older patient in respiratory distress, it is best to a. begin listening at the apices. b. begin listening at the lung bases. c. begin listening on the anterior chest. d. ask the patient to breathe through the nose with the mouth closed.

B

When caring for a patient who is 3 hours postoperative laryngectomy, the nurse's highest priority assessment would be A) Patient comfort. B) Airway patency. C) Incisional drainage. D) Blood pressure and heart rate.

B

Which of the following tasks can the registered nurse (RN) delegate to nursing assistive personnel (NAP) in the care of a stable patient who has a tracheostomy? A) Assessing the need for suctioning B) Suctioning the patient's oropharynx C) Assessing the patient's swallowing ability D) Maintaining appropriate cuff inflation pressure

B

Which information obtained by the nurse about a patient who has been diagnosed with both human immunodeficiency virus (HIV) and active tuberculosis (TB) disease is most important to communicate to the health care provider? a. The Mantoux test had an induration of only 8 mm. b. The chest-x-ray showed infiltrates in the upper lobes. c. The patient is being treated with antiretrovirals for HIV infection. d. The patient has a cough that is productive of blood-tinged mucus.

ANS: C Drug interactions can occur between the antiretrovirals used to treat HIV infection and the medications used to treat tuberculosis. The other data are expected in a patient with HIV and TB disease. DIF: Cognitive Level: Application REF: 556 OBJ: Special Questions: Prioritization TOP: Nursing Process: Assessment

The nurse analyzes the results of a patient's arterial blood gases (ABGs). Which finding would require immediate action? a. The bicarbonate level (HCO3-) is 31 mEq/L. b. The arterial oxygen saturation (SaO2) is 92%. c. The partial pressure of CO2 in arterial blood (PaCO2) is 31 mm Hg. d. The partial pressure of oxygen in arterial blood (PaO2) is 59 mm Hg.

ANS: D All the values are abnormal, but the low PaO2 indicates that the patient is at the point on the oxyhemoglobin dissociation curve where a small change in the PaO2 will cause a large drop in the O2 saturation and a decrease in tissue oxygenation. The nurse should intervene immediately to improve the patient's oxygenation.

A patient with pneumonia has a fever of 101.4° F (38.6° C), a nonproductive cough, and an oxygen saturation of 88%. The patient complains of weakness, fatigue, and needs assistance to get out of bed. Which nursing diagnosis should the nurse assign as the highest priority? a. Hyperthermia related to infectious illness b. Impaired transfer ability related to weakness c. Ineffective airway clearance related to thick secretions d. Impaired gas exchange related to respiratory congestion

ANS: D All these nursing diagnoses are appropriate for the patient, but the patient's oxygen saturation indicates that all body tissues are at risk for hypoxia unless the gas exchange is improved.

20. A patient arrives in the ear, nose, and throat clinic complaining of a piece of tissue being "stuck up my nose" and with foul-smelling nasal drainage from the right nare. Which action should the nurse take first? a. Notify the clinic health care provider. b. Obtain aerobic culture specimens of the drainage. c. Ask the patient about how the cotton got into the nose. d. Have the patient occlude the left nare and blow the nose.

ANS: D Because the highest priority action is to remove the foreign object from the nare, the nurse's first action should be to assist the patient to remove the object. The other actions are also appropriate but should be done after attempting to clear the nose.

When assessing the respiratory system of an older patient, which finding indicates that the nurse should take immediate action? a. Weak cough effort b. Barrel-shaped chest c. Dry mucous membranes d. Bilateral crackles at lung bases

ANS: D Crackles in the lower half of the lungs indicate that the patient may have an acute problem such as heart failure. The nurse should immediately accomplish further assessments, such as oxygen saturation, and notify the health care provider. A barrel-shaped chest, hyperresonance to percussion, and a weak cough effort are associated with aging. Further evaluation may be needed, but immediate action is not indicated. An older patient has a less forceful cough and fewer and less functional cilia. Mucous membranes tend to be drier.

When assessing a patient who has just arrived after an automobile accident, the emergency department nurse notes tachycardia and absent breath sounds over the right lung. For which intervention will the nurse prepare the patient? a. Emergency pericardiocentesis b. Stabilization of the chest wall with tape c. Administration of an inhaled bronchodilator d. Insertion of a chest tube with a chest drainage system

ANS: D The patient's history and absent breath sounds suggest a right-sided pneumothorax or hemothorax, which will require treatment with a chest tube and drainage. The other therapies would be appropriate for an acute asthma attack, flail chest, or cardiac tamponade, but the patient's clinical manifestations are not consistent with these problems

Which of the following is the priority nursing assessment in the care of a patient who has a tracheostomy? A) Electrolyte levels and daily weights B) Assessment of speech and swallowing C) Respiratory rate and oxygen saturation D) Pain assessment and assessment of mobility

C

A patient is admitted with active tuberculosis (TB). The nurse should question a health care provider's order to discontinue airborne precautions unless which assessment finding is documented? A. Chest x-ray shows no upper lobe infiltrates. B. TB medications have been taken for 6 months. C. Mantoux testing shows an induration of 10 mm. D. Three sputum smears for acid-fast bacilli are negative.

D

Which assessment finding of the respiratory system does the nurse interpret as abnormal? a. Inspiratory chest expansion of 1 in b. Percussion resonance over the lung bases c. Symmetric chest expansion and contraction d. Bronchial breath sounds in the lower lung fields

D

The RN has conducted discharge teaching w/ a pt dx with TB, who has been receiving medication for 1.5 wks. The RN determines that the pt has understood the information if the client makes which statement? a) I need to cont. drug therapy for 2 months. b) I can't shop at the mall for the next 6 months c) I can return to work if a sputum culture comes back neg. d) I should not be contagious after 2 to 3 wks of medication therapy

D Medication therapy is 6-12 months. Pt generally considered noncontagious after 2-3 wks of med therapy. Wear a mask if exposed to crowds until med is effective. Allowed to return to work after three neg sputum cultures.

One week after a thoracotomy, a patient with chest tubes (CTs) to water-seal drainage has an air leak into the closed chest drainage system (CDS). Which patient assessment warrants follow-up nursing interventions?

Chest tube with a loose-fitting dressing RATIONALE: If the dressing at the CT insertion site is loose, an air leak will occur and will need to be sealed. The water-seal chamber usually has 2 cm of water, but having more water will not contribute to an air leak, and it should not be drained from the CDS. No new drainage does not indicate an air leak but may indicate the CT is no longer needed. If there is a pneumothorax, the chest tube should remove the air.

The nurse, when auscultating the lower lungs of the patient, hears these breath sounds. How should the nurse document these sounds?

Coarse crackles RATIONALE: Coarse crackles are a series of long-duration, discontinuous, low-pitched sounds caused by air passing through an airway intermittently occluded by mucus, an unstable bronchial wall, or a fold of mucosa. Coarse crackles are evident on inspiration and at times expiration. Stridor is a continuous crowing sound of constant pitch from partial obstruction of larynx or trachea. Rhonchi are a continuous rumbling, snoring, or rattling sound from obstruction of large airways with secretions. Bronchovesicular sounds are normal sounds heard anteriorly over the mainstem bronchi on either side of the sternum and posteriorly between the scapulae with a medium pitch and intensity.

Which finding in a patient hospitalized with bronchiectasis is most important to report to the health care provider? Scattered rhonchi and wheezes heard bilaterally Complaint of sharp chest pain with deep breathing Respiratory rate 28 breaths/minute while ambulating in hallway Cough productive of bloody, purulent mucus

Cough productive of bloody, purulent mucus

The patient is calling the clinic with a cough. What assessment should be made first before the nurse advises the patient?

Cough sound, sputum production, pattern RATIONALE: The sound of the cough, sputum production and description, as well as pattern of the cough's occurrence (including acute or chronic) and what its occurrence is related to are the first assessments to be made to determine the severity. Frequency of the cough will not provide a lot of information. Family history can help to determine a genetic cause of the cough. Hematemesis is vomiting blood and not as important as hemoptysis. Smoking is an important risk factor for COPD and lung cancer and may cause a cough. Medications may or may not contribute to a cough as does residence location. Weight loss, activity intolerance, and orthopnea may be related to respiratory or cardiac problems, but are not as important when dealing with a cough.

The patient with Parkinson's disease has a pulse oximetry reading of 72%, but he is not displaying any other signs of decreased oxygenation. What is most likely contributing to his low SpO2 level?

Motion RATIONALE: Motion is the most likely cause of the low SpO2 for this patient with Parkinson's disease. Anemia, dark skin color, and thick acrylic nails as well as low perfusion, bright fluorescent lights, and intravascular dyes may also cause an inaccurate pulse oximetry result. There is no mention of these or reason to suspect these in this question.

During an assessment of a 45-year-old patient with asthma, the nurse notes wheezing and dyspnea. The nurse interprets that these symptoms are related to what pathophysiologic change? Laryngospasm Pulmonary edema Narrowing of the airway Overdistention of the alveoli

Narrowing of the airway Narrowing of the airway by persistent but variable inflammation leads to reduced airflow, making it difficult for the patient to breathe and producing the characteristic wheezing. Laryngospasm, pulmonary edema, and overdistention of the alveoli do not produce wheezing

When admitting a 45-year-old female with a diagnosis of pulmonary embolism, the nurse will assess the patient for which risk factors (select all that apply)?

Obesity Correct Malignancy Correct Cigarette smoking Correct Prolonged air travel RATIONALE: An increased risk of pulmonary embolism is associated with obesity, malignancy, heavy cigarette smoking, and prolonged air travel with reduced mobility. Other risk factors include deep vein thrombosis, immobilization, surgery within the last 3 months, oral contraceptives and hormone therapy, heart failure, pregnancy, and clotting disorders.

The nurse administers prescribed therapies for a patient with cor pulmonale and right-sided heart failure. Which assessment would best evaluate the effectiveness of the therapies? Palpate for heaves or thrills over the heart. Auscultate for crackles in the lungs. Observe for distended neck veins. Review hemoglobin and hematocrit values.

Observe for distended neck veins.

While ambulating a patient with metastatic lung cancer, the nurse observes a drop in oxygen saturation from 93% to 86%. Which nursing intervention is most appropriate based upon these findings?

Obtain a physician's order for supplemental oxygen to be used during ambulation and other activity. RATIONALE: An oxygen saturation level that drops below 90% with activity indicates that the patient is not tolerating the exercise and needs to use supplemental oxygen. The patient will need to rest to resaturate. ABGs or moving the probe will not be needed as the pulse oximeter was working at the beginning of the walk.

The patient has decided to use the voice rehabilitation that offers the best speech quality even though it must be cleaned regularly. The nurse knows that this is what kind of voice rehabilitation? A. Electromyograph B. Intraoral electrolarynx C. Neck type electrolarynx D. Transesophageal puncture

D) The transesophageal puncture provides a fistula between the esophagus and trachea with a one-way valved prosthesis to prevent aspiration from the esophagus to the trachea. Air moves from the lungs, vibrates against the esophagus, and words are formed with the tongue and lips as the air moves out the mouth. The electromyography and both electrolarynx methods produce low-pitched mechanical sounds.

The nurse, who has administered a first dose of oral prednisone to a patient with asthma, writes on the care plan to begin monitoring for which patient parameters? Apical pulse Daily weight Bowel sounds Deep tendon reflexes

Daily weight Correct Corticosteroids such as prednisone can lead to weight gain. For this reason, it is important to monitor the patient's daily weight. The drug should not affect the apical pulse, bowel sounds, or deep tendon reflexes.

The patient had video-assisted thoracic surgery (VATS) to perform a lobectomy. What does the nurse know is the reason for using this type of surgery?

Less discomfort and faster return to normal activity RATIONALE: The VATS procedure uses minimally invasive incisions that cause less discomfort and allow faster healing and return to normal activity as well as lower morbidity risk and fewer complications. Many surgeries can be done for lung cancer, but pneumonectomy via thoracotomy is the most common surgery for lung cancer. The incision for a thoracotomy is commonly a medial sternotomy or a lateral approach. A chest tube will be needed postoperatively for VATS.

To promote airway clearance in a patient with pneumonia, what should the nurse instruct the patient to do (select all that apply)?

Maintain adequate fluid intake. Correct Splint the chest when coughing. Correct Instruct patient to cough at end of exhalation.* RATIONALE: Maintaining adequate fluid intake liquefies secretions, allowing easier expectoration. The nurse should instruct the patient to splint the chest while coughing. This will reduce discomfort and allow for a more effective cough. Coughing at the end of exhalation promotes a more effective cough. The patient should be positioned in an upright sitting position (high Fowler's) with head slightly flexed.

The nurse notes that a patient has incisional pain, a poor cough effort, and scattered rhonchi after a thoracotomy. Which action should the nurse take first? Observe the patient use the incentive spirometer. Medicate the patient with prescribed morphine. Assist the patient to sit upright in a chair. Splint the patient's chest during coughing.

Medicate the patient with prescribed morphine.

A patient with recurrent shortness of breath has just had a bronchoscopy. What is a priority nursing action immediately following the procedure?

Monitor the patient for laryngeal edema. RATIONALE: Priorities for assessment are the patient's airway and breathing, both of which may be compromised after bronchoscopy by laryngeal edema. These assessment parameters supersede the importance of loss of consciousness (LOC), pain, heart rate, and blood pressure, although the nurse should also be assessing these.

During an assessment of a 45-year-old patient with asthma, the nurse notes wheezing and dyspnea. The nurse interprets that these symptoms are related to what pathophysiologic change? a) Laryngospasm b) Pulmonary edema c) Narrowing of the airway d) Overdistention of the alveoli

c) Narrowing of the airway Narrowing of the airway by persistent but variable inflammation leads to reduced airflow, making it difficult for the patient to breathe and producing the characteristic wheezing. Laryngospasm, pulmonary edema, and overdistention of the alveoli do not produce wheezing.

When the patient is diagnosed with a lung abscess, what should the nurse teach the patient?

Oral antibiotics will be used when the patient and x-ray shows evidence of improvement. RATIONALE: IV antibiotics are used until the patient and x-ray show evidence of improvement. Then oral antibiotics are used for a prolonged period of time. Culture and sensitivity testing is done during the course of antibiotic therapy to ensure that the infecting organism is not becoming resistant to the antibiotic as well as at the completion of the antibiotic therapy. Lobectomy surgery is only needed when reinfection of a large cavitary lesion occurs or to establish a diagnosis when there is evidence of a neoplasm or other underlying problem.

A patient with a pleural effusion is scheduled for a thoracentesis. Which action should the nurse take to prepare the patient for the procedure? Remove the water pitcher and remind the patient not to eat or drink anything for 6 hours. Start a peripheral IV line to administer the necessary sedative drugs. Obtain a large collection device to hold 2 to 3 liters of pleural fluid at one time. Position the patient sitting upright on the edge of the bed and leaning forward.

Position the patient sitting upright on the edge of the bed and leaning forward.

What nursing intervention is most appropriate to enhance oxygenation in a patient with unilateral malignant lung disease?

Positioning patient with "good lung" down RATIONALE: Therapeutic positioning identifies the best position for the patient, thus assuring stable oxygenation status. Research indicates that positioning the patient with the unaffected lung (good lung) dependent best promotes oxygenation in patients with unilateral lung disease. For bilateral lung disease, the right lung down has best ventilation and perfusion. Increasing fluid intake and performing postural drainage will facilitate airway clearance, but positioning is most appropriate to enhance oxygenation.

A pt who is HIV-pos has had a TB skin test. The RN notes a 7mm area of induration at the site of the skin test and interprets the result as which finding? Positive Negative Inconclusive Need for repeat testing

Positive The pt w/ HIV is considered to have a pos skin test with area of induration larger than 5mm. A pt w/o HIV is positive with an induration larger than 10mm. HIV makes a pt immunosuppressed.

A patient with a recent history of a dry cough has had a chest x-ray that revealed the presence of nodules. In an effort to determine whether the nodules are malignant or benign, what is the primary care provider likely to order?

Positron emission tomography (PET) RATIONALE: PET is used to distinguish benign and malignant pulmonary nodules. Because malignant lung cells have an increased uptake of glucose, the PET scan (which uses an IV radioactive glucose preparation) can demonstrate increased uptake of glucose in malignant lung cells. This differentiation cannot be made using CT, a pulmonary angiogram, or thoracentesis.

The nurse teaches pursed lip breathing to a patient who is newly diagnosed with chronic obstructive pulmonary disease (COPD). The nurse reinforces that this technique will assist respiration by which mechanism? Loosening secretions so that they may be coughed up more easily Promoting maximal inhalation for better oxygenation of the lungs Preventing bronchial collapse and air trapping in the lungs during exhalation Increasing the respiratory rate and giving the patient control of respiratory patterns

Preventing bronchial collapse and air trapping in the lungs during exhalation The purpose of pursed lip breathing is to slow down the exhalation phase of respiration, which decreases bronchial collapse and subsequent air trapping in the lungs during exhalation. It does not affect secretions, inhalation, or increase the rate of breathing.

The nurse determines that the patient is not experiencing adverse effects of albuterol (Proventil) after noting which patient vital sign? Pulse rate of 72/minute Temperature of 98.4° F Oxygen saturation 96% Respiratory rate of 18/minute

Pulse rate of 72/minute Correct Albuterol is a β2-agonist that can sometimes cause adverse cardiovascular effects. These would include tachycardia and angina. A pulse rate of 72 indicates that the patient did not experience tachycardia as an adverse effect.

The mother of an infant diagnosed with bronchiolitis asks the nurse what causes this disease. The nurse's response would be based on the knowledge that the majority of infections that cause bronchiolitis are a result of:

RSV

When the patient is experiencing metabolic acidosis secondary to type 1 diabetes mellitus, what physiologic response should the nurse expect to assess in the patient?

Rapid respiratory rate RATIONALE: When a patient with type 1 diabetes has hyperglycemia and ketonemia causing metabolic acidosis, the physiologic response is to increase the respiratory rate and tidal volume to blow off the excess CO2. Vomiting and increased urination may occur with hyperglycemia, but not as physiologic responses to metabolic acidosis. The heart rate will increase.

An occupational health nurse works at a manufacturing plant where there is potential exposure to inhaled dust. Which action, if recommended by the nurse, will be most helpful in reducing the incidence of lung disease? Teach about symptoms of lung disease. Require the use of protective equipment. Treat workers with pulmonary fibrosis. Monitor workers for coughing and wheezing.

Require the use of protective equipment.

The patient's arterial blood gas results show the PaO2 at 65 mmHg and the SaO2 at 80%. What early manifestations should the nurse expect to observe in this patient?

Restlessness, tachypnea, tachycardia, and diaphoresis RATIONALE: With inadequate oxygenation, early manifestations include restlessness, tachypnea, tachycardia, and diaphoresis, decreased urinary output, and unexplained fatigue. The unexplained confusion, dyspnea at rest, hypotension, and diaphoresis; combativeness, retractions with breathing, cyanosis, and decreased urinary output; coma, accessory muscle use, cool and clammy skin, and unexplained fatigue occur as later manifestations of inadequate oxygenation. P479

The nurse is assisting a patient to learn self-administration of beclomethasone, two puffs inhaled every 6 hours. What should the nurse explain as the best way to prevent oral infection while taking this medication? Chew a hard candy before the first puff of medication. Rinse the mouth with water before each puff of medication. Ask for a breath mint following the second puff of medication. Rinse the mouth with water following the second puff of medication.

Rinse the mouth with water following the second puff of medication. Because beclamethosone is a corticosteroid, the patient should rinse the mouth with water following the second puff of medication to reduce the risk of fungal overgrowth and oral infection.

The nurse is caring for a 73-year-old patient who underwent a left total knee arthroplasty. On the third postoperative day, the patient complains of shortness of breath, slight chest pain, and that "something is wrong." Temperature is 98.4° F, blood pressure 130/88, respirations 36/minute, and oxygen saturation 91% on room air. What action should the nurse take first?

Sit the patient up in bed as tolerated and apply oxygen. RATIONALE: The patient's clinical picture is most likely pulmonary embolus, and the first action the nurse takes should be to assist with the patient's respirations. For this reason, the nurse should sit the patient up as tolerated and apply oxygen before notifying the physician. The nitroglycerin tablet would not be helpful, and the oxygenation status is a bigger problem than the slight chest pain at this time.

The RN performs an admission assessment on a client w/ a dx of TB. The RN should check the results of which diagnostic test that will confirm this dx? Chest x-ray Bronchoscopy Sputum Culture TB skin test

Sputum culture TB is definitively diagnosed through culture and isolation of Mycobacterium tuberculosis.

The nurse completes an admission assessment on a patient with asthma. Which information given by patient is most indicative of a need for a change in therapy? The patient's heart rate increases after using the albuterol (Proventil) inhaler. The patient uses albuterol (Proventil) before any aerobic exercise. The patient's only medications are albuterol (Proventil) and salmeterol (Serevent). The patient says that the asthma symptoms are worse every spring.

The patient's only medications are albuterol (Proventil) and salmeterol (Serevent).

During the assessment in the ED, the nurse is palpating the patient's chest. Which finding is a medical emergency?

Trachea moved to the left RATIONALE: Tracheal deviation is a medical emergency when it is caused by a tension pneumothorax. Tactile fremitus increases with pneumonia or pulmonary edema and decreases in pleural effusion or lung hyperinflation. Diminished chest movement occurs with barrel chest, restrictive disease, and neuromuscular disease.

In assessment of the patient with acute respiratory distress, what should the nurse expect to observe (select all that apply)?

Tripod position Accessory muscle use RATIONALE: Tripod position and accessory muscle use indicate moderate to severe respiratory distress. Cyanosis may be related to anemia, decreased oxygen transfer in the lungs, or decreased cardiac output. Therefore it is a nonspecific and unreliable indicator of only respiratory distress. Kussmaul respirations occur when the patient is in metabolic acidosis to increase CO2 excretion. Increased AP diameter occurs with lung hyperinflation from COPD, cystic fibrosis, or with advanced age.

Employee health test results reveal a tuberculosis (TB) skin test of 16-mm induration and a negative chest x-ray for a staff nurse working on the pulmonary unit. The nurse has no symptoms of TB. Which information should the occupational health nurse plan to teach the staff nurse? Need for annual repeat TB skin testing Use and side effects of isoniazid (INH) Bacille Calmette-Guérin (BCG) vaccine Standard four-drug therapy for TB

Use and side effects of isoniazid (INH)

A patient with an acute exacerbation of chronic obstructive pulmonary disease (COPD) needs to receive precise amounts of oxygen. Which equipment should the nurse prepare to use? Oxygen tent Venturi mask Nasal cannula Oxygen-conserving cannula

Venturi mask The Venturi mask delivers precise concentrations of oxygen and should be selected whenever this is a priority concern. The other methods are less precise in terms of amount of oxygen delivered.

Before discharge, the nurse discusses activity levels with a 61-year-old patient with chronic obstructive pulmonary disease (COPD) and pneumonia. Which exercise goal is most appropriate once the patient is fully recovered from this episode of illness? Slightly increase activity over the current level. Swim for 10 min/day, gradually increasing to 30 min/day. Limit exercise to activities of daily living to conserve energy. Walk for 20 min/day, keeping the pulse rate less than 130 beats/min.

Walk for 20 min/day, keeping the pulse rate less than 130 beats/min. The patient will benefit from mild aerobic exercise that does not stress the cardiorespiratory system. The patient should be encouraged to walk for 20 min/day, keeping the pulse rate less than 75% to 80% of maximum heart rate (220 - patient's age).

The nurse is caring for a patient with an acute exacerbation of asthma. Following initial treatment, what finding indicates to the nurse that the patient's respiratory status is improving? Wheezing becomes louder. Cough remains nonproductive. Vesicular breath sounds decrease. Aerosol bronchodilators stimulate coughing.

Wheezing becomes louder. The primary problem during an exacerbation of asthma is narrowing of the airway and subsequent diminished air exchange. As the airways begin to dilate, wheezing gets louder because of better air exchange. Vesicular breath sounds will increase with improved respiratory status. After a severe asthma exacerbation, the cough may be productive and stringy. Coughing after aerosol bronchodilators may indicate a problem with the inhaler or its use.

The nurse identifies the nursing diagnosis of activity intolerance for a patient with asthma. In patients with asthma, the nurse assesses for which etiologic factor for this nursing diagnosis? Work of breathing Fear of suffocation Effects of medications Anxiety and restlessness

Work of breathing When the patient does not have sufficient gas exchange to engage in activity, the etiologic factor is often the work of breathing. When patients with asthma do not have effective respirations, they use all available energy to breathe and have little left over for purposeful activity. Fear of suffocation, effects of medications or anxiety, and restlessness are not etiologies for activity intolerance for a patient with asthma.

A plan of care for the patient with COPD could include (select all that apply): a. exercise such as walking b. high flow rate of O2 administration c. low-dose chronic oral corticosteroid therapy d. use of peak flow meter to monitor the progression of COPD e. breathing exercises such as pursed-lip breathing that focus on exhalation

a & e

In evaluating an asthmatic patient's knowledge of self-care, the nurse recognizes that additional instruction is needed when the patient says, a. "I use my corticosteroid inhaler when I feel short of breath." b. "I get a flu shot every year and see my health care provider if I have an upper respiratory tract infection." c. "I use my inhaler before I visit my aunt who has a cat, but I only visit for a few minutes because of my allergies." d. "I walk 30 minutes every day but sometimes I have to use my bronchodilator inhaler before walking to prevent me from getting short of breath."

a. "I use my corticosteroid inhaler when I feel short of breath."

A patient who has bronchiectasis asks the nurse, "What conditions would warrant a call to the clinic?" a. Blood clots in the sputum b. Sticky sputum on a hot day c. Increased shortness of breath after eating a large meal d. Production of large amounts of sputum on a daily basis

a. Blood clots in the sputum

Which of the following interventions is most appropriate for helping parents to cope with a child newly diagnosed with bronchopulmonary dysplasia? a. Teach cardiopulmonary resuscitation b. Refer them to support groups c. Help parents identify necessary lifestyle changes d. Evaluate and assess parents; stress and anxiety levels

a. Teach cardiopulmonary resuscitation

To prevent excessive pressure on tracheal capillaries, pressure in the cuff on a tracheostomy tube should be a. monitored every 2-3 days b. less than 20 mm Hg or 25 cm H2O c. less than 30 mm Hg or 35 cm H2O d. sufficient to fill the pilot balloon until it is tense

b higher pressures may compress tracheal capillaries, limit blood flow, and predispose to tracheal necrosis.

The nurse is teaching a patient how to self-administer ipratropium (Atrovent) via a metered dose inhaler (MDI). Which instruction given by the nurse is most appropriate to help the patient learn the proper inhalation technique? a) "Avoid shaking the inhaler before use." b) "Breathe out slowly before positioning the inhaler." c) "Using a spacer should be avoided for this type of medication." d) "After taking a puff, hold the breath for 30 seconds before exhaling."

b) "Breathe out slowly before positioning the inhaler." It is important to breathe out slowly before positioning the inhaler. This allows the patient to take a deeper breath while inhaling the medication, thus enhancing the effectiveness of the dose. The inhaler should be shaken well. A spacer may be used. Holding the breath after the inhalation of medication helps keep the medication in the lungs, but 30 seconds will not be possible for a patient with COPD.

The nurse determines that the patient understood medication instructions about the use of a spacer device when taking inhaled medications after hearing the patient state what as the primary benefit? a) "I will pay less for medication because it will last longer." b) "More of the medication will get down into my lungs to help my breathing." c) "Now I will not need to breathe in as deeply when taking the inhaler medications." d) "This device will make it so much easier and faster to take my inhaled medications."

b) "More of the medication will get down into my lungs to help my breathing." A spacer assists more medication to reach the lungs, with less being deposited in the mouth and the back of the throat. It does not affect the cost or increase the speed of using the inhaler.

Which test result identifies that a patient with asthma is responding to treatment? a) An increase in CO2 levels b) A decreased exhaled nitric oxide c) A decrease in white blood cell count d) An increase in serum bicarbonate levels

b) A decreased exhaled nitric oxide Nitric oxide levels are increased in the breath of people with asthma. A decrease in the exhaled nitric oxide concentration suggests that the treatment may be decreasing the lung inflammation associated with asthma and adherence to treatment. An increase in CO2 levels, decreased white blood cell count, and increased serum bicarbonate levels do not indicate a positive response to treatment in the asthma patient.

The nurse, who has administered a first dose of oral prednisone to a patient with asthma, writes on the care plan to begin monitoring for which patient parameters? a) Apical pulse b) Daily weight c) Bowel sounds d) Deep tendon reflexes

b) Daily weight Corticosteroids such as prednisone can lead to weight gain. For this reason, it is important to monitor the patient's daily weight. The drug should not affect the apical pulse, bowel sounds, or deep tendon reflexes.

When planning teaching for the patient with chronic obstructive pulmonary disease (COPD), the nurse understands that what causes the manifestations of the disease? a) An overproduction of the antiprotease α1 -antitrypsin b) Hyperinflation of alveoli and destruction of alveolar walls c) Hypertrophy and hyperplasia of goblet cells in the bronchi d) Collapse and hypoventilation of the terminal respiratory unit

b) Hyperinflation of alveoli and destruction of alveolar walls In COPD there are structural changes that include hyperinflation of alveoli, destruction of alveolar walls, destruction of alveolar capillary walls, narrowing of small airways, and loss of lung elasticity. An autosomal recessive deficiency of antitrypsin may cause COPD. Not all patients with COPD have excess mucus production by the increased number of goblet cells.

The nurse is evaluating if a patient understands how to safely determine whether a metered dose inhaler (MDI) is empty. The nurse interprets that the patient understands this important information to prevent medication underdosing when the patient describes which method to check the inhaler? a) Place it in water to see if it floats. b) Keep track of the number of inhalations used. c) Shake the canister while holding it next to the ear d) Check the indicator line on the side of the canister.

b) Keep track of the number of inhalations used. It is no longer appropriate to see if a canister floats in water or not since this is not an accurate way to determine the remaining inhaler doses. The best method to determine when to replace an inhaler is by knowing the maximum puffs available per MDI and then replacing it after the number of days when those inhalations have been used. (100 puffs/2 puffs each day = 50 days)

When admitting a patient with a diagnosis of asthma exacerbation, the nurse will assess for what potential triggers (select all that apply)? a) Exercise b) Allergies c) Emotional stress d) Decreased humidity e) Upper respiratory infections

a, b, c, & e Although the exact mechanism of asthma is unknown, there are several triggers that may precipitate an attack. These include allergens, exercise, air pollutants, upper respiratory infections, drug and food additives, psychologic factors, and gastroesophageal reflux disease (GERD).

A patient is concerned that he may have asthma. Of the symptoms that he relates to the nurse, which ones suggest asthma or risk factors for asthma (select all that apply)? a. Allergic rhinitis b. Prolonged inhalation c. History of skin allergies d. Cough, especially at night e. Gastric reflux or heartburn

a, c, d, & e

Which treatments in CF would the nurse expect to implement in the management plan of patients with CF (select all that apply)? a. sperm banking b. IV corticosteroids on a chronic basis c. Airway clearance techniques (e.g., Acapella) d. GoLYTELY given PRN for severe constipation e. Inhaled tobramycin to combat Pseudomonas infection

a, c, d, & e

The nurse is teaching a patient how to self-administer ipratropium (Atrovent) via a metered dose inhaler (MDI). Which instruction given by the nurse is most appropriate to help the patient learn the proper inhalation technique? "Avoid shaking the inhaler before use." "Breathe out slowly before positioning the inhaler." "Using a spacer should be avoided for this type of medication." "After taking a puff, hold the breath for 30 seconds before exhaling."

"Breathe out slowly before positioning the inhaler." It is important to breathe out slowly before positioning the inhaler. This allows the patient to take a deeper breath while inhaling the medication, thus enhancing the effectiveness of the dose. The inhaler should be shaken well. A spacer may be used. Holding the breath after the inhalation of medication helps keep the medication in the lungs, but 30 seconds will not be possible for a patient with COPD.

The physician has prescribed salmeterol (Serevent) for a patient with asthma. In reviewing the use of dry powder inhalers (DPIs) with the patient, what instructions should the nurse provide? "Close lips tightly around the mouthpiece and breathe in deeply and quickly." "To administer a DPI, you must use a spacer that holds the medicine so that you can inhale it." "You will know you have correctly used the DPI when you taste or sense the medicine going into your lungs." "Hold the inhaler several inches in front of your mouth and breathe in slowly, holding the medicine as long as possible."

"Close lips tightly around the mouthpiece and breathe in deeply and quickly." The patient should be instructed to tightly close the lips around the mouthpiece and breathe in deeply and quickly to ensure the medicine moves down deeply into the lungs. Dry powder inhalers do not require spacer devices. The patient may not taste or sense the medicine going into the lungs.

A 55-year-old patient with increasing dyspnea is being evaluated for a possible diagnosis of chronic obstructive pulmonary disease (COPD). When teaching a patient about pulmonary function testing (PFT) for this condition, what is the most important question the nurse should ask? "Have you taken any bronchodilators in the past 6 hours?" "Are you claustrophobic?" "Are you allergic to shellfish?" "Do you have any metal implants or prostheses?"

"Have you taken any bronchodilators in the past 6 hours?"

The nurse evaluates that discharge teaching for a patient hospitalized with pneumonia has been effective when the patient makes which statement about measures to prevent a relapse?

"I must have a follow-up chest x-ray in 6 to 8 weeks to evaluate the pneumonia's resolution." RATIONALE: The follow-up chest x-ray will be done in 6 to 8 weeks to evaluate pneumonia resolution. A patient should seek medical treatment for upper respiratory infections that persist for more than 7 days. It may be important for the patient to continue with coughing and deep breathing exercises for 6 to 8 weeks, not 12 weeks, until all of the infection has cleared from the lungs. Increased fluid intake, not caloric intake, is required to liquefy secretions.

The patient with HIV has been diagnosed with Candida albicans, an opportunistic infection. The nurse knows the patient needs more teaching when she says,

"I need to be isolated from my family and friends so they won't get it." RATIONALE: The patient with an opportunistic fungal infection does not need to be isolated because it is not transmitted from person to person. This immunocompromised patient will be likely to have a serious infection so it will be treated with IV amphotericin B. The effectiveness of the therapy can be monitored with fungal serology titers.

The nurse determines that the patient understood medication instructions about the use of a spacer device when taking inhaled medications after hearing the patient state what as the primary benefit? "I will pay less for medication because it will last longer." "More of the medication will get down into my lungs to help my breathing." "Now I will not need to breathe in as deeply when taking the inhaler medications." "This device will make it so much easier and faster to take my inhaled medications."

"More of the medication will get down into my lungs to help my breathing." A spacer assists more medication to reach the lungs, with less being deposited in the mouth and the back of the throat. It does not affect the cost or increase the speed of using the inhaler.

A patient has been receiving oxygen per nasal cannula while hospitalized for COPD. The patient asks the nurse whether oxygen use will be needed at home. What is the most appropriate response by the nurse? "Long-term home oxygen therapy should be used to prevent respiratory failure." "Oxygen will not be needed until or unless you are in the terminal stages of this disease." "Long-term home oxygen therapy should be used to prevent heart problems related to COPD." "You will not need oxygen until your oxygen saturation drops to 88% and you have symptoms of hypoxia."

"You will not need oxygen until your oxygen saturation drops to 88% and you have symptoms of hypoxia." Long-term oxygen therapy in the home will not be considered until the oxygen saturation is less than or equal to 88% and the patient has signs of tissue hypoxia, such as cor pulmonale, erythrocytosis, or impaired mental status. PaO2 less than 55 mm Hg will also allow home oxygen therapy to be considered.

A client uses a metered-dose inhaler (MDI) to aid in management of his asthma. Which action by the client indicates to the nurse that he needs further instruction regarding its use? Select all that apply. 1. Activation of the MDI is not coordinated with inspiration. 2. The client inspires rapidly when using the MDI. 3. The client holds his breath for 3 seconds after inhaling with the MDI. 4. The client shakes the MDI after use. 5. The client performs puffs in rapid succession.

1, 2, 3, 4, 5. Utilization of an MDI requires coordination between activation and inspiration; deep breaths to ensure that medication is distributed into the lungs, holding the breath for 10 seconds or as long as possible to disperse the medication into the lungs, shaking up the medication in the MDI before use, and a sufficient amount of time between puffs to provide an adequate amount of inhalation medication.

Which of the following is significant data to gather from a client who has been diagnosed with pneumonia? Select all that apply. 1. Quality of breath sounds. 2. Presence of bowel sounds. 3. Occurence of chest pain. 4. Amount of peripheral edema. 5. Color of nail beds.

1, 3, 5. A respiratory assessment, which includes auscultating breath sounds and assessing the color of the nail beds, is a priority for clients with pneumonia. Assessing for the presence of chest pain is also an important respiratory assessment as chest pain can interfere with the client's ability to breathe deeply. Auscultating bowel sounds and assessing for peripheral edema may be appropriate assessments, but these are not priority assessments for the client with pneumonia.

The nurse administers two 325 mg aspirin every 4 hours to a client with pneumonia. The nurse should evaluate the outcome of administering the drug by assessing which of the following? Select all that apply. 1. Decreased pain when breathing. 2. Prolonged clotting time. 3. Decreased temperature. 4. Decreased respiratory rate. 5. Increased ability to expectorate secretions.

1, 3. Aspirin is administered to clients with pneumonia because it is an analgesic that helps control chest discomfort and an antipyretic that helps reduce fever. Aspirin has an anticoagulant effect, but that is not the reason for prescribing it for a client with pneumonia, and the use of the drug will be short term. Aspirin does not affect the respiratory rate, and does not facilitate expectoration of secretions.

The nurse is teaching the client how to use a metered-dose inhaler (MDI) to administer a corticosteroid. Which of the following client actions indicates that he is using the MDI correctly? Select all that apply. 1. The inhaler is held upright. 2. The head is tilted down while inhaling the medicine. 3. The client waits 5 minutes between puffs. 4. The mouth is rinsed with water following administration. 5. The client lies supine for 15 minutes following administration.

1, 4. The client should shake the inhaler and hold it upright when administering the drug. The head should be tilted back slightly. The client should wait about 1 to 2 minutes between puffs. The mouth should be rinsed following the use of a corticosteroid MDI to decrease the likelihood of developing an oral infection. The client does not need to lie supine; instead, the client will likely to be able to breathe more freely if sitting upright.

Which of the following findings would most likely indicate the presence of a respiratory infection in a client with asthma? 1. Cough productive of yellow sputum. 2. Bilateral expiratory wheezing. 3. Chest tightness. 4. Respiratory rate of 30 breaths/ minute.

1. A cough productive of yellow sputum is the most likely indicator of a respiratory infection. The other signs and symptoms- wheezing, chest tightness, and increased respiratory rate- are all findings associated with an asthma attack and do not necessarily mean an infection is present.

A client is prescribed metaproterenol (Alupent) via a metered-dose inhaler, two puffs every 4 hours. The nurse instructs the client to report adverse effects. Which of the following are potential adverse effects of metaproterenol? 1. Irregular heartbeat. 2. Constipation. 3. Pedal edema. 4. Decreased pulse rate.

1. Irregular heartbeats should be reported promptly to the care provider. Metaproterenol (Alupent) may cause irregular heartbeat, tachycardia, or anginal pain because of its adrenergic effect on beta-adrenergic receptors in the heart. It is not recommended for use in clients with known cardiac disorders. Metaproterenol does not cause constipation, pedal edema, or bradycardia.

When developing a discharge plan to manage the care of a client with chronic obstructive pulmonary disease (COPD), the nurse should advise the the client to expect to: 1. Develop respiratory infections easily. 2. Maintain current status. 3. Require less supplemental oxygen. 4. Show permanent improvement.

1. A client with COPD is at high risk for development of respiratory infections. COPD is slowly progressive; therefore, maintaining current status and establishing a goal that the client will require less supplemental oxygen are unrealistic expectations. Treatment may slow progression of the disease, but permanent improvement is highly unlikely.

Which of the following is a priority goal for the client with chronic obstructive pulmonary disease (COPD)? 1. Maintaining functional ability. 2. Minimizing chest pain. 3. Increasing carbon dioxide levels in the blood. 4. Treating infectious agents.

1. A priority goal for the client with COPD is to manage the signs and symptoms of the disease process so as to maintain the client's functional ability. Chest pain is not a typical symptom of COPD. The carbon dioxide concentration in the blood is increased to an abnormal level in clients with COPD; it would not be a goal to increase the level further. Preventing infection would be a goal of care for the client with COPD.

The client with pneumonia develops mild constipation, and the nurse administers docusate sodium (Colace) as ordered. This drug works by: 1. Softening the stool. 2. Lubricating the stool. 3. Increasing stool bulk. 4. Stimulating peristalsis.

1. Docusate sodium (Colace) is a stool softener that allows fluid and fatty substances to enter the stool and soften it. Docusate sodium does not lubricate the stool, increase stool bulk, or stimulate peristalsis.

A client with deep vein thrombosis suddenly develops dyspnea, tachypnea, and chest discomfort. What should the nurse do first? 1. Elevate the head of the bed 30 to 45 degrees. 2. Encourage the client to cough and deep breathe. 3. Auscultate the lungs to detect abnormal breath sounds. 4. Contact the physician.

1. Elevating the head of the bed facilitates breathing because the lungs are able to expand as the diaphragm descends. Coughing and deep breathing do not alleviate the symptoms of a pulmonary embolus, nor does lung auscultation. The physician must be kept informed of changes in a client's status, but the priority in this case is alleviating the symptoms.

Which of the following physical assessment findings are normal for a client with advanced chronic obstructive pulmonary disease (COPD)? 1. Increased anteroposterior chest diameter. 2. Underdeveloped neck muscles. 3. Collapsed neck veins. 4. Increased chest excursions with respiration.

1. Increased anteroposterior chest diameter is characteristic of advanced COPD. Air is trapped in the overextended alveoli, and the ribs are fixed in an inspiratory position. The result is the typical barrel-chested appearance. Overly developed, not underdeveloped, neck muscles are associated with COPD because of their increased use in the work of breathing. Distended, not collapsed, neck veins are associated with COPD as a symptom of the heart failure that the client may experience secondary to the increased workload on the heart to pump blood into the pulmonary vasculature. Diminished, not increased, chest excursion is associated with COPD.

Which of the following health promotion activities should the nurse include in the discharge teaching plan for a client with asthma? 1. Incorporate physical exercise as tolerated into the daily routine. 2. Monitor peak flow numbers after meals and at bedtime. 3. Eliminate stressors in the work and home environment. 4. Use sedatives to ensure uninterrupted sleep at night.

1. Physical exercise is beneficial and should be incorporated as tolerated into the client's schedule. Peak flow numbers should be monitored daily, usually in the morning (before taking medication). Peak flow does not need to be monitored after each meal. Stressors in the client's life should be modified but cannot be totally eliminated. Although adequate sleep is important, it is not recommended that sedatives be routinely taken to induce sleep.

A client experiencing a severe asthma attack has the following arterial blood gas: pH 7.33; Pco2 48; Po2 58; HCO3 26. Which of the following orders should the nurse perform first? 1. Albuterol (Proventil) nebulizer. 2. Chest x-ray. 3. Ipratropium (Atrovent) inhaler. 4. Sputum culture.

1. The arterial blood gas reveals a respiratory acidosis with hypoxia. A quick-acting bronchodilator, albuterol, should be administered via nebulizer to improve gas exchange. Ipratropium is a maintenance treatment for bronchospasm that can be used with albuterol. A chest x-ray and sputum sample can be obtained once the client is stable.

A 79-year-old female client is admitted to the hospital with a diagnosis of bacterial pneumonia. While obtaining the client's health history, the nurse learns that the client has osteoarthritis, follows a vegetarian diet, and is very concerned with cleanliness. Which of the following would most likely be a predisposing factor for the diagnosis of pneumonia? 1. Age. 2. Osteoarthritis. 3. Vegetarian diet. 4. Daily bathing.

1. The client's age is a predisposing factor for pneumonia; pneumonia is more common in elderly or debilitated clients. Other predisposing factors include smoking, upper respiratory tract infections, malnutrition, immunosuppression, and the presence of a chronic illness. Osteoarthritis, a nutritionally sound vegetarian diet, and frequent bathing are not predisposing factors for pneumonia.

The nurse is planning to teach a client with chronic obstructive pulmonary disease how to cough effectively. Which of the following instructions should be included? 1. Take a deep abdominal breath, bend forward, and cough three or four times on exhalation. 2. Lie flat on the back, splint the thorax, take two deep breaths, and cough. 3. Take several rapid, shallow breaths and then cough forcefully. 4. Assume a side-lying position, extend the arm over the head, and alternate deep breathing with coughing.

1. The goal of effective coughing is to conserve energy, facilitate removal of secretions, and minimize airway collapse. The client should assume a sitting position with feet on the floor if possible. The client should bend forward slightly and, using pursed-lip breathing, exhale. After resuming an upright position, the client should use abdominal breathing to slowly and deeply inhale. After repeating this process three or four times, the client should take a deep abdominal breath, bend forward, and cough three or four times upon exhalation (" huff" cough). Lying flat does not enhance lung expansion; sitting upright promotes full expansion of the thorax. Shallow breathing does not facilitate removal of secretions, and forceful coughing promotes collapse of airways. A side-lying position does not allow for adequate chest expansion to promote deep breathing.

A client with pneumonia is experiencing pleuritic chest pain. The nurse should assess the client for: 1. A mild but constant aching in the chest. 2. Severe midsternal pain. 3. Moderate pain that worsens on inspiration. 4. Muscle spasm pain that accompanies coughing.

3. Chest pain in pneumonia is generally caused by friction between the pleural layers. It is more severe on inspiration than on expiration, secondary to chest wall movement. Pleuritic chest pain is usually described as sharp, not mild or aching. Pleuritic chest pain is not localized to the sternum, and it is not the result of a muscle spasm.

A nurse is caring for a hospitalized infant with bronchiolitis. Diagnostic tests have confirmed respiratory syncytial virus (RSV). On the basis of this finding, which of the following would be the appropriate nursing action?

1. Initiate strict enteric precautions. 2. Wear a mask when caring for the child. 3. Plan to move the infant to a room with another child with RSV. 4. Leave the infant in the present room, because RSV is not contagious. Rationale: RSV is a highly communicable disorder, but it is not transmitted via the airborne route. It is usually transferred by the hands, and meticulous handwashing is necessary to decrease the spread of organisms. The infant with RSV is isolated in a single room or placed in a room with another child with RSV. Enteric precautions are not necessary; however, the nurse should wear a gown when the soiling of clothing may occur.

A nurse is preparing for the admission of an infant with a diagnosis of bronchiolitis caused by the respiratory syncytial virus (RSV). Choose the interventions that would be included in the plan of care. Select all that apply.

1. Place the infant in a private room. 2. Place the infant in a room near the nurses' station. 3. Ensure that the infant's head is in a flexed position. 4. Wear a mask at all times when in contact with the infant. 5. Place the child in a tent that delivers warm, humidified air. 6. Position the infant side-lying, with the head lower than the chest. Rationale: The infant with RSV should be isolated in a private room or in a room with another child with RSV. The infant should be placed in a room near the nurses' station for close observation. The infant should be positioned with the head and chest at a 30- to 40-degree angle and the neck slightly extended to maintain an open airway and to decrease pressure on the diaphragm. Cool, humidified oxygen is delivered to relieve dyspnea, hypoxemia, and insensible water loss from tachypnea. Contact precautions (wearing gloves and a gown) reduce the nosocomial transmission of RSV.

A client with chronic obstructive pulmonary disease (COPD) is experiencing dyspnea and has a low PaO2 level. The nurse plans to administer oxygen as ordered. Which of the following statements is true concerning oxygen administration to a client with COPD? 1. High oxygen concentrations will cause coughing and dyspnea. 2. High oxygen concentrations may inhibit the hypoxic stimulus to breathe. 3. Increased oxygen use will cause the client to become dependent on the oxygen. 4. Administration of oxygen is contraindicated in clients who are using bronchodilators.

2. Clients who have a long history of COPD may retain carbon dioxide (CO2). Gradually the body adjusts to the higher CO2 concentration, and the high levels of CO2 no longer stimulate the respiratory center. The major respiratory stimulant then becomes hypoxemia. Administration of high concentrations of oxygen eliminates this respiratory stimulus and leads to hypoventilation. Oxygen can be drying if it is not humidified, but it does not cause coughing and dyspnea. Increased oxygen use will not create an oxygen dependency; clients should receive oxygen as needed. Oxygen is not contraindicated with the use of bronchodilators.

The nurse is instructing a client with COPD how to do pursed-lip breathing. In which order should the nurse explain the steps to the client? 1. "Breathe in normally through your nose for 2 counts (while counting to yourself, one, two)." 2. "Relax your neck and shoulder muscles." 3. "Pucker your lips as if you were going to whistle." 4. "Breathe out slowly through pursed lips for 4 counts (while counting to yourself, one, two, three, four)."

2, 1, 3, 4. The nurse should instruct the client to first relax the neck and shoulders and then take several normal breaths. After taking a breath in, the client should pucker the lips, and finally breathe out through pursed lips.

Which of the following is an expected outcome for an elderly client following treatment for bacterial pneumonia? 1. A respiratory rate of 25 to 30 breaths/ minute. 2. The ability to perform activities of daily living without dyspnea. 3. A maximum loss of 5 to 10 lb of body weight. 4. Chest pain that is minimized by splinting the rib cage.

2. An expected outcome for a client recovering from pneumonia would be the ability to perform activities of daily living without experiencing dyspnea. A respiratory rate of 25 to 30 breaths/ minute indicates the client is experiencing tachypnea, which would not be expected on recovery. A weight loss of 5 to 10 lb is undesirable; the expected outcome would be to maintain normal weight. A client who is recovering from pneumonia should experience decreased or no chest pain.

When performing postural drainage, which of the following factors promotes the movement of secretions from the lower to the upper respiratory tract? 1. Friction between the cilia. 2. Force of gravity. 3. Sweeping motion of cilia. 4. Involuntary muscle contractions.

2. The principle behind using postural drainage is that gravity will help move secretions from smaller to larger airways. Postural drainage is best used after percussion has loosened secretions. Coughing or suctioning is then used to remove secretions. Movement of cilia is not sufficient to move secretions. Muscle contractions do not move secretions within the lungs.

A client with bacterial pneumonia is to be started on I.V. antibiotics. Which of the following diagnostic tests must be completed before antibiotic therapy begins? 1. Urinalysis. 2. Sputum culture. 3. Chest radiograph. 4. Red blood cell count.

2. A sputum specimen is obtained for culture to determine the causative organism. After the organism is identified, an appropriate antibiotic can be prescribed. Beginning antibiotic therapy before obtaining the sputum specimen may alter the results of the test. Neither a urinalysis, a chest radiograph, nor a red blood cell count needs to be obtained before initiation of antibiotic therapy for pneumonia.

The nurse should teach the client with asthma that which of the following is one of the most common precipitating factors of an acute asthma attack? 1. Occupational exposure to toxins. 2. Viral respiratory infections. 3. Exposure to cigarette smoke. 4. Exercising in cold temperatures.

2. The most common precipitator of asthma attacks is viral respiratory infection. Clients with asthma should avoid people who have the flu or a cold and should get yearly flu vaccinations. Environmental exposure to toxins or heavy particulate matter can trigger asthma attacks; however, far fewer asthmatics are exposed to such toxins than are exposed to viruses. Cigarette smoke can also trigger asthma attacks, but to a lesser extent than viral respiratory infections. Some asthmatic attacks are triggered by exercising in cold weather.

When caring for the client who is receiving an aminoglycoside antibiotic, the nurse should monitor which of the following laboratory values? 1. Serum sodium. 2. Serum potassium. 3. Serum creatinine. 4. Serum calcium.

3. It is essential to monitor serum creatinine in the client receiving an aminoglycoside antibiotic because of the potential of this type of drug to cause acute tubular necrosis. Aminoglycoside antibiotics do not affect serum sodium, potassium, or calcium levels.

The nurse teaches a client with chronic obstructive pulmonary disease (COPD) to assess for signs and symptoms of right-sided heart failure. Which of the following signs and symptoms should be included in the teaching plan? 1. Clubbing of nail beds. 2. Hypertension. 3. Peripheral edema. 4. Increased appetite.

3. Right-sided heart failure is a complication of COPD that occurs because of pulmonary hypertension. Signs and symptoms of right-sided heart failure include peripheral edema, jugular venous distention, hepatomegaly, and weight gain due to increased fluid volume. Clubbing of nail beds is associated with conditions of chronic hypoxemia. Hypertension is associated with left-sided heart failure. Clients with heart failure have decreased appetites.

A nurse in the newborn nursery is monitoring a preterm newborn infant for respiratory distress syndrome. Which assessment signs if noted in the newborn infant would alert the nurse to the possibility of this syndrome? 1. Hypotension and Bradycardia 2. Tachypnea and retractions 3. Acrocyanosis and grunting 4. The presence of a barrel chest with grunting

2. The infant with respiratory distress syndrome may present with signs of cyanosis, tachypnea or apnea, nasal flaring, chest wall retractions, or audible grunts.

The nurse in the emergency department receives arterial blood gas results for four recently admitted patients with obstructive pulmonary disease. Which patient will require the most rapid action by the nurse? 65-year-old with ABG results: pH 7.31, PaCO2 58 mm Hg, and PaO2 64 mm Hg 22-year-old with ABG results: pH 7.28, PaCO2 60 mm Hg, and PaO2 58 mm Hg 34-year-old with ABG results: pH 7.48, PaCO2 30 mm Hg, and PaO2 65 mm Hg 45-year-old with ABG results: pH 7.34, PaCO2 33 mm Hg, and PaO2 80 mm Hg

22-year-old with ABG results: pH 7.28, PaCO2 60 mm Hg, and PaO2 58 mm Hg

Which of the following mental status changes may occur when a client with pneumonia is first experiencing hypoxia? 1. Coma. 2. Apathy. 3. Irritability. 4. Depression.

3. Clients who are experiencing hypoxia characteristically exhibit irritability, restlessness, or anxiety as initial mental status changes. As the hypoxia becomes more pronounced, the client may become confused and combative. Coma is a late clinical manifestation of hypoxia. Apathy and depression are not symptoms of hypoxia.

A client with pneumonia has a temperature of 102.6 ° F (39.2 ° C), is diaphoretic, and has a productive cough. The nurse should include which of the following measures in the plan of care? 1. Position changes every 4 hours. 2. Nasotracheal suctioning to clear secretions. 3. Frequent linen changes 4. Frequent offering of a bedpan.

3. Frequent linen changes are appropriate for this client because of the diaphoresis. Diaphoresis produces general discomfort. The client should be kept dry to promote comfort. Position changes need to be done every 2 hours. Nasotracheal suctioning is not indicated with the client's productive cough. Frequent offering of a bedpan is not indicated by the data provided in this scenario.

A client who has been taking flunisolide (AeroBid), two inhalations a day, for treatment of asthma.has painful, white patches in his mouth. Which response by the nurse would be most appropriate? 1. "This is an anticipated adverse effect of your medication. It should go away in a couple of weeks." 2. "You are using your inhaler too much and it has irritated your mouth." 3. "You have developed a fungal infection from your medication. It will need to be treated with an antifungal agent." 4. "Be sure to brush your teeth and floss daily. Good oral hygiene will treat this problem."

3. Use of oral inhalant corticosteroids such as flunisolide (AeroBid) can lead to the development of oral thrush, a fungal infection. Once developed, thrush must be treated by antifungal therapy; it will not resolve on its own. Fungal infections can develop even without overuse of the corticosteroid inhaler. Although good oral hygiene can help prevent development of a fungal infection, it cannot be used alone to treat the problem.

Which of the following diets would be most appropriate for a client with chronic obstructive pulmonary disease (COPD)? 1. Low-fat, low-cholesterol diet. 2. Bland, soft diet. 3. Low-sodium diet. 4. High-calorie, high-protein diet.

4. The client should eat high-calorie, high-protein meals to maintain nutritional status and prevent weight loss that results from the increased work of breathing. The client should be encouraged to eat small, frequent meals. A low-fat, low-cholesterol diet is indicated for clients with coronary artery disease. The client with COPD does not necessarily need to follow a sodium-restricted diet, unless otherwise medically indicated. There is no need for the client to eat bland, soft foods.

A nurse on the newborn nursery floor is caring for a neonate. On assessment the infant is exhibiting signs of cyanosis, tachypnea, nasal flaring, and grunting. Respiratory distress syndrome is diagnosed, and the physician prescribes surfactant replacement therapy. The nurse would prepare to administer this therapy by: 1. Subcutaneous injection 2. Intravenous injection 3. Instillation of the preparation into the lungs through an endotracheal tube 4. Intramuscular injection

3. The aim of therapy in RDS is to support the disease until the disease runs its course with the subsequent development of surfactant. The infant may benefit from surfactant replacement therapy. In surfactant replacement, an exogenous surfactant preparation is instilled into the lungs through an endotracheal tube.

After change-of-shift report, which patient should the nurse assess first? 28-year-old with a history of a lung transplant and a temperature of 101° F (38.3° C) 72-year-old with cor pulmonale who has 4+ bilateral edema in his legs and feet 40-year-old with a pleural effusion who is complaining of severe stabbing chest pain 64-year-old with lung cancer and tracheal deviation after subclavian catheter insertion

64-year-old with lung cancer and tracheal deviation after subclavian catheter insertion

Which of the following is an expected outcome of pursed-lip breathing for clients with emphysema? 1. To promote oxygen intake. 2. To strengthen the diaphragm. 3. To strengthen the intercostal muscles. 4. To promote carbon dioxide elimination.

4. Pursed-lip breathing prolongs exhalation and prevents air trapping in the alveoli, thereby promoting carbon dioxide elimination. By prolonging exhalation and helping the client relax, pursed-lip breathing helps the client learn to control the rate and depth of respiration. Pursed-lip breathing does not promote the intake of oxygen, strengthen the diaphragm, or strengthen intercostal muscles.

The cyanosis that accompanies bacterial pneumonia is primarily caused by which of the following? 1. Decreased cardiac output. 2. Pleural effusion. 3. Inadequate peripheral circulation. 4. Decreased oxygenation of the blood.

4. A client with pneumonia has less lung surface available for the diffusion of gases because of the inflammatory pulmonary response that creates lung exudate and results in reduced oxygenation of the blood. The client becomes cyanotic because blood is not adequately oxygenated in the lungs before it enters the peripheral circulation. Decreased cardiac output may be a comorbid condition in some clients with pneumonia; however, it is not the cause of cyanosis. Pleural effusions are a potential complication of pneumonia but are not the primary cause of decreased oxygenation. Inadequate peripheral circulation is also not the cause of the cyanosis that develops with bacterial pneumonia.

Which of the following is an appropriate expected outcome for an adult client with well-controlled asthma? 1. Chest X-ray demonstrates minimal hyperinflation. 2. Temperature remains lower than 100 ° F (37. 8 ° C). 3. Arterial blood gas analysis demonstrates a decrease in PaO2. 4. Breath sounds are clear.

4. Between attacks, breath sounds should be clear on auscultation with good air flow present throughout lung fields. Chest X-rays should be normal. The client should remain afebrile. Arterial blood gases should be normal.

When instructing clients on how to decrease the risk of chronic obstructive pulmonary disease (COPD), the nurse should emphasize which of the following? 1. Participate regularly in aerobic exercises. 2. Maintain a high-protein diet. 3. Avoid exposure to people with known respiratory infections. 4. Abstain from cigarette smoking.

4. Cigarette smoking is the primary cause of COPD. Other risk factors include exposure to environmental pollutants and chronic asthma. Participating in an aerobic exercise program, although beneficial, will not decrease the risk of COPD. Insufficient protein intake and exposure to people with respiratory infections do not increase the risk of COPD.

The nurse assesses the respiratory status of a client who is experiencing an exacerbation of chronic obstructive pulmonary disease (COPD) secondary to an upper respiratory tract infection. Which of the following findings would be expected? 1. Normal breath sounds. 2. Prolonged inspiration. 3. Normal chest movement. 4. Coarse crackles and rhonchi.

4. Exacerbations of COPD are commonly caused by respiratory infections. Coarse crackles and rhonchi would be auscultated as air moves through airways obstructed with secretions. In COPD, breath sounds are diminished because of an enlarged anteroposterior diameter of the chest. Expiration, not inspiration, becomes prolonged. Chest movement is decreased as lungs become overdistended.

A 34-year-old female with a history of asthma is admitted to the emergency department. The nurse notes that the client is dyspneic, with a respiratory rate of 35 breaths/ minute, nasal flaring, and use of accessory muscles. Auscultation of the lung fields reveals greatly diminished breath sounds. Based on these findings, which action should the nurse take to initiate care of the client? 1. Initiate oxygen therapy and reassess the client in 10 minutes. 2. Draw blood for an arterial blood gas analysis and send the client for a chest X-ray. 3. Encourage the client to relax and breathe slowly through the mouth. 4. Administer bronchodilators.

4. In an acute asthma attack, diminished or absent breath sounds can be an ominous sign indicating lack of air movement in the lungs and impending respiratory failure. The client requires immediate intervention with inhaled bronchodilators, I.V. corticosteroids and, possibly, I.V. theophylline (Theo-Dur). Administering oxygen and reassessing the client 10 minutes later would delay needed medical intervention, as would drawing blood for an arterial blood gas analysis and obtaining a chest X-ray. It would be futile to encourage the client to relax and breathe slowly without providing the necessary pharmacologic intervention.

Which of the following indicates that the client with chronic obstructive pulmonary disease (COPD) who has been discharged to home understands his care plan? 1. The client promises to do pursed-lip breathing at home. 2. The client states actions to reduce pain. 3. The client says that he will use oxygen via a nasal cannula at 5 L/ minute. 4. The client agrees to call the physician if dyspnea on exertion increases.

4. Increasing dyspnea on exertion indicates that the client may be experiencing complications of COPD. Therefore, the nurse should notify the physician. Extracting promises from clients is not an outcome criterion. Pain is not a common symptom of COPD. Clients with COPD use low-flow oxygen supplementation (1 to 2 L/ minute) to avoid suppressing the respiratory drive, which, for these clients, is stimulated by hypoxia. .

The nurse monitors a patient after chest tube placement for a hemopneumothorax. The nurse is most concerned if which assessment finding is observed? Subcutaneous emphysema at the insertion site 400 mL of blood in the collection chamber A large air leak in the water-seal chamber Complaint of pain with each deep inspiration

400 mL of blood in the collection chamber

The nurse is caring for a patient with chronic obstructive pulmonary disorder (COPD) and pneumonia who has an order for arterial blood gases to be drawn. What is the minimum length of time the nurse should plan to hold pressure on the puncture site?

5 minutes RATIONALE: After obtaining blood for an arterial blood gas measurement, the nurse should hold pressure on the puncture site for 5 minutes by the clock to be sure that bleeding has stopped. An artery is an elastic vessel under much higher pressure than veins, and significant blood loss or hematoma formation could occur if the time is insufficient.

A diabetic patient's arterial blood gas (ABG) results are pH 7.28; PaCO2 34 mm Hg; PaO2 85 mm Hg; HCO3- 18 mEq/L. The nurse would expect which finding? a. Intercostal retractions b. Kussmaul respirations c. Low oxygen saturation (SpO2) d. Decreased venous O2 pressure

ANS: B Kussmaul (deep and rapid) respirations are a compensatory mechanism for metabolic acidosis. The low pH and low bicarbonate result indicate metabolic acidosis. Intercostal retractions, a low oxygen saturation rate, and a decrease in venous O2 pressure would not be caused by acidosis.

Which action is appropriate for the nurse to delegate to unlicensed assistive personnel (UAP)? a. Listen to a patient's lung sounds for wheezes or rhonchi. b. Label specimens obtained during percutaneous lung biopsy. c. Instruct a patient about how to use home spirometry testing. d. Measure induration at the site of a patient's intradermal skin test.

ANS: B Labeling of specimens is within the scope of practice of UAP. The other actions require nursing judgment and should be done by licensed nursing personnel.

The community health RN is conducting an educational session w/ community members regarding this symptoms associated w/ TB. Which is one of the first manifestations associated with TB? Dyspnea Chest pain A bloody, productive cough A cough with the expectoration of mucoid sputum

A cough with the expectoration of mucoid sputum

Which test result identifies that a patient with asthma is responding to treatment? An increase in CO2 levels A decreased exhaled nitric oxide A decrease in white blood cell count An increase in serum bicarbonate levels

A decreased exhaled nitric oxide Nitric oxide levels are increased in the breath of people with asthma. A decrease in the exhaled nitric oxide concentration suggests that the treatment may be decreasing the lung inflammation associated with asthma and adherence to treatment. An increase in CO2 levels, decreased white blood cell count, and increased serum bicarbonate levels do not indicate a positive response to treatment in the asthma patient.

The nurse notices clear nasal drainage in a patient newly admitted with facial trauma, including a nasal fracture. The nurse should A) Test the drainage for the presence of glucose. B) Suction the nose to maintain airway clearance. C) Document the findings and continue monitoring. D) Apply a drip pad and reassure the patient this is normal.

A) Clear nasal drainage suggests leakage of cerebrospinal fluid (CSF). The drainage should be tested for the presence of glucose, which would indicate the presence of CSF. Suctioning should not be done. Documenting the findings and monitoring are important after notifying the health care provider. A drip pad may be applied, but the patient should not be reassured that this is normal.

When initially teaching a patient the supraglottic swallow following a radical neck dissection, with which of the following foods or fluids should the nurse begin? A) Cola B) Applesauce C) French fries D) White grape juice

A) When learning the supraglottic swallow, it may be helpful to start with carbonated beverages because the effervescence provides clues about the liquid's position. Thin, watery fluids should be avoided because they are difficult to swallow and increase the risk of aspiration. Nonpourable pureed foods, such as applesauce, would decrease the risk of aspiration, but carbonated beverages are the better choice with which to start.

An experienced nurse instructs a new nurse about how to care for a patient with dyspnea caused by a pulmonary fungal infection. Which action by the new nurse indicates a need for further teaching? a. Listening to the patient's lung sounds several times during the shift b. Placing the patient on droplet precautions and in a private hospital room c. Increasing the oxygen flow rate to keep the oxygen saturation above 90% d. Monitoring patient serology results to identify the specific infecting organism

ANS: B Fungal infections are not transmitted from person to person. Therefore no isolation procedures are necessary. The other actions by the new nurse are appropriate

The RN is preparing a lsit of home care instructions for a client who has been hospitalized and treated for TB. Which instuctions should the RN include on the list? Select all that apply. a) Activities should be resumed gradually. b) Avoid contact with other individuals, except family members, for at least 6 months. c) A sputum culture is needed every 2 to 4 wks once medication therapy is initiated. d) Respiratory isolation is not necessary b/c family members already hace been exposed. e) Cover the mouth and nose when coughing or sneezing and put used tissues in plastic bags f) when one sputum culture is neg, the client is no longer considered infectious and usually can return to former employment

A, C, D, E -follow med regimen exactly - after 2-3 wks of therapy, it is unlikely to infect anyone - resume activities gradually - diet rich in iron, protein, and vit C for healing and preventing recurrence - when results of three sputum cultures are neg, then the pt is no longer considered infectious.

5. An infant with a congenital heart defect is receiving palivizumab (Synagis). The purpose of this is to: A. prevent RSV infection. B. prevent secondary bacterial infection. C. decrease toxicity of antiviral agents. D. make isolation of infant with RSV unnecessary.

A. prevent RSV infection. Synagis is a monoclonal antibody specific for RSV. Monthly administration is initiated to prevent infection with RSV. Given monthly (28-31 days apart)

The nurse notes new onset confusion in an older patient who is normally alert and oriented. In which order should the nurse take the following actions? (Put a comma and a space between each answer choice [A, B, C, D].) a. Obtain the oxygen saturation. b. Check the patient's pulse rate. c. Document the change in status. d. Notify the health care provider

ANS: A, B, D, C Assessment for physiologic causes of new onset confusion such as pneumonia, infection, or perfusion problems should be the first action by the nurse. Airway and oxygenation should be assessed first, then circulation. After assessing the patient, the nurse should notify the health care provider. Finally, documentation of the assessments and care should be done.

The nurse notes new onset confusion in an 89-year-old patient in a long-term care facility. The patient is normally alert and oriented. In which order should the nurse take the following actions? Put a comma and space between each answer choice (a, b, c, d, etc.) ____________________ a. Obtain the oxygen saturation. b. Check the patient's pulse rate. c. Document the change in status. d. Notify the health care provider.

ANS: A, B, D, C Assessment for physiologic causes of new onset confusion such as pneumonia, infection, or perfusion problems should be the first action by the nurse. Airway and oxygenation should be assessed first, then circulation. After assessing the patient, the nurse should notify the health care provider. Finally, documentation of the assessments and care should be done. DIF: Cognitive Level: Analysis REF: 549 | 551 OBJ: Special Questions: Alternate Item Format, Prioritization TOP: Nursing Process: Implementation MSC: NCLEX: Physiolog

A patient with acute dyspnea is scheduled for a spiral computed tomography (CT) scan. Which information obtained by the nurse is a priority to communicate to the health care provider before the CT? a. Allergy to shellfish b. Apical pulse of 104 c. Respiratory rate of 30 d. Oxygen saturation of 90%

ANS: A Because iodine-based contrast media is used during a spiral CT, the patient may need to have the CT scan without contrast or be premedicated before injection of the contrast media. The increased pulse, low oxygen saturation, and tachypnea all indicate a need for further assessment or intervention but do not indicate a need to modify the CT procedure.

The clinic nurse teaches a patient with a 42 pack-year history of cigarette smoking about lung disease. Which information will be most important for the nurse to include? a. Options for smoking cessation b. Reasons for annual sputum cytology testing c. Erlotinib (Tarceva) therapy to prevent tumor risk d. Computed tomography (CT) screening for lung cancer

ANS: A Because smoking is the major cause of lung cancer, the most important role for the nurse is teaching patients about the benefits of and means of smoking cessation. CT scanning is currently being investigated as a screening test for high-risk patients. However, if there is a positive finding, the person already has lung cancer. Erlotinib may be used in patients who have lung cancer, but it is not used to reduce the risk of developing cancer.

19. After being hit by a baseball, a patient arrives in the emergency department with a possible nasal fracture. Which finding by the nurse is most important to report to the health care provider? a. Clear nasal drainage b. Complaint of nasal pain c. Bilateral nose swelling and bruising d. Inability to breathe through the nose

ANS: A Clear nasal drainage may indicate a meningeal tear with leakage of cerebrospinal fluid. This would place the patient at risk for complications such as meningitis. The other findings are typical with a nasal fracture and do not indicate any complications.

The nurse administers prescribed therapies for a patient with cor pulmonale and right-sided heart failure. Which assessment would best evaluate the effectiveness of the therapies? a. Observe for distended neck veins. b. Auscultate for crackles in the lungs. c. Palpate for heaves or thrills over the heart. d. Review hemoglobin and hematocrit values.

ANS: A Cor pulmonale is right ventricular failure caused by pulmonary hypertension, so clinical manifestations of right ventricular failure such as peripheral edema, jugular venous distention, and right upper-quadrant abdominal tenderness would be expected. Crackles in the lungs are likely to be heard with left-sided heart failure. Findings in cor pulmonale include evidence of right ventricular hypertrophy on electrocardiogram ECG and an increase in intensity of the second heart sound. Heaves or thrills are not common with cor pulmonale. Chronic hypoxemia leads to polycythemia and increased total blood volume and viscosity of the blood. The hemoglobin and hematocrit values are more likely to be elevated with cor pulmonale than decreased.

6. The nurse is caring for a mechanically ventilated patient with a cuffed tracheostomy tube. Which action by the nurse would best determine if the cuff has been properly inflated? a. Use a manometer to ensure cuff pressure is at an appropriate level. b. Check the amount of cuff pressure ordered by the health care provider. c. Suction the patient first with a fenestrated inner cannula to clear secretions. d. Insert the decannulation plug before the nonfenestrated inner cannula is removed.

ANS: A Measurement of cuff pressure using a manometer to ensure that cuff pressure is 20 mm Hg or lower will avoid compression of the tracheal wall and capillaries. Never insert the decannulation plug in a tracheostomy tube until the cuff is deflated and the nonfenestrated inner cannula is removed. Otherwise, the patient's airway is occluded. A health care provider's order is not required to determine safe cuff pressure. A nonfenestrated inner cannula must be used to suction a patient to prevent tracheal damage occurring from the suction catheter passing through the fenestrated openings.

An older patient is receiving standard multidrug therapy for tuberculosis (TB). The nurse should notify the health care provider if the patient exhibits which finding? a. Yellow-tinged skin b. Orange-colored sputum c. Thickening of the fingernails d. Difficulty hearing high-pitched voices

ANS: A Noninfectious hepatitis is a toxic effect of isoniazid (INH), rifampin, and pyrazinamide, and patients who develop hepatotoxicity will need to use other medications. Changes in hearing and nail thickening are not expected with the four medications used for initial TB drug therapy. Presbycusis is an expected finding in the older adult patient. Orange discoloration of body fluids is an expected side effect of rifampin and not an indication to call the health care provider.

18. The nurse is caring for a hospitalized older patient who has nasal packing in place to treat a nosebleed. Which assessment finding will require the most immediate action by the nurse? a. The oxygen saturation is 89%. b. The nose appears red and swollen. c. The patient's temperature is 100.1° F (37.8° C). d. The patient complains of level 8 (0 to 10 scale) pain.

ANS: A Older patients with nasal packing are at risk of aspiration or airway obstruction. An O2 saturation of 89% should alert the nurse to further assess for these complications. The other assessment data also indicate a need for nursing action but not as immediately as the low O2 saturation.

The nurse cares for a patient who has just had a thoracentesis. Which assessment information obtained by the nurse is a priority to communicate to the health care provider? a. Oxygen saturation is 88%. b. Blood pressure is 145/90 mm Hg. c. Respiratory rate is 22 breaths/minute when lying flat. d. Pain level is 5 (on 0 to 10 scale) with a deep breath.

ANS: A Oxygen saturation would be expected to improve after a thoracentesis. A saturation of 88% indicates that a complication such as pneumothorax may be occurring. The other assessment data also indicate a need for ongoing assessment or intervention, but the low oxygen saturation is the priority.

A patient experiences a chest wall contusion as a result of being struck in the chest with a baseball bat. The emergency department nurse would be most concerned if which finding is observed during the initial assessment? a. Paradoxic chest movement b. Complaint of chest wall pain c. Heart rate of 110 beats/minute d. Large bruised area on the chest

ANS: A Paradoxic chest movement indicates that the patient may have flail chest, which can severely compromise gas exchange and can rapidly lead to hypoxemia. Chest wall pain, a slightly elevated pulse rate, and chest bruising all require further assessment or intervention, but the priority concern is poor gas exchange.

8. A nurse obtains a health history from a patient who has a 35 pack-year smoking history. The patient complains of hoarseness and tightness in the throat and difficulty swallowing. Which question is most important for the nurse to ask? a. "How much alcohol do you drink in an average week?" b. "Do you have a family history of head or neck cancer?" c. "Have you had frequent streptococcal throat infections?" d. "Do you use antihistamines for upper airway congestion?"

ANS: A Prolonged alcohol use and smoking are associated with the development of laryngeal cancer, which the patient's symptoms and history suggest. Family history is not a risk factor for head or neck cancer. Frequent antihistamine use would be asked about if the nurse suspected allergic rhinitis, but the patient's symptoms are not suggestive of this diagnosis. Streptococcal throat infections also may cause these clinical manifestations, but patients with this type of infection will also have pain and a fever.

A patient is admitted to the emergency department complaining of sudden onset shortness of breath and is diagnosed with a possible pulmonary embolus. How should the nurse prepare the patient for diagnostic testing to confirm the diagnosis? a. Start an IV so contrast media may be given. b. Ensure that the patient has been NPO for at least 6 hours. c. Inform radiology that radioactive glucose preparation is needed. d. Instruct the patient to undress to the waist and remove any metal objects.

ANS: A Spiral computed tomography (CT) scans are the most commonly used test to diagnose pulmonary emboli, and contrast media may be given IV. A chest x-ray may be ordered but will not be diagnostic for a pulmonary embolus. Preparation for a chest x-ray includes undressing and removing any metal. Bronchoscopy is used to detect changes in the bronchial tree, not to assess for vascular changes, and the patient should be NPO 6 to 12 hours before the procedure. Positron emission tomography (PET) scans are most useful in determining the presence of malignancy, and a radioactive glucose preparation is used.

12. Which action should the nurse take first when a patient develops a nosebleed? a. Pinch the lower portion of the nose for 10 minutes. b. Pack the affected nare tightly with an epistaxis balloon. c. Obtain silver nitrate that will be needed for cauterization. d. Apply ice compresses over the patient's nose and cheeks.

ANS: A The first nursing action for epistaxis is to apply direct pressure by pinching the nostrils. Application of cold packs may decrease blood flow to the area, but will not be sufficient to stop bleeding. Cauterization and nasal packing are medical interventions that may be needed if pressure to the nares does not stop the bleeding, but these are not the first actions to take for a nosebleed.

The nurse assesses a patient with chronic obstructive pulmonary disease (COPD) who has been admitted with increasing dyspnea over the last 3 days. Which finding is most important for the nurse to report to the health care provider? a. Respirations are 36 breaths/minute. b. Anterior-posterior chest ratio is 1:1. c. Lung expansion is decreased bilaterally. d. Hyperresonance to percussion is present.

ANS: A The increase in respiratory rate indicates respiratory distress and a need for rapid interventions such as administration of oxygen or medications. The other findings are common chronic changes occurring in patients with COPD.

When teaching the patient who is receiving standard multidrug therapy for tuberculosis (TB) about possible toxic effects of the antitubercular medications, the nurse will give instructions to notify the health care provider if the patient develops a. yellow-tinged skin. b. changes in hearing. c. orange-colored sputum. d. thickening of the fingernails.

ANS: A Noninfectious hepatitis is a toxic effect of isoniazid (INH), rifampin, and pyrazinamide, and patients who develop hepatotoxicity will need to use other medications. Changes in hearing and nail thickening are not expected with the four medications used for initial TB drug therapy. Orange discoloration of body fluids is an expected side effect of rifampin and not an indication to call the health care provider. DIF: Cognitive Level: Application REF: 555 | 556 TOP: Nursing Process: Implementation MSC: NCLEX: Physiological Integrity

Which information will the nurse include in the patient teaching plan for a patient who is receiving rifampin (Rifadin) for treatment of tuberculosis? a. "Your urine, sweat, and tears will be orange colored." b. "Read a newspaper daily to check for changes in vision." c. "Take vitamin B6 daily to prevent peripheral nerve damage." d. "Call the health care provider if you notice any hearing loss."

ANS: A Orange-colored body secretions are a side effect of rifampin. The other adverse effects are associated with other antituberculosis medications. DIF: Cognitive Level: Application REF: 555 TOP: Nursing Process: Planning MSC: NCLEX: Physiological Integrity

A patient experiences a steering wheel injury as a result of an automobile accident. During the initial assessment, the emergency department nurse would be most concerned about a. paradoxic chest movement. b. the complaint of chest wall pain. c. a heart rate of 110 beats/minute. d. a large bruised area on the chest.

ANS: A Paradoxic chest movement indicates that the patient may have flail chest, which can severely compromise gas exchange and can rapidly lead to hypoxemia. Chest wall pain, a slightly elevated pulse rate, and chest bruising all require further assessment or intervention, but the priority concern is poor gas exchange. DIF: Cognitive Level: Application REF: 567 | 569 TOP: Nursing Process: Assessment MSC: NCLEX: Physiological Integrity

The nurse observes nursing assistive personnel (NAP) doing all the following activities when caring for a patient with right lower lobe pneumonia. The nurse will need to intervene when NAP a. lower the head of the patient's bed to 10 degrees. b. splint the patient's chest during coughing. c. help the patient to ambulate to the bathroom. d. assist the patient to a bedside chair for meals.

ANS: A Positioning the patient with the head of the bed lowered will decrease ventilation. The other actions are appropriate for a patient with pneumonia. DIF: Cognitive Level: Application REF: 552-553 OBJ: Special Questions: Delegation TOP: Nursing Process: Implementation MSC: NCLEX: Safe and Effective Care Environment

After 2 months of tuberculosis (TB) treatment with a standard four-drug regimen, a patient continues to have positive sputum smears for acid-fast bacilli (AFB). Which action should the nurse take next? a. Ask the patient whether medications have been taken as directed. b. Discuss the need to use some different medications to treat the TB. c. Schedule the patient for directly observed therapy three times weekly. d. Educate about using a 2-drug regimen for the last 4 months of treatment.

ANS: A The first action should be to determine whether the patient has been compliant with drug therapy because negative sputum smears would be expected if the TB bacillus is susceptible to the medications and if the medications have been taken correctly. Depending on whether the patient has been compliant or not, different medications or directly observed therapy may be indicated. A two-drug regimen will be used only if the sputum smears are negative for AFB. DIF: Cognitive Level: Application REF: 556-557 TOP: Nursing Process: Implementation MSC: NCLEX: Physiological Integrity

A staff nurse has a tuberculosis (TB) skin test of 16-mm induration. A chest radiograph is negative, and the nurse has no symptoms of TB. The occupational health nurse will plan on teaching the staff nurse about the a. use and side effects of isoniazid (INH). b. standard four-drug therapy for TB. c. need for annual repeat TB skin testing. d. bacille Calmette-Guérin (BCG) vaccine.

ANS: A The nurse is considered to have a latent TB infection and should be treated with INH daily for 6 to 9 months. The four-drug therapy would be appropriate if the nurse had active TB. TB skin testing is not done for individuals who have already had a positive skin test. BCG vaccine is not used in the United States and would not be helpful for this individual, who already has a TB infection. DIF: Cognitive Level: Application REF: 556-557 TOP: Nursing Process: Planning MSC: NCLEX: Health Promotion and Maintenance

A patient with a possible pulmonary embolism complains of chest pain and difficulty breathing. The nurse finds a heart rate of 142, BP reading of 100/60, and respirations of 42. The nurse's first action should be to a. elevate the head of the bed to 45 to 60 degrees. b. administer the ordered pain medication. c. notify the patient's health care provider. d. offer emotional support and reassurance.

ANS: A The patient has symptoms consistent with a pulmonary embolism. Elevating the head of the bed will improve ventilation and gas exchange. The other actions can be accomplished after the head is elevated (and oxygen is started). DIF: Cognitive Level: Application REF: 580 OBJ: Special Questions: Prioritization TOP: Nursing Process: Implementation MSC: NCLEX: Physiological Integrity

An hour after a thoracotomy, a patient complains of incisional pain at a level 7 out of 10 and has decreased left-sided breath sounds. The pleural drainage system has 100 mL of bloody drainage and a large air leak. Which action is best for the nurse to take next? a. Administer the prescribed PRN morphine. b. Assist the patient to deep breathe and cough. c. Milk the chest tube gently to remove any clots. d. Tape the area around the insertion site of the chest tube.

ANS: A The patient is unlikely to take deep breaths or cough until the pain level is lower. A chest tube output of 100 mL is not unusual in the first hour after thoracotomy and would not require milking of the chest tube. An air leak is expected in the initial postoperative period after thoracotomy. DIF: Cognitive Level: Application REF: 573-574 TOP: Nursing Process: Implementation MSC: NCLEX: Physiological Integrity

A patient has just been admitted with probable bacterial pneumonia and sepsis. Which order should the nurse implement first? a. Chest x-ray via stretcher b. Blood cultures from two sites c. Ciprofloxacin (Cipro) 400 mg IV d. Acetaminophen (Tylenol) rectal suppository

ANS: B Initiating antibiotic therapy rapidly is essential, but it is important that the cultures be obtained before antibiotic administration. The chest x-ray and acetaminophen administration can be done last.

A patient who was admitted the previous day with pneumonia complains of a sharp pain "whenever I take a deep breath." Which action will the nurse take next? a. Listen to the patient's lungs. b. Administer the PRN morphine. c. Have the patient cough forcefully. d. Notify the patient's health care provider.

ANS: A The patient's statement indicates that pleurisy or a pleural effusion may have developed and the nurse will need to listen for a pleural friction rub and/or decreased breath sounds. Assessment should occur before administration of pain medications. The patient is unlikely to be able to cough forcefully until pain medication has been administered. The nurse will want to obtain more assessment data before calling the health care provider. DIF: Cognitive Level: Application REF: 576 OBJ: Special Questions: Prioritization TOP: Nursing Process: Assessment MSC: NCLEX: Physiological Integrity

Following assessment of a patient with pneumonia, the nurse identifies a nursing diagnosis of ineffective airway clearance. Which information best supports this diagnosis? a. Weak, nonproductive cough effort b. Large amounts of greenish sputum c. Respiratory rate of 28 breaths/minute d. Resting pulse oximetry (SpO2) of 85%

ANS: A The weak, nonproductive cough indicates that the patient is unable to clear the airway effectively. The other data would be used to support diagnoses such as impaired gas exchange and ineffective breathing pattern. DIF: Cognitive Level: Application REF: 551-552 TOP: Nursing Process: Diagnosis MSC: NCLEX: Physiological Integrity

Which factors will the nurse consider when calculating the CURB-65 score for a patient with pneumonia (select all that apply)? a. Age b. Blood pressure c. Respiratory rate d. Oxygen saturation e. Presence of confusion f. Blood urea nitrogen (BUN) level

ANS: A, B, C, E, F Data collected for the CURB-65 are mental status (confusion), BUN (elevated), blood pressure (decreased), respiratory rate (increased), and age (65 and older). The other information is also essential to assess, but are not used for CURB-65 scoring.

2. The nurse is reviewing the medical records for five patients who are scheduled for their yearly physical examinations in September. Which patients should receive the inactivated influenza vaccination (select all that apply)? a. A 76-year-old nursing home resident b. A 36-year-old female patient who is pregnant c. A 42-year-old patient who has a 15 pack-year smoking history d. A 30-year-old patient who takes corticosteroids for rheumatoid arthritis e. A 24-year-old patient who has allergies to penicillin and cephalosporins

ANS: A, B, D Current guidelines suggest that healthy individuals between 6 months and age 49 receive intranasal immunization with live, attenuated influenza vaccine. Individuals who are pregnant, residents of nursing homes, or are immunocompromised or who have chronic medical conditions should receive inactivated vaccine by injection. The corticosteroid use by the 30-year-old increases the risk for infection.

1. The clinic nurse is teaching a patient with acute sinusitis. Which interventions should the nurse plan to include in the teaching session (select all that apply)? a. Decongestants can be used to relieve swelling. b. Blowing the nose should be avoided to decrease the nosebleed risk. c. Taking a hot shower will increase sinus drainage and decrease pain. d. Saline nasal spray can be made at home and used to wash out secretions. e. You will be more comfortable if you keep your head in an upright position.

ANS: A, C, D, E The steam and heat from a shower will help thin secretions and improve drainage. Decongestants can be used to relieve swelling. Patients can use either over-the-counter (OTC) sterile saline solutions or home-prepared saline solutions to thin and remove secretions. Maintaining an upright posture decreases sinus pressure and the resulting pain. Blowing the nose after a hot shower or using the saline spray is recommended to expel secretions.

When caring for a patient who is hospitalized with active tuberculosis (TB), the nurse observes a student nurse who is assigned to take care of a patient. Which action, if performed by the student nurse, would require an intervention by the nurse? a. The patient is offered a tissue from the box at the bedside. b. A surgical face mask is applied before visiting the patient. c. A snack is brought to the patient from the unit refrigerator. d. Hand washing is performed before entering the patient's room.

ANS: B A high-efficiency particulate-absorbing (HEPA) mask, rather than a standard surgical mask, should be used when entering the patient's room because the HEPA mask can filter out 100% of small airborne particles. Hand washing before entering the patient's room is appropriate. Because anorexia and weight loss are frequent problems in patients with TB, bringing food to the patient is appropriate. The student nurse should perform hand washing after handling a tissue that the patient has used, but no precautions are necessary when giving the patient an unused tissue.

16. An infant is hospitalized with RSV bronchiolitis. The priority nursing diagnosis is: a. fatigue related to increased work of breathing. b. ineffective breathing pattern related to airway inflammation and increased secretions. c. risk for fluid volume deficit related to tachypnea and decreased oral intake. d. fear and/or anxiety related to dyspnea and hospitalization.

ANS: B An ineffective breathing pattern is the priority nursing diagnosis for an infant hospitalized with RSV infection.

A patient with idiopathic pulmonary arterial hypertension (IPAH) is receiving nifedipine (Procardia). Which assessment would best indicate to the nurse that the patient's condition is improving? a. Blood pressure (BP) is less than 140/90 mm Hg. b. Patient reports decreased exertional dyspnea. c. Heart rate is between 60 and 100 beats/minute. d. Patient's chest x-ray indicates clear lung fields.

ANS: B Because a major symptom of IPAH is exertional dyspnea, an improvement in this symptom would indicate that the medication was effective. Nifedipine will affect BP and heart rate, but these parameters would not be used to monitor the effectiveness of therapy for a patient with IPAH. The chest x-ray will show clear lung fields even if the therapy is not effective.

After the nurse has received change-of-shift report, which patient should the nurse assess first? a. A patient with pneumonia who has crackles in the right lung base b. A patient with possible lung cancer who has just returned after bronchoscopy c. A patient with hemoptysis and a 16-mm induration with tuberculin skin testing d. A patient with chronic obstructive pulmonary disease (COPD) and pulmonary function testing (PFT) that indicates low forced vital capacity

ANS: B Because the cough and gag are decreased after bronchoscopy, this patient should be assessed for airway patency. The other patients do not have clinical manifestations or procedures that require immediate assessment by the nurse.

The nurse receives change-of-shift report on the following four patients. Which patient should the nurse assess first? a. A 23-year-old patient with cystic fibrosis who has pulmonary function testing scheduled b. A 46-year-old patient on bed rest who is complaining of sudden onset of shortness of breath c. A 77-year-old patient with tuberculosis (TB) who has four antitubercular medications due in 15 minutes d. A 35-year-old patient who was admitted the previous day with pneumonia and has a temperature of 100.2° F (37.8° C)

ANS: B Patients on bed rest who are immobile are at high risk for deep vein thrombosis (DVT). Sudden onset of shortness of breath in a patient with a DVT suggests a pulmonary embolism and requires immediate assessment and action such as oxygen administration. The other patients should also be assessed as soon as possible, but there is no indication that they may need immediate action to prevent clinical deterioration.

The nurse provides discharge instructions to a patient who was hospitalized for pneumonia. Which statement, if made by the patient, indicates a good understanding of the instructions? a. "I will call the doctor if I still feel tired after a week." b. "I will continue to do the deep breathing and coughing exercises at home." c. "I will schedule two appointments for the pneumonia and influenza vaccines." d. "I'll cancel my chest x-ray appointment if I'm feeling better in a couple weeks."

ANS: B Patients should continue to cough and deep breathe after discharge. Fatigue is expected for several weeks. The Pneumovax and influenza vaccines can be given at the same time in different arms. Explain that a follow-up chest x-ray needs to be done in 6 to 8 weeks to evaluate resolution of pneumonia

A patient with a chronic cough has a bronchoscopy. After the procedure, which intervention by the nurse is most appropriate? a. Elevate the head of the bed to 80 to 90 degrees. b. Keep the patient NPO until the gag reflex returns. c. Place on bed rest for at least 4 hours after bronchoscopy. d. Notify the health care provider about blood-tinged mucus.

ANS: B Risk for aspiration and maintaining an open airway is the priority. Because a local anesthetic is used to suppress the gag/cough reflexes during bronchoscopy, the nurse should monitor for the return of these reflexes before allowing the patient to take oral fluids or food. Blood-tinged mucus is not uncommon after bronchoscopy. The patient does not need to be on bed rest, and the head of the bed does not need to be in the high-Fowler's position.

17. Which nursing action could the registered nurse (RN) working in a skilled care hospital unit delegate to an experienced licensed practical/vocational nurse (LPN/LVN) caring for a patient with a permanent tracheostomy? a. Assess the patient's risk for aspiration. b. Suction the tracheostomy when needed. c. Teach the patient about self-care of the tracheostomy. d. Determine the need for replacement of the tracheostomy tube.

ANS: B Suctioning of a stable patient can be delegated to LPNs/LVNs. Patient assessment and patient teaching should be done by the RN.

The nurse teaches a patient about the transmission of pulmonary tuberculosis (TB). Which statement, if made by the patient, indicates that teaching was effective? a. "I will avoid being outdoors whenever possible." b. "My husband will be sleeping in the guest bedroom." c. "I will take the bus instead of driving to visit my friends." d. "I will keep the windows closed at home to contain the germs."

ANS: B Teach the patient how to minimize exposure to close contacts and household members. Homes should be well ventilated, especially the areas where the infected person spends a lot of time. While still infectious, the patient should sleep alone, spend as much time as possible outdoors, and minimize time in congregate settings or on public transportation.

16. The nurse obtains the following assessment data on an older patient who has influenza. Which information will be most important for the nurse to communicate to the health care provider? a. Fever of 100.4° F (38° C) b. Diffuse crackles in the lungs c. Sore throat and frequent cough d. Myalgia and persistent headache

ANS: B The crackles indicate that the patient may be developing pneumonia, a common complication of influenza, which would require aggressive treatment. Myalgia, headache, mild temperature elevation, and sore throat with cough are typical manifestations of influenza and are treated with supportive care measures such as over-the-counter (OTC) pain relievers and increased fluid intake.

A patient is admitted to the emergency department with an open stab wound to the left chest. What is the first action that the nurse should take? a. Position the patient so that the left chest is dependent. b. Tape a nonporous dressing on three sides over the chest wound. c. Cover the sucking chest wound firmly with an occlusive dressing. d. Keep the head of the patient's bed at no more than 30 degrees elevation.

ANS: B The dressing taped on three sides will allow air to escape when intrapleural pressure increases during expiration, but it will prevent air from moving into the pleural space during inspiration. Placing the patient on the left side or covering the chest wound with an occlusive dressing will allow trapped air in the pleural space and cause tension pneumothorax. The head of the bed should be elevated to 30 to 45 degrees to facilitate breathing

14. Following a laryngectomy a patient coughs violently during suctioning and dislodges the tracheostomy tube. Which action should the nurse take first? a. Cover stoma with sterile gauze and ventilate through stoma. b. Attempt to reinsert the tracheostomy tube with the obturator in place. c. Assess the patient's oxygen saturation and notify the health care provider. d. Ventilate the patient with a manual bag and face mask until the health care provider arrives.

ANS: B The first action should be to attempt to reinsert the tracheostomy tube to maintain the patient's airway. Assessing the patient's oxygenation is an important action, but it is not the most appropriate first action in this situation. Covering the stoma with a dressing and manually ventilating the patient may be an appropriate action if the nurse is unable to reinsert the tracheostomy tube. Ventilating with a facemask is not appropriate for a patient with a total laryngectomy because there is a complete separation between the upper airway and the trachea.

A patient is diagnosed with both human immunodeficiency virus (HIV) and active tuberculosis (TB) disease. Which information obtained by the nurse is most important to communicate to the health care provider? a. The Mantoux test had an induration of 7 mm. b. The chest-x-ray showed infiltrates in the lower lobes. c. The patient is being treated with antiretrovirals for HIV infection. d. The patient has a cough that is productive of blood-tinged mucus.

ANS: C Drug interactions can occur between the antiretrovirals used to treat HIV infection and the medications used to treat TB. The other data are expected in a patient with HIV and TB.

After 2 months of tuberculosis (TB) treatment with isoniazid (INH), rifampin (Rifadin), pyrazinamide (PZA), and ethambutol, a patient continues to have positive sputum smears for acid-fast bacilli (AFB). Which action should the nurse take next? a. Teach about treatment for drug-resistant TB treatment. b. Ask the patient whether medications have been taken as directed. c. Schedule the patient for directly observed therapy three times weekly. d. Discuss with the health care provider the need for the patient to use an injectable antibiotic.

ANS: B The first action should be to determine whether the patient has been compliant with drug therapy because negative sputum smears would be expected if the TB bacillus is susceptible to the medications and if the medications have been taken correctly. Assessment is the first step in the nursing process. Depending on whether the patient has been compliant or not, different medications or directly observed therapy may be indicated. The other options are interventions based on assumptions until an assessment has been completed.

The nurse monitors a patient after chest tube placement for a hemopneumothorax. The nurse is most concerned if which assessment finding is observed? a. A large air leak in the water-seal chamber b. 400 mL of blood in the collection chamber c. Complaint of pain with each deep inspiration d. Subcutaneous emphysema at the insertion site

ANS: B The large amount of blood may indicate that the patient is in danger of developing hypovolemic shock. An air leak would be expected immediately after chest tube placement for a pneumothorax. Initially, brisk bubbling of air occurs in this chamber when a pneumothorax is evacuated. The pain should be treated but is not as urgent a concern as the possibility of continued hemorrhage. Subcutaneous emphysema should be monitored but is not unusual in a patient with pneumothorax. A small amount of subcutaneous air is harmless and will be reabsorbed.

A patient with newly diagnosed lung cancer tells the nurse, "I don't think I'm going to live to see my next birthday." Which response by the nurse is best? a. "Would you like to talk to the hospital chaplain about your feelings?" b. "Can you tell me what it is that makes you think you will die so soon?" c. "Are you afraid that the treatment for your cancer will not be effective?" d. "Do you think that taking an antidepressant medication would be helpful?"

ANS: B The nurse's initial response should be to collect more assessment data about the patient's statement. The answer beginning "Can you tell me what it is" is the most open-ended question and will offer the best opportunity for obtaining more data. The answer beginning, "Are you afraid" implies that the patient thinks that the cancer will be immediately fatal, although the patient's statement may not be related to the cancer diagnosis. The remaining two answers offer interventions that may be helpful to the patient, but more assessment is needed to determine whether these interventions are appropriate.

22. When assessing a patient with a sore throat, the nurse notes anterior cervical lymph node swelling, a temperature of 101.6° F (38.7° C), and yellow patches on the tonsils. Which action will the nurse anticipate taking? a. Teach the patient about the use of expectorants. b. Use a swab to obtain a sample for a rapid strep antigen test. c. Discuss the need to rinse the mouth out after using any inhalers. d. Teach the patient to avoid use of nonsteroidal antiinflammatory drugs (NSAIDs).

ANS: B The patient's clinical manifestations are consistent with streptococcal pharyngitis and the nurse will anticipate the need for a rapid strep antigen test and/or cultures. Because patients with streptococcal pharyngitis usually do not have a cough, use of expectorants will not be anticipated. Rinsing the mouth out after inhaler use may prevent fungal oral infections, but the patient's assessment data are not consistent with a fungal infection. NSAIDs are frequently prescribed for pain and fever relief with pharyngitis.

14. The nurse caring for an infant born at 36 weeks of gestation assesses tremors and a weak cry. The nurse is aware that these are symptoms of: a. respiratory distress syndrome. b. hypoglycemia. c. necrotizing enterocolitis. d. renal failure.

ANS: B The preterm infant, before 38 weeks, should be assessed for hypoglycemia because the infant's glycogen stores are not adequate.

The nurse develops a plan of care to prevent aspiration in a high-risk patient. Which nursing action will be most effective? a. Turn and reposition immobile patients at least every 2 hours. b. Place patients with altered consciousness in side-lying positions. c. Monitor for respiratory symptoms in patients who are immunosuppressed. d. Insert nasogastric tube for feedings for patients with swallowing problems.

ANS: B The risk for aspiration is decreased when patients with a decreased level of consciousness are placed in a side-lying or upright position. Frequent turning prevents pooling of secretions in immobilized patients but will not decrease the risk for aspiration in patients at risk. Monitoring of parameters such as breath sounds and oxygen saturation will help detect pneumonia in immunocompromised patients, but it will not decrease the risk for aspiration. Conditions that increase the risk of aspiration include decreased level of consciousness (e.g., seizure, anesthesia, head injury, stroke, alcohol intake), difficulty swallowing, and nasogastric intubation with or without tube feeding. With loss of consciousness, the gag and cough reflexes are depressed, and aspiration is more likely to occur. Other high-risk groups are those who are seriously ill, have poor dentition, or are receiving acid-reducing medications.

A patient with acute shortness of breath is admitted to the hospital. Which action should the nurse take during the initial assessment of the patient? a. Ask the patient to lie down to complete a full physical assessment. b. Briefly ask specific questions about this episode of respiratory distress. c. Complete the admission database to check for allergies before treatment. d. Delay the physical assessment to first complete pulmonary function tests.

ANS: B When a patient has severe respiratory distress, only information pertinent to the current episode is obtained, and a more thorough assessment is deferred until later. Obtaining a comprehensive health history or full physical examination is unnecessary until the acute distress has resolved. Brief questioning and a focused physical assessment should be done rapidly to help determine the cause of the distress and suggest treatment. Checking for allergies is important, but it is not appropriate to complete the entire admission database at this time. The initial respiratory assessment must be completed before any diagnostic tests or interventions can be ordered.

A patient with a pleural effusion is scheduled for a thoracentesis. Which action should the nurse take to prepare the patient for the procedure? a. Start a peripheral IV line to administer the necessary sedative drugs. b. Position the patient sitting upright on the edge of the bed and leaning forward. c. Obtain a large collection device to hold 2 to 3 liters of pleural fluid at one time. d. Remove the water pitcher and remind the patient not to eat or drink anything for 6 hours.

ANS: B When the patient is sitting up, fluid accumulates in the pleural space at the lung bases and can more easily be located and removed. The patient does not usually require sedation for the procedure, and there are no restrictions on oral intake because the patient is not sedated or unconscious. Usually only 1000 to 1200 mL of pleural fluid is removed at one time. Rapid removal of a large volume can result in hypotension, hypoxemia, or pulmonary edema.

When developing a teaching plan for a patient with a 42 pack-year history of cigarette smoking, it will be most important for the nurse to include information about a. computed tomography (CT) screening for lung cancer. b. options for smoking cessation. c. reasons for annual sputum cytology testing. d. erlotinib (Tarceva) therapy to prevent tumor risk.

ANS: B Because smoking is the major cause of lung cancer, the most important role for the nurse is educating patients about the benefits of and means of smoking cessation. Early screening of at-risk patients using sputum cytology, chest x-ray, or CT scanning has not been effective in reducing mortality. Erlotinib may be used in patients who have lung cancer but not to reduce risk for developing tumors. DIF: Cognitive Level: Application REF: 563 | 565 TOP: Nursing Process: Planning MSC: NCLEX: Health Promotion and Maintenance

During assessment of the chest in a patient with pneumococcal pneumonia, the nurse would expect to find a. vesicular breath sounds. b. increased tactile fremitus. c. dry, nonproductive cough. d. hyperresonance to percussion.

ANS: B Increased tactile fremitus over the area of pulmonary consolidation is expected with bacterial pneumonias. Dullness to percussion would be expected. Pneumococcal pneumonia typically presents with a loose, productive cough. Adventitious breath sounds such as crackles and wheezes are typical. DIF: Cognitive Level: Application REF: 549 TOP: Nursing Process: Assessment MSC: NCLEX: Physiological Integrity

Which assessment information obtained by the nurse when caring for a patient who has just had a thoracentesis is most important to communicate to the health care provider? a. BP is 150/90 mm Hg. b. Oxygen saturation is 89%. c. Pain level is 5/10 with a deep breath. d. Respiratory rate is 24 when lying flat.

ANS: B Oxygen saturation would be expected to improve after a thoracentesis. A saturation of 89% indicates that a complication such as pneumothorax may be occurring. The other assessment data also indicate a need for ongoing assessment or intervention, but the low oxygen saturation is the priority. DIF: Cognitive Level: Application REF: 576 OBJ: Special Questions: Prioritization TOP: Nursing Process: Assessment MSC: NCLEX: Physiological Integrity

The nurse is performing tuberculosis (TB) screening in a clinic that has many patients who have immigrated to the United States. Before doing a TB skin test on a patient, which question is most important for the nurse to ask? a. "Is there any family history of TB?" b. "Have you received the bacille Calmette-Guérin (BCG) vaccine for TB?" c. "How long have you lived in the United States?" d. "Do you take any over-the-counter (OTC) medications?"

ANS: B Patients who have received the BCG vaccine will have a positive Mantoux test. Another method for screening (such as a chest x-ray) will need to be used in determining whether the patient has a TB infection. The other information also may be valuable but is not as pertinent to the decision about doing TB skin testing. DIF: Cognitive Level: Application REF: 557 OBJ: Special Questions: Prioritization TOP: Nursing Process: Assessment MSC: NCLEX: Physiological Integrity

A patient with primary pulmonary hypertension (PPH) is receiving nifedipine (Procardia). The nurse will evaluate that the treatment is effective if a. the BP is less than 140/90 mm Hg. b. the patient reports decreased exertional dyspnea. c. the heart rate is between 60 and 100 beats/minute. d. the patient's chest x-ray indicates clear lung fields.

ANS: B Since a major symptom of PPH is exertional dyspnea, an improvement in this symptom would indicate that the medication was effective. Nifedipine will affect BP and heart rate, but these parameters would not be used to monitor effectiveness of therapy for a patient with PPH. The chest x-ray will show clear lung fields even if the therapy is not effective. DIF: Cognitive Level: Application REF: 582 TOP: Nursing Process: Evaluation MSC: NCLEX: Physiological Integrity

The health care provider inserts a chest tube in a patient with a hemopneumothorax. When monitoring the patient after the chest tube placement, the nurse will be most concerned about a. a large air leak in the water-seal chamber. b. 400 mL of blood in the collection chamber. c. complaint of pain with each deep inspiration. d. subcutaneous emphysema at the insertion site.

ANS: B The large amount of blood may indicate that the patient is in danger of developing hypovolemic shock. A large air leak would be expected immediately after chest tube placement for pneumothorax. The pain should be treated but is not as urgent a concern as the possibility of continued hemorrhage. Subcutaneous emphysema should be monitored but is not unusual in a patient with pneumothorax. DIF: Cognitive Level: Application REF: 572 TOP: Nursing Process: Assessment MSC: NCLEX: Physiological Integrity

A patient with newly diagnosed lung cancer tells the nurse, "I think I am going to die pretty soon." Which response by the nurse is best? a. "Would you like to talk to the hospital chaplain about your feelings?" b. "Can you tell me what it is that makes you think you will die so soon?" c. "Are you afraid that the treatment for your cancer will not be effective?" d. "Do you think that taking an antidepressant medication would be helpful?"

ANS: B The nurse's initial response should be to collect more assessment data about the patient's statement. The answer beginning "Can you tell me what it is" is the most open-ended question and will offer the best opportunity for obtaining more data. The answer beginning, "Are you afraid" implies that the patient thinks that the cancer will be immediately fatal, although the patient's statement may not be related to the cancer diagnosis. The remaining two answers offer interventions that may be helpful to the patient, but more assessment is needed to determine whether these interventions are appropriate. DIF: Cognitive Level: Application REF: 565 TOP: Nursing Process: Implementation MSC: NCLEX: Psychosocial Integrity

Which nursing action will be most effective in preventing aspiration pneumonia in patients who are at risk? a. Turn and reposition immobile patients at least every 2 hours. b. Place patients with altered consciousness in side-lying positions. c. Monitor for respiratory symptoms in patients who are immunosuppressed. d. Provide for continuous subglottic aspiration in patients receiving enteral feedings.

ANS: B The risk for aspiration is decreased when patients with a decreased level of consciousness are placed in a side-lying or upright position. Frequent turning prevents pooling of secretions in immobilized patients but will not decrease the risk for aspiration in patients at risk. Monitoring of parameters such as breath sounds and oxygen saturation will help detect pneumonia in immunocompromised patients, but it will not decrease the risk for aspiration. Continuous subglottic suction is recommended for intubated patients but not for all patients receiving enteral feedings. DIF: Cognitive Level: Application REF: 551 TOP: Nursing Process: Implementation MSC: NCLEX: Physiological Integrity

A patient with a pleural effusion is scheduled for a thoracentesis. Before the procedure, the nurse will plan to a. start a peripheral intravenous line to administer the necessary sedative drugs. b. position the patient sitting upright on the edge of the bed and leaning forward. c. remove the water pitcher and remind the patient not to eat or drink anything for 6 hours. d. instruct the patient about the importance of incentive spirometer use after the procedure.

ANS: B When the patient is sitting up, fluid accumulates in the pleural space at the lung bases and can more easily be located and removed. The lung will expand after the effusion is removed; incentive spirometry is not needed to assure alveolar expansion. The patient does not usually require sedation for the procedure, and there are no restrictions on oral intake because the patient is not sedated or unconscious. DIF: Cognitive Level: Application REF: 576 TOP: Nursing Process: Planning MSC: NCLEX: Physiological Integrity

A patient is scheduled for a computed tomography (CT) of the chest with contrast media. Which assessment findings should the nurse immediately report to the health care provider (select all that apply)? a. Patient is claustrophobic. b. Patient is allergic to shellfish. c. Patient recently used a bronchodilator inhaler. d. Patient is not able to remove a wedding band. e. Blood urea nitrogen (BUN) and serum creatinine levels are elevated.

ANS: B, E Because the contrast media is iodine-based and may cause dehydration and decreased renal blood flow, asking about iodine allergies (such as allergy to shellfish) and monitoring renal function before the CT scan are necessary. The other actions are not contraindications for CT of the chest, although they may be for other diagnostic tests, such as magnetic resonance imaging (MRI) or pulmonary function testing (PFT).

5. The nurse auscultating breath sounds of an infant with respiratory syncytial virus would immediately report the assessment of: a. respiration rate decrease from 40 to 32 breaths/min. b. heart rate decrease from 110 to 100 beats/min. c. "quiet chest" from previous assessment of wheezing. d. oxygen saturation of 90%.

ANS: C A "quiet chest" after assessment of wheezing indicates occlusion of air pathways and impending respiratory arrest. All other options are within normal range for infants undergoing oxygen administration.

The nurse notes that a patient has incisional pain, a poor cough effort, and scattered rhonchi after a thoracotomy. Which action should the nurse take first? a. Assist the patient to sit upright in a chair. b. Splint the patient's chest during coughing. c. Medicate the patient with prescribed morphine. d. Observe the patient use the incentive spirometer.

ANS: C A major reason for atelectasis and poor airway clearance in patients after chest surgery is incisional pain (which increases with deep breathing and coughing). The first action by the nurse should be to medicate the patient to minimize incisional pain. The other actions are all appropriate ways to improve airway clearance but should be done after the morphine is given.

A patient in metabolic alkalosis is admitted to the emergency department, and pulse oximetry (SpO2) indicates that the O2 saturation is 94%. Which action should the nurse take next? a. Administer bicarbonate. b. Complete a head-to-toe assessment. c. Place the patient on high-flow oxygen. d. Obtain repeat arterial blood gases (ABGs).

ANS: C Although the O2 saturation is adequate, the left shift in the oxyhemoglobin dissociation curve will decrease the amount of oxygen delivered to tissues, so high oxygen concentrations should be given. Bicarbonate would worsen the patient's condition. A head-to-toe assessment and repeat ABGs may be implemented. However, the priority intervention is to give high-flow oxygen.

5. A patient with a tracheostomy has a new order for a fenestrated tracheostomy tube. Which action should the nurse include in the plan of care in collaboration with the speech therapist? a. Leave the tracheostomy inner cannula inserted at all times. b. Place the decannulation cap in the tube before cuff deflation. c. Assess the ability to swallow before using the fenestrated tube. d. Inflate the tracheostomy cuff during use of the fenestrated tube.

ANS: C Because the cuff is deflated when using a fenestrated tube, the patient's risk for aspiration should be assessed before changing to a fenestrated tracheostomy tube. The decannulation cap is never inserted before cuff deflation because to do so would obstruct the patient's airway. The cuff is deflated and the inner cannula removed to allow air to flow across the patient's vocal cords when using a fenestrated tube.

A patient who has a right-sided chest tube following a thoracotomy has continuous bubbling in the suction-control chamber of the collection device. Which action by the nurse is most appropriate? a. Document the presence of a large air leak. b. Notify the surgeon of a possible pneumothorax. c. Take no further action with the collection device. d. Adjust the dial on the wall regulator to decrease suction.

ANS: C Continuous bubbling is expected in the suction-control chamber and indicates that the suction-control chamber is connected to suction. An air leak would be detected in the water-seal chamber. There is no evidence of pneumothorax. Increasing or decreasing the vacuum source will not adjust the suction pressure. The amount of suction applied is regulated by the amount of water in this chamber and not by the amount of suction applied to the system.

A patient has acute bronchitis with a nonproductive cough and wheezes. Which topic should the nurse plan to include in the teaching plan? a. Purpose of antibiotic therapy b. Ways to limit oral fluid intake c. Appropriate use of cough suppressants d. Safety concerns with home oxygen therapy

ANS: C Cough suppressants are frequently prescribed for acute bronchitis. Because most acute bronchitis is viral in origin, antibiotics are not prescribed unless there are systemic symptoms. Fluid intake is encouraged. Home oxygen is not prescribed for acute bronchitis, although it may be used for chronic bronchitis.

The nurse completes a shift assessment on a patient admitted in the early phase of heart failure. When auscultating the patient's lungs, which finding would the nurse most likely hear? a. Continuous rumbling, snoring, or rattling sounds mainly on expiration b. Continuous high-pitched musical sounds on inspiration and expiration c. Discontinuous, high-pitched sounds of short duration heard on inspiration d. A series of long-duration, discontinuous, low-pitched sounds during inspiration

ANS: C Fine crackles are likely to be heard in the early phase of heart failure. Fine crackles are discontinuous, high-pitched sounds of short duration heard on inspiration. Rhonchi are continuous rumbling, snoring, or rattling sounds mainly on expiration. Course crackles are a series of long-duration, discontinuous, low-pitched sounds during inspiration. Wheezes are continuous high-pitched musical sounds on inspiration and expiration.

The nurse teaches a patient about pulmonary function testing (PFT). Which statement, if made by the patient, indicates teaching was effective? a. "I will use my inhaler right before the test." b. "I won't eat or drink anything 8 hours before the test." c. "I should inhale deeply and blow out as hard as I can during the test." d. "My blood pressure and pulse will be checked every 15 minutes after the test."

ANS: C For PFT, the patient should inhale deeply and exhale as long, hard, and fast as possible. The other actions are not needed with PFT. The administration of inhaled bronchodilators should be avoided 6 hours before the procedure.

The nurse provides preoperative instruction for a patient scheduled for a left pneumonectomy for cancer of the lung. Which information should the nurse include about the patient's postoperative care? a. Positioning on the right side b. Bed rest for the first 24 hours c. Frequent use of an incentive spirometer d. Chest tube placement with continuous drainage

ANS: C Frequent deep breathing and coughing are needed after chest surgery to prevent atelectasis. To promote gas exchange, patients after pneumonectomy are positioned on the surgical side. Early mobilization decreases the risk for postoperative complications such as pneumonia and deep vein thrombosis. In a pneumonectomy, chest tubes may or may not be placed in the space from which the lung was removed. If a chest tube is used, it is clamped and only released by the surgeon to adjust the volume of serosanguineous fluid that will fill the space vacated by the lung. If the cavity overfills, it could compress the remaining lung and compromise the cardiovascular and pulmonary function. Daily chest x-rays can be used to assess the volume and space.

10. A patient who had a total laryngectomy has a nursing diagnosis of hopelessness related to loss of control of personal care. Which information obtained by the nurse is the best indicator that this identified problem is resolving? a. The patient lets the spouse provide tracheostomy care. b. The patient allows the nurse to suction the tracheostomy. c. The patient asks how to clean the tracheostomy stoma and tube. d. The patient uses a communication board to request "No Visitors."

ANS: C Independently caring for the laryngectomy tube indicates that the patient has regained control of personal care and hopelessness is at least partially resolved. Letting the nurse and spouse provide care and requesting no visitors may indicate that the patient is still experiencing hopelessness.

The nurse observes a student who is listening to a patient's lungs who is having no problems with breathing. Which action by the student indicates a need to review respiratory assessment skills? a. The student starts at the apices of the lungs and moves to the bases. b. The student compares breath sounds from side to side avoiding bony areas. c. The student places the stethoscope over the posterior chest and listens during inspiration. d. The student instructs the patient to breathe slowly and a little more deeply than normal through the mouth.

ANS: C Listening only during inspiration indicates the student needs a review of respiratory assessment skills. At each placement of the stethoscope, listen to at least one cycle of inspiration and expiration. During chest auscultation, instruct the patient to breathe slowly and a little deeper than normal through the mouth. Auscultation should proceed from the lung apices to the bases, comparing opposite areas of the chest, unless the patient is in respiratory distress or will tire easily. If so, start at the bases (see Fig. 26-7). Place the stethoscope over lung tissue, not over bony prominences.

A patient who is taking rifampin (Rifadin) for tuberculosis calls the clinic and reports having orange discolored urine and tears. Which is the best response by the nurse? a. Ask if the patient is experiencing shortness of breath, hives, or itching. b. Ask the patient about any visual abnormalities such as red-green color discrimination. c. Explain that orange discolored urine and tears are normal while taking this medication. d. Advise the patient to stop the drug and report the symptoms to the health care provider.

ANS: C Orange-colored body secretions are a side effect of rifampin. The patient does not have to stop taking the medication. The findings are not indicative of an allergic reaction. Alterations in red-green color discrimination commonly occurs when taking ethambutol (Myambutol), which is a different TB medication.

An occupational health nurse works at a manufacturing plant where there is potential exposure to inhaled dust. Which action, if recommended by the nurse, will be most helpful in reducing the incidence of lung disease? a. Treat workers with pulmonary fibrosis. b. Teach about symptoms of lung disease. c. Require the use of protective equipment. d. Monitor workers for coughing and wheezing.

ANS: C Prevention of lung disease requires the use of appropriate protective equipment such as masks. The other actions will help in recognition or early treatment of lung disease but will not be effective in prevention of lung damage. Repeated exposure eventually results in diffuse pulmonary fibrosis. Fibrosis is the result of tissue repair after inflammation.

An infant with bronchiolitis is hospitalized. The causative organism is respiratory syncytial virus (RSV). The nurse knows that a child infected with this virus requires the following isolation: a. reverse isolation. b. airborne isolation. c. Contact Precautions. d. Standard Precautions.

ANS: C RSV is transmitted through droplets. In addition to Standard Precautions and hand washing, Contact Precautions are required. Caregivers must use gloves and gowns when entering the room. Care is taken not to touch their own eyes or mucous membranes with a contaminated gloved hand. Children are placed in a private room or in a room with other children with RSV infections. Reverse isolation focuses on keeping bacteria away from the infant. With RSV, other children need to be protected from exposure to the virus. The virus is not airborne.

The health care provider writes an order for bacteriologic testing for a patient who has a positive tuberculosis skin test. Which action should the nurse take? a. Teach about the reason for the blood tests. b. Schedule an appointment for a chest x-ray. c. Teach about the need to get sputum specimens for 2 to 3 consecutive days. d. Instruct the patient to expectorate three specimens as soon as possible.

ANS: C Sputum specimens are obtained on 2 to 3 consecutive days for bacteriologic testing for M. tuberculosis. The patient should not provide all the specimens at once. Blood cultures are not used for tuberculosis testing. A chest x-ray is not bacteriologic testing. Although the findings on chest x-ray examination are important, it is not possible to make a diagnosis of TB solely based on chest x-ray findings because other diseases can mimic the appearance of TB.

While caring for a patient with respiratory disease, the nurse observes that the patient's SpO2 drops from 93% to 88% while the patient is ambulating in the hallway. What is the priority action of the nurse? a. Notify the health care provider. b. Document the response to exercise. c. Administer the PRN supplemental O2. d. Encourage the patient to pace activity.

ANS: C The drop in SpO2 to 85% indicates that the patient is hypoxemic and needs supplemental oxygen when exercising. The other actions are also important, but the first action should be to correct the hypoxemia.

The nurse is caring for a patient with idiopathic pulmonary arterial hypertension (IPAH) who is receiving epoprostenol (Flolan). Which assessment information requires the most immediate action by the nurse? a. The oxygen saturation is 94%. b. The blood pressure is 98/56 mm Hg. c. The patient's central IV line is disconnected. d. The international normalized ratio (INR) is prolonged.

ANS: C The half-life of this drug is 6 minutes, so the nurse will need to restart the infusion as soon as possible to prevent rapid clinical deterioration. The other data also indicate a need for ongoing monitoring or intervention, but the priority action is to reconnect the infusion.

Which action by the nurse will be most effective in decreasing the spread of pertussis in a community setting? a. Providing supportive care to patients diagnosed with pertussis b. Teaching family members about the need for careful hand washing c. Teaching patients about the need for adult pertussis immunizations d. Encouraging patients to complete the prescribed course of antibiotics

ANS: C The increased rate of pertussis in adults is thought to be due to decreasing immunity after childhood immunization. Immunization is the most effective method of protecting communities from infectious diseases. Hand washing should be taught, but pertussis is spread by droplets and contact with secretions. Supportive care does not shorten the course of the disease or the risk for transmission. Taking antibiotics as prescribed does assist with decreased transmission, but patients are likely to have already transmitted the disease by the time the diagnosis is made.

A patient with right lower-lobe pneumonia has been treated with IV antibiotics for 3 days. Which assessment data obtained by the nurse indicates that the treatment has been effective? a. Bronchial breath sounds are heard at the right base. b. The patient coughs up small amounts of green mucus. c. The patient's white blood cell (WBC) count is 9000/µL. d. Increased tactile fremitus is palpable over the right chest.

ANS: C The normal WBC count indicates that the antibiotics have been effective. All the other data suggest that a change in treatment is needed.

Employee health test results reveal a tuberculosis (TB) skin test of 16-mm induration and a negative chest x-ray for a staff nurse working on the pulmonary unit. The nurse has no symptoms of TB. Which information should the occupational health nurse plan to teach the staff nurse? a. Standard four-drug therapy for TB b. Need for annual repeat TB skin testing c. Use and side effects of isoniazid (INH) d. Bacille Calmette-Guérin (BCG) vaccine

ANS: C The nurse is considered to have a latent TB infection and should be treated with INH daily for 6 to 9 months. The four-drug therapy would be appropriate if the nurse had active TB. TB skin testing is not done for individuals who have already had a positive skin test. BCG vaccine is not used in the United States for TB and would not be helpful for this individual, who already has a TB infection.

3. The nurse discusses management of upper respiratory infections (URI) with a patient who has acute sinusitis. Which statement by the patient indicates that additional teaching is needed? a. "I can take acetaminophen (Tylenol) to treat my discomfort." b. "I will drink lots of juices and other fluids to stay well hydrated." c. "I can use my nasal decongestant spray until the congestion is all gone." d. "I will watch for changes in nasal secretions or the sputum that I cough up."

ANS: C The nurse should clarify that nasal decongestant sprays should be used for no more than 3 days to prevent rebound vasodilation and congestion. The other responses indicate that the teaching has been effective.

4. A nurse who is caring for patient with a tracheostomy tube in place has just auscultated rhonchi bilaterally. If the patient is unsuccessful in coughing up secretions, what action should the nurse take? a. Encourage increased incentive spirometer use. b. Encourage the patient to increase oral fluid intake. c. Put on sterile gloves and use a sterile catheter to suction. d. Preoxygenate the patient for 3 minutes before suctioning.

ANS: C This patient needs suctioning now to secure a patent airway. Sterile gloves and a sterile catheter are used when suctioning a tracheostomy. Preoxygenation for 3 minutes is not necessary. Incentive spirometer (IS) use opens alveoli and can induce coughing, which can mobilize secretions. However, the patient with a tracheostomy may not be able to use an incentive spirometer. Increasing oral fluid intake would not moisten and help mobilize secretions in a timely manner.

When caring for a patient who is hospitalized with active tuberculosis (TB), the nurse observes a family member who is visiting the patient. The nurse will need to intervene if the family member a. washes the hands before entering the patient's room. b. hands the patient a tissue from the box at the bedside. c. puts on a surgical face mask before visiting the patient. d. brings food from a "fast-food" restaurant to the patient.

ANS: C A high-efficiency particulate-absorbing (HEPA) mask, rather than a standard surgical mask, should be used when entering the patient's room because the HEPA mask can filter out 100% of small airborne particles. Hand washing before visiting the patient is not necessary, but there is no reason for the nurse to stop the family member from doing this. Because anorexia and weight loss are frequent problems in patients with TB, bringing food from outside the hospital is appropriate. The family member should wash the hands after handling a tissue that the patient has used, but no precautions are necessary when giving the patient an unused tissue. DIF: Cognitive Level: Application REF: 557 TOP: Nursing Process: Implementation MSC: NCLEX: Physiological Integrity

The nurse notes that a patient has incisional pain, a poor cough effort, and scattered rhonchi after a thoracotomy. Which action should the nurse take first? a. Assist the patient to sit up at the bedside. b. Splint the patient's chest during coughing. c. Medicate the patient with the prescribed morphine. d. Have the patient use the prescribed incentive spirometer.

ANS: C A major reason for atelectasis and poor airway clearance in patients after chest surgery is incisional pain (which increases with deep breathing and coughing). The first action by the nurse should be to medicate the patient to minimize incisional pain. The other actions are all appropriate ways to improve airway clearance but should be done after the morphine is given. DIF: Cognitive Level: Application REF: 574 OBJ: Special Questions: Prioritization TOP: Nursing Process: Implementation MSC: NCLEX: Physiological Integrity

The nurse recognizes that the goals of teaching regarding the transmission of pulmonary tuberculosis (TB) have been met when the patient with TB a. demonstrates correct use of a nebulizer. b. washes dishes and personal items after use. c. covers the mouth and nose when coughing. d. reports daily to the public health department.

ANS: C Covering the mouth and nose will help decrease airborne transmission of TB. The other actions will not be effective in decreasing the spread of TB. DIF: Cognitive Level: Application REF: 557 TOP: Nursing Process: Evaluation MSC: NCLEX: Health Promotion and Maintenance

A patient who has just been admitted with pneumococcal pneumonia has a temperature of 101.6° F with a frequent cough and is complaining of severe pleuritic chest pain. Which of these prescribed medications should the nurse give first? a. guaifenesin (Robitussin) b. acetaminophen (Tylenol) c. azithromycin (Zithromax) d. codeine phosphate (Codeine)

ANS: C Early initiation of antibiotic therapy has been demonstrated to reduce mortality. The other medications also are appropriate and should be given as soon as possible, but the priority is to start antibiotic therapy. DIF: Cognitive Level: Application REF: 549 OBJ: Special Questions: Prioritization TOP: Nursing Process: Implementation MSC: NCLEX: Physiological Integrity

When providing preoperative instruction for a patient scheduled for a left pneumonectomy for cancer of the lung, the nurse informs the patient that the postoperative care includes a. positioning on the right side. b. bed rest for the first 24 hours. c. frequent use of an incentive spirometer. d. chest tubes to water-seal chest drainage.

ANS: C Frequent deep breathing and coughing are needed after chest surgery to prevent atelectasis. To promote gas exchange, patients after pneumonectomy are positioned on the surgical side. Chest tubes are not usually used after pneumonectomy because the affected side is allowed to fill with fluid. Early mobilization decreases the risk for postoperative complications such as pneumonia and deep vein thrombosis. DIF: Cognitive Level: Application REF: 573 TOP: Nursing Process: Planning MSC: NCLEX: Physiological Integrity

Which statement by a patient who has been hospitalized for pneumonia indicates a good understanding of the discharge instructions given by the nurse? a. "I will call the doctor if I still feel tired after a week." b. "I will need to use home oxygen therapy for 3 months." c. "I will continue to do the deep breathing and coughing exercises at home." d. "I will schedule two appointments for the pneumonia and influenza vaccines."

ANS: C Patients should continue to cough and deep breathe after discharge. Fatigue for several weeks is expected. Home oxygen therapy is not needed with successful treatment of pneumonia. The pneumovax and influenza vaccines can be given at the same time. DIF: Cognitive Level: Application REF: 552 TOP: Nursing Process: Evaluation MSC: NCLEX: Physiological Integrity

After the nurse has received change-of-shift report about the following four patients, which patient should be assessed first? a. A 77-year-old patient with tuberculosis (TB) who has four antitubercular medications due in 15 minutes b. A 23-year-old patient with cystic fibrosis who has pulmonary function testing scheduled c. A 46-year-old patient who has a deep vein thrombosis and is complaining of sudden onset shortness of breath. d. A 35-year-old patient who was admitted the previous day with pneumonia and has a temperature of 100.2° F (37.8° C)

ANS: C Sudden onset shortness of breath in a patient with a deep vein thrombosis suggests a pulmonary embolism and requires immediate assessment and actions such as oxygen administration. The other patients also should be assessed as soon as possible, but there is no indication that they may need immediate action to prevent clinical deterioration. DIF: Cognitive Level: Application REF: 577-578 OBJ: Special Questions: Multiple Patients TOP: Nursing Process: Planning MSC: NCLEX: Physiological Integrity

A patient is admitted to the emergency department with an open stab wound to the right chest. What is the first action that the nurse should take? a. Position the patient so that the right chest is dependent. b. Keep the head of the patient's bed at no more than 30 degrees elevation. c. Tape a nonporous dressing on three sides over the chest wound. d. Cover the sucking chest wound firmly with an occlusive dressing.

ANS: C The dressing taped on three sides will allow air to escape when intrapleural pressure increases during expiration, but it will prevent air from moving into the pleural space during inspiration. Placing the patient on the right side or covering the chest wound with an occlusive dressing will allow trapped air in the pleural space and cause tension pneumothorax. The head of the bed should be elevated to 30 to 45 degrees to facilitate breathing. DIF: Cognitive Level: Application REF: 567 OBJ: Special Questions: Prioritization TOP: Nursing Process: Implementation MSC: NCLEX: Physiological Integrity

An alcoholic and homeless patient is diagnosed with active tuberculosis (TB). Which intervention by the nurse will be most effective in ensuring adherence with the treatment regimen? a. Arrange for a friend to administer the medication on schedule. b. Give the patient written instructions about how to take the medications. c. Teach the patient about the high risk for infecting others unless treatment is followed. d. Arrange for a daily noon meal at a community center where the drug will be administered.

ANS: D Directly observed therapy is the most effective means for ensuring compliance with the treatment regimen, and arranging a daily meal will help ensure that the patient is available to receive the medication. The other nursing interventions may be appropriate for some patients but are not likely to be as helpful for this patient.

A patient who has just been admitted with community-acquired pneumococcal pneumonia has a temperature of 101.6° F with a frequent cough and is complaining of severe pleuritic chest pain. Which prescribed medication should the nurse give first? a. Codeine b. Guaifenesin (Robitussin) c. Acetaminophen (Tylenol) d. Piperacillin/tazobactam (Zosyn)

ANS: D Early initiation of antibiotic therapy has been demonstrated to reduce mortality. The other medications are also appropriate and should be given as soon as possible, but the priority is to start antibiotic therapy.

A patient who has a history of chronic obstructive pulmonary disease (COPD) was hospitalized for increasing shortness of breath and chronic hypoxemia (SaO2 levels of 89% to 90%). In planning for discharge, which action by the nurse will be most effective in improving compliance with discharge teaching? a. Start giving the patient discharge teaching on the day of admission. b. Have the patient repeat the instructions immediately after teaching. c. Accomplish the patient teaching just before the scheduled discharge. d. Arrange for the patient's caregiver to be present during the teaching.

ANS: D Hypoxemia interferes with the patient's ability to learn and retain information, so having the patient's caregiver present will increase the likelihood that discharge instructions will be followed. Having the patient repeat the instructions will indicate that the information is understood at the time, but it does not guarantee retention of the information. Because the patient is likely to be distracted just before discharge, giving discharge instructions just before discharge is not ideal. The patient is likely to be anxious and even more hypoxemic than usual on the day of admission, so teaching about discharge should be postponed.

Which intervention will the nurse include in the plan of care for a patient who is diagnosed with a lung abscess? a. Teach the patient to avoid the use of over-the-counter expectorants. b. Assist the patient with chest physiotherapy and postural drainage. c. Notify the health care provider immediately about any bloody or foul-smelling sputum. d. Teach about the need for prolonged antibiotic therapy after discharge from the hospital.

ANS: D Long-term antibiotic therapy is needed for effective eradication of the infecting organisms in lung abscess. Chest physiotherapy and postural drainage are not recommended for lung abscess because they may lead to spread of the infection. Foul smelling and bloody sputum are common clinical manifestations in lung abscess. Expectorants may be used because the patient is encouraged to cough

The nurse completes discharge teaching for a patient who has had a lung transplant. The nurse evaluates that the teaching has been effective if the patient makes which statement? a. "I will make an appointment to see the doctor every year." b. "I will stop taking the prednisone if I experience a dry cough." c. "I will not worry if I feel a little short of breath with exercise." d. "I will call the health care provider right away if I develop a fever."

ANS: D Low-grade fever may indicate infection or acute rejection so the patient should notify the health care provider immediately if the temperature is elevated. Patients require frequent follow-up visits with the transplant team. Annual health care provider visits would not be sufficient. Home oxygen use is not an expectation after lung transplant. Shortness of breath should be reported. Low-grade fever, fatigue, dyspnea, dry cough, and oxygen desaturation are signs of rejection. Immunosuppressive therapy, including prednisone, needs to be continued to prevent rejection.

1. The nurse teaches a patient about discharge instructions after a rhinoplasty. Which statement, if made by the patient, indicates that the teaching was successful? a. "I can take 800 mg ibuprofen for pain control." b. "I will safely remove and reapply nasal packing daily." c. "My nose will look normal after 24 hours when the swelling goes away." d. "I will keep my head elevated for 48 hours to minimize swelling and pain."

ANS: D Maintaining the head in an elevated position will decrease the amount of nasal swelling. NSAIDs, such as ibuprofen, increase the risk for postoperative bleeding and should not be used postoperatively. The patient would not be taught to remove or reapply nasal packing, which is usually removed by the surgeon on the day after surgery. Although return to a preinjury appearance is the goal of the surgery, it is not always possible to achieve this result, especially in the first few weeks after surgery.

A lobectomy is scheduled for a patient with stage I non-small cell lung cancer. The patient tells the nurse, "I would rather have chemotherapy than surgery." Which response by the nurse is most appropriate? a. "Are you afraid that the surgery will be very painful?" b. "Did you have bad experiences with previous surgeries?" c. "Surgery is the treatment of choice for stage I lung cancer." d. "Tell me what you know about the various treatments available."

ANS: D More assessment of the patient's concerns about surgery is indicated. An open-ended response will elicit the most information from the patient. The answer beginning, "Surgery is the treatment of choice" is accurate, but it discourages the patient from sharing concerns about surgery. The remaining two answers indicate that the nurse has jumped to conclusions about the patient's reasons for not wanting surgery. Chemotherapy is the primary treatment for small cell lung cancer. In non-small cell lung cancer, chemotherapy may be used in the treatment of nonresectable tumors or as adjuvant therapy to surgery

A patient is admitted with active tuberculosis (TB). The nurse should question a health care provider's order to discontinue airborne precautions unless which assessment finding is documented? a. Chest x-ray shows no upper lobe infiltrates. b. TB medications have been taken for 6 months. c. Mantoux testing shows an induration of 10 mm. d. Three sputum smears for acid-fast bacilli are negative.

ANS: D Negative sputum smears indicate that Mycobacterium tuberculosis is not present in the sputum, and the patient cannot transmit the bacteria by the airborne route. Chest x-rays are not used to determine whether treatment has been successful. Taking medications for 6 months is necessary, but the multidrug-resistant forms of the disease might not be eradicated after 6 months of therapy. Repeat Mantoux testing would not be done because the result will not change even with effective treatment.

21. The nurse is caring for a patient who has acute pharyngitis caused by Candida albicans. Which action is appropriate for the nurse to include in the plan of care? a. Avoid giving patient warm liquids to drink. b. Assess patient for allergies to penicillin antibiotics. c. Teach the patient about the need to sleep in a warm, dry environment. d. Teach patient to "swish and swallow" prescribed oral nystatin (Mycostatin).

ANS: D Oral or pharyngeal fungal infections are treated with nystatin solution. The goal of the "swish and swallow" technique is to expose all of the oral mucosa to the antifungal agent. Warm liquids may be soothing to a sore throat. The patient should be taught to use a cool mist humidifier. There is no need to assess for penicillin/cephalosporin allergies because Candida albicans infection is treated with antifungals.

The nurse is performing tuberculosis (TB) skin tests in a clinic that has many patients who have immigrated to the United States. Which question is most important for the nurse to ask before the skin test? a. "Is there any family history of TB?" b. "How long have you lived in the United States?" c. "Do you take any over-the-counter (OTC) medications?" d. "Have you received the bacille Calmette-Guérin (BCG) vaccine for TB?"

ANS: D Patients who have received the BCG vaccine will have a positive Mantoux test. Another method for screening (such as a chest x-ray) will need to be used in determining whether the patient has a TB infection. The other information also may be valuable but is not as pertinent to the decision about doing TB skin testing.

The nurse supervises unlicensed assistive personnel (UAP) who are providing care for a patient with right lower lobe pneumonia. The nurse should intervene if which action by UAP is observed? a. UAP splint the patient's chest during coughing. b. UAP assist the patient to ambulate to the bathroom. c. UAP help the patient to a bedside chair for meals. d. UAP lower the head of the patient's bed to 15 degrees.

ANS: D Positioning the patient with the head of the bed lowered will decrease ventilation. The other actions are appropriate for a patient with pneumonia.

The nurse provides discharge teaching for a patient who has two fractured ribs from an automobile accident. Which statement, if made by the patient, would indicate that teaching has been effective? a. "I am going to buy a rib binder to wear during the day." b. "I can take shallow breaths to prevent my chest from hurting." c. "I should plan on taking the pain pills only at bedtime so I can sleep." d. "I will use the incentive spirometer every hour or two during the day."

ANS: D Prevention of the complications of atelectasis and pneumonia is a priority after rib fracture. This can be ensured by deep breathing and coughing. Use of a rib binder, shallow breathing, and taking pain medications only at night are likely to result in atelectasis

The nurse admits a patient who has a diagnosis of an acute asthma attack. Which statement indicates that the patient may need teaching regarding medication use? a. "I have not had any acute asthma attacks during the last year." b. "I became short of breath an hour before coming to the hospital." c. "I've been taking Tylenol 650 mg every 6 hours for chest-wall pain." d. "I've been using my albuterol inhaler more frequently over the last 4 days."

ANS: D The increased need for a rapid-acting bronchodilator should alert the patient that an acute attack may be imminent and that a change in therapy may be needed. The patient should be taught to contact a health care provider if this occurs. The other data do not indicate any need for additional teaching.

13. A nurse is caring for a patient who has had a total laryngectomy and radical neck dissection. During the first 24 hours after surgery what is the priority nursing action? a. Monitor for bleeding. b. Maintain adequate IV fluid intake. c. Suction tracheostomy every eight hours. d. Keep the patient in semi-Fowler's position.

ANS: D The most important goals after a laryngectomy and radical neck dissection are to maintain the airway and ensure adequate oxygenation. Keeping the patient in a semi-Fowler's position will decrease edema and limit tension on the suture lines to help ensure an open airway. Maintenance of IV fluids and monitoring for bleeding are important, but maintaining an open airway is the priority. Tracheostomy care and suctioning should be provided as needed. During the immediate postoperative period, the patient with a laryngectomy requires frequent suctioning of the tracheostomy tube.

2. The nurse plans to teach a patient how to manage allergic rhinitis. Which information should the nurse include in the teaching plan? a. Hand washing is the primary way to prevent spreading the condition to others. b. Use of oral antihistamines for 2 weeks before the allergy season may prevent reactions. c. Corticosteroid nasal sprays will reduce inflammation, but systemic effects limit their use. d. Identification and avoidance of environmental triggers are the best way to avoid symptoms.

ANS: D The most important intervention is to assist the patient in identifying and avoiding potential allergens. Intranasal corticosteroids (not oral antihistamines) should be started several weeks before the allergy season. Corticosteroid nasal sprays have minimal systemic absorption. Acute viral rhinitis (the common cold) can be prevented by washing hands.

A patient with a possible pulmonary embolism complains of chest pain and difficulty breathing. The nurse finds a heart rate of 142 beats/minute, blood pressure of 100/60 mmHg, and respirations of 42 breaths/minute. Which action should the nurse take first? a. Administer anticoagulant drug therapy. b. Notify the patient's health care provider. c. Prepare patient for a spiral computed tomography (CT). d. Elevate the head of the bed to a semi-Fowler's position.

ANS: D The patient has symptoms consistent with a pulmonary embolism (PE). Elevating the head of the bed will improve ventilation and gas exchange. The other actions can be accomplished after the head is elevated (and oxygen is started). A spiral CT may be ordered by the health care provider to identify PE. Anticoagulants may be ordered after confirmation of the diagnosis of PE.

An hour after a thoracotomy, a patient complains of incisional pain at a level 7 (based on 0 to 10 scale) and has decreased left-sided breath sounds. The pleural drainage system has 100 mL of bloody drainage and a large air leak. Which action is best for the nurse to take next? a. Milk the chest tube gently to remove any clots. b. Clamp the chest tube momentarily to check for the origin of the air leak. c. Assist the patient to deep breathe, cough, and use the incentive spirometer. d. Set up the patient controlled analgesia (PCA) and administer the loading dose of morphine.

ANS: D The patient is unlikely to take deep breaths or cough until the pain level is lower. A chest tube output of 100 mL is not unusual in the first hour after thoracotomy and would not require milking of the chest tube. An air leak is expected in the initial postoperative period after thoracotomy.

9. A patient scheduled for a total laryngectomy and radical neck dissection for cancer of the larynx asks the nurse, "Will I be able to talk normally after surgery?" What is the best response by the nurse? a. "You will breathe through a permanent opening in your neck, but you will not be able to communicate orally." b. "You won't be able to talk right after surgery, but you will be able to speak again after the tracheostomy tube is removed." c. "You won't be able to speak as you used to, but there are artificial voice devices that will give you the ability to speak normally." d. "You will have a permanent opening into your neck, and you will need to have rehabilitation for some type of voice restoration."

ANS: D Voice rehabilitation is planned after a total laryngectomy, and a variety of assistive devices are available to restore communication. Although the ability to communicate orally is changed, it would not be appropriate to tell a patient that this ability would be lost. Artificial voice devices do not permit normal-sounding speech. In a total laryngectomy, the vocal cords are removed, so normal speech is impossible.

A patient with pneumonia has a fever of 101.2° F (38.5° C), a nonproductive cough, and an oxygen saturation of 89%. The patient is very weak and needs assistance to get out of bed. The priority nursing diagnosis for the patient is a. hyperthermia related to infectious illness. b. impaired transfer ability related to weakness. c. ineffective airway clearance related to thick secretions. d. impaired gas exchange related to respiratory congestion.

ANS: D All these nursing diagnoses are appropriate for the patient, but the patient's oxygen saturation indicates that all body tissues are at risk for hypoxia unless the gas exchange is improved. DIF: Cognitive Level: Application REF: 552-553 OBJ: Special Questions: Prioritization TOP: Nursing Process: Diagnosis MSC: NCLEX: Physiological Integrity

A patient who has a right-sided chest tube following a thoracotomy has continuous bubbling in the suction-control chamber of the collection device. The most appropriate action by the nurse is to a. document the presence of a large air leak. b. obtain and attach a new collection device. c. notify the surgeon of a possible pneumothorax. d. take no further action with the collection device.

ANS: D Continuous bubbling is expected in the suction-control chamber and indicates that the suction-control chamber is connected to suction. An air leak would be detected in the water-seal chamber. There is no evidence of pneumothorax. A new collection device is needed when the collection chamber is filled. DIF: Cognitive Level: Application REF: 572 TOP: Nursing Process: Implementation MSC: NCLEX: Physiological Integrity

To determine the effectiveness of prescribed therapies for a patient with cor pulmonale and right-sided heart failure, which assessment will the nurse make? a. Lung sounds b. Heart sounds c. Blood pressure d. Peripheral edema

ANS: D Cor pulmonale is right ventricular failure caused by pulmonary hypertension, so clinical manifestations of right ventricular failure such as peripheral edema, jugular vein distention, and right upper-quadrant abdominal tenderness would be expected. Abnormalities in lung sounds, blood pressure, or heart sounds are not caused by cor pulmonale. DIF: Cognitive Level: Application REF: 582 TOP: Nursing Process: Evaluation MSC: NCLEX: Physiological Integrity

An alcoholic and homeless patient is diagnosed with active tuberculosis (TB). Which intervention by the nurse will be most effective in ensuring adherence with the treatment regimen? a. Educating the patient about the long-term impact of TB on health b. Giving the patient written instructions about how to take the medications c. Teaching the patient about the high risk for infecting others unless treatment is followed d. Arranging for a daily noontime meal at a community center and giving the medication then

ANS: D Directly observed therapy is the most effective means for ensuring compliance with the treatment regimen, and arranging a daily meal will help to ensure that the patient is available to receive the medication. The other nursing interventions may be appropriate for some patients, but are not likely to be as helpful with this patient. DIF: Cognitive Level: Application REF: 556 TOP: Nursing Process: Implementation MSC: NCLEX: Physiological Integrity

Which of these orders will the nurse act on first for a patient who has just been admitted with probable bacterial pneumonia and sepsis? a. Administer aspirin suppository. b. Send to radiology for chest x-ray. c. Give ciprofloxacin (Cipro) 400 mg IV. d. Obtain blood cultures from two sites.

ANS: D Initiating antibiotic therapy rapidly is essential, but it is important that the cultures be obtained before antibiotic administration. The chest radiograph and aspirin administration can be done last. DIF: Cognitive Level: Application REF: 549 OBJ: Special Questions: Prioritization TOP: Nursing Process: Implementation MSC: NCLEX: Physiological Integrity

After discharge teaching has been completed for a patient who has had a lung transplant, the nurse will evaluate that the teaching has been effective if the patient states a. "I will make an appointment to see the doctor every year." b. "I will not turn the home oxygen up higher than 2 L/minute." c. "I will not worry if I feel a little short of breath with exercise." d. "I will call the health care provider right away if I develop a fever."

ANS: D Low-grade fever may indicate infection or acute rejection, so the patient should notify the health care provider immediately if the temperature is elevated. Patients require frequent follow-up visits with the transplant team; annual health care provider visits would not be sufficient. Home oxygen use is not an expectation after lung transplant. Shortness of breath should be reported. DIF: Cognitive Level: Application REF: 583 TOP: Nursing Process: Evaluation MSC: NCLEX: Physiological Integrity

A lobectomy is scheduled for a patient with stage I non-small cell lung cancer. The patient tells the nurse, "I would rather have radiation than surgery." Which response by the nurse is most appropriate? a. "Are you afraid that the surgery will be very painful?" b. "Did you have bad experiences with previous surgeries?" c. "Surgery is the treatment of choice for stage I lung cancer." d. "Tell me what you know about the various treatments available."

ANS: D More assessment of the patient's concerns about surgery is indicated. An open-ended response will elicit the most information from the patient. The answer beginning, "Surgery is the treatment of choice" is accurate, but it discourages the patient from sharing concerns about surgery. The remaining two answers indicate that the nurse has jumped to conclusions about the patient's reasons for not wanting surgery. DIF: Cognitive Level: Application REF: 565 TOP: Nursing Process: Implementation MSC: NCLEX: Psychosocial Integrity

Which information about a patient who has a recent history of tuberculosis (TB) indicates that the nurse can discontinue airborne isolation precautions? a. Chest x-ray shows no upper lobe infiltrates. b. TB medications have been taken for 6 months. c. Mantoux testing shows an induration of 10 mm. d. Three sputum smears for acid-fast bacilli are negative.

ANS: D Negative sputum smears indicate that M. tuberculosis is not present in the sputum, and the patient cannot transmit the bacteria by the airborne route. Chest x-rays are not used to determine whether treatment has been successful. Taking medications for 6 months is necessary, but the multidrug-resistant forms of the disease might not be eradicated after 6 months of therapy. Repeat Mantoux testing would not be done since it will not change even with effective treatment. DIF: Cognitive Level: Application REF: 557 TOP: Nursing Process: Implementation MSC: NCLEX: Physiological Integrity

Which action by the occupational health nurse at a manufacturing plant where there is potential exposure to inhaled dust will be most helpful in reducing incidence of lung disease? a. Teach about symptoms of lung disease. b. Treat workers who inhale dust particles. c. Monitor workers for shortness of breath. d. Require the use of protective equipment.

ANS: D Prevention of lung disease requires the use of appropriate protective equipment such as masks. The other actions will help in recognition or early treatment of lung disease, but will not be effective in prevention of lung damage. DIF: Cognitive Level: Application REF: 560-561 TOP: Nursing Process: Assessment MSC: NCLEX: Health Promotion and Maintenance

When assessing a 24-year-old patient who has just arrived after an automobile accident, the emergency department nurse notes that the breath sounds are absent on the right side. The nurse will anticipate the need for a. emergency pericardiocentesis. b. stabilization of the chest wall with tape. c. administration of an inhaled bronchodilator. d. insertion of a chest tube with a chest drainage system.

ANS: D The patient's history and absent breath sounds suggest a right-sided pneumothorax or hemothorax, which will require treatment with a chest tube and drainage. The other therapies would be appropriate for an acute asthma attack, flail chest, or cardiac tamponade, but the patient's clinical manifestations are not consistent with these problems. DIF: Cognitive Level: Application REF: 567 TOP: Nursing Process: Planning MSC: NCLEX: Physiological Integrity

During an otoscopic examination on an infant, in which direction is the pinna pulled? a. Down and back b. Down and forward c. Up and forward d. Up and back

ANSWER: A Correct position for an infant's ear examination is to pull the pinna down and back. Pulling the pinna down and forward is the correct position for a child age 3 years and over. Pulling the pinna up and forward will not allow sufficient visualization of the ear. Pulling the pinna up and back will not allow sufficient visualization of the ear.

Which method should the nurse use to view the tonsils and oropharynx of a cooperative 6-year-old child? a. Ask child to open mouth wide and say "aah." Correct b. Ask child to open mouth wide and then place the tongue blade in the center back area of the tongue. c. Examine the mouth when the child is crying to avoid use of tongue blade. d. Pinch nostrils closed until the child opens his or her mouth and then insert the tongue blade.

ANSWER: A If the child is cooperative, the child can open his or her mouth and move the tongue around for the examiner. A tongue blade is not necessary to visualize the tonsils and oropharynx if the child cooperates. During crying there is insufficient opportunity to completely visualize the tonsils and oropharynx. This is traumatic. There is no reason to use such measures, especially with cooperative children.

The charge nurse is observing a student nurse obtain a temperature on a pediatric patient. The nurse should intervene when observing the student: (Select all that apply.) a. obtain a rectal thermometer probe for a child with diarrhea. b. attempt to take an oral temperature on a child who is receiving oxygen. c. take an oral temperature on a 12-year-old child who ate ice cream 2 hours ago. d. documenting an axillary temperature for a 3-year-old child. e. taking an axillary temperature on a 3-week-old infant.

ANSWER: A, B, D a) Rectal measurement remains the clinical gold standard for the precise diagnosis of fever in infants and children compared with other methods. However, this procedure is more invasive and is contraindicated for infants less than 1 month old, children with recent rectal surgery, children with diarrhea or anorectal lesions, and children receiving chemotherapy. b) Oral temperatures are considered the standard for temperature measurement but are contraindicated in children who have an altered level of consciousness, are receiving oxygen, are mouth breathing, are experiencing mucositis, had recent oral surgery or trauma, or are under 5 years old. c) An oral temperature is appropriate for a 12-year-old child who has not had anything hot or cold to eat or drink recently. d) An axillary temperature is appropriate for a 3-year-old child. This is inconsistent and insensitive in infants and children over 1 month old. Axillary temperatures are inconsistent and insensitive in infants and children over 1 month old. The charge nurse should intervene to assess if a definitive temperature is needed. The temperature may need to be taken by a different route. For infants less than 1 month old, the American Academy of Pediatrics (2001) recommends axillary temperatures.

A nurse is conducting a health history on an adolescent. Components of the health history include: (Select all that apply.) a. sexual history. b. review of systems. c. physical assessment. d. growth measurements. e. family medical history.

ANSWER: A, B, E Sexual history is a component of the health history. Review of systems is a component of the health history. Review of family medical history is a component of the health history. Physical assessment is a component of the physical examination. Growth measurements are a component of the physical examination.

When assessing a preschooler's chest, the nurse would expect: a. respiratory movements to be chiefly thoracic. b. anteroposterior diameter to be equal to the transverse diameter. c. intercostal retractions on respiratory movement. d. movement of the chest wall to be symmetric bilaterally and coordinated with breathing.

ANSWER: D At this age breathing is a coordinated function and is primarily abdominal or diaphragmatic. Thoracic breathing occurs in older children, particularly girls. Anteroposterior diameter is equal to transverse diameter in infants. As the child grows, the chest normally increases in the transverse direction; thus the anteroposterior diameter is less than the lateral diameter. Intercostal retractions indicate respiratory distress. The preschool-age child should have symmetric chest movement bilaterally and a coordinated breathing pattern.

The nurse is assessing skin turgor in a child. The nurse grasps the skin on the abdomen between the thumb and index finger, pulls it taut, and quickly releases it. The tissue remains suspended, or tented, for a few seconds, then slowly falls back on the abdomen. Which evaluation can the nurse correctly assume? a. The tissue shows normal elasticity. b. The child is properly hydrated. c. The assessment is done incorrectly. d. The child has poor skin turgor.

ANSWER: D In normal elasticity the skin would return immediately to its original position. If the child is properly hydrated, skin turgor would be elastic. This is the correct way to assess turgor. "Tenting" is the term for poor skin turgor.

Superficial palpation of the abdomen is often perceived by the child as tickling. Which measure by the nurse is MOST likely to minimize this sensation and promote relaxation? a. Palpating another area simultaneously b. Asking the child not to laugh or move if it tickles c. Beginning with deeper palpation and gradually progressing to superficial palpation d. Having the child "help" with palpation by placing his or her hand over the palpating hand

ANSWER: D This would not promote relaxation and would make it more difficult to perform the abdominal assessment. This may only contribute to the child's laughter or may prove frustrating to both the child and the nurse. Deeper palpation enhances the "tickling" sensation, not lessen it. This allows the nurse to perform the assessment while including the child in the care.

The patient has an order for each of the following inhalers. Which one should the nurse offer to the patient at the onset of an asthma attack? Albuterol (Proventil) Salmeterol (Serevent) Beclomethasone (Qvar) Ipratropium bromide (Atrovent)

Albuterol (Proventil) Correct Albuterol is a short-acting bronchodilator that should be given initially when the patient experiences an asthma attack. Salmeterol (Serevent) is a long-acting β2-adrenergic agonist, which is not used for acute asthma attacks. Beclomethasone (Qvar) is a corticosteroid inhaler and not recommended for an acute asthma attack. Ipratropium bromide (Atrovent) is an anticholinergic agent that is less effective than β2-adrenergic agonists. It may be used in an emergency with a patient unable to tolerate short-acting β2-adrenergic agonists (SABAs).

When admitting a patient with a diagnosis of asthma exacerbation, the nurse will assess for what potential triggers (select all that apply)? Exercise Allergies Emotional stress Decreased humidity Upper respiratory infections

Although the exact mechanism of asthma is unknown, there are several triggers that may precipitate an attack. These include allergens, exercise, air pollutants, upper respiratory infections, drug and food additives, psychologic factors, and gastroesophageal reflux disease (GERD).

A neonate experiences meconium aspiration at the time of delivery and develops respiratory distress syndrome (RDS). Which nursing diagnosis would be most appropriate for an infant diagnosed with this disorder? A. Risk for Infection B. Risk for Aspiration C. Impaired Gas Exchange D. Dysfunctional Ventilatory Weaning Response

Answer: C Rationale: Impaired gas exchange is the most appropriate nursing diagnosis because meconium aspiration interferes with the exchange of O2 and CO2. Risk for infection is present but is not as high a priority as impaired gas exchange. Risk for aspiration has already occurred. Dysfunctional ventilatory weaning response may be appropriate i the newborn demonstrates difficulty with the ventilatory weaning process

Compared with an infant born vaginally an infant born via cesarean section is more likely to manifest which condition? A. crib death syndrome B. neurological deficits C. failure to thrive syndrome D. Respiratory distress syndrome

Answer: D Rationale: Research has shown that respiratory distress syndrome (RDS) is more common in infants born by cesarean birth without labor than in those born vaginally. The other answer options A, B, and C are not associated with cesarean births

When the patient with a persisting cough is diagnosed with pertussis (instead of acute bronchitis), the nurse knows that treatment will include which type of medication?

Antibiotic RATIONALE: Pertussis, unlike acute bronchitis, is caused by a gram-negative bacillus, Bordella pertussis, which must be treated with antibiotics. Corticosteroids and bronchodilators are not helpful in reducing symptoms. Cough suppressants and antihistamines are ineffective and may induce coughing episodes with pertussis.

A 45-year-old man with asthma is brought to the emergency department by automobile. He is short of breath and appears frightened. During the initial nursing assessment, which clinical manifestation might be present as an early manifestation during an exacerbation of asthma? Anxiety Cyanosis Bradycardia Hypercapnia

Anxiety An early manifestation during an asthma attack is anxiety because the patient is acutely aware of the inability to get sufficient air to breathe. He will be hypoxic early on with decreased PaCO2 and increased pH as he is hyperventilating. If cyanosis occurs, it is a later sign. The pulse and blood pressure will be increased.

The patient is hospitalized with pneumonia. Which diagnostic test should be used to measure the efficiency of gas transfer in the lung and tissue oxygenation?

Arterial blood gases RATIONALE: Arterial blood gases are used to assess the efficiency of gas transfer in the lung and tissue oxygenation as is pulse oximetry. Thoracentesis is used to obtain specimens for diagnostic evaluation, remove pleural fluid, or instill medication into the pleural space. Bronchoscopy is used for diagnostic purposes, to obtain biopsy specimens, and to assess changes resulting from treatment. Pulmonary function tests measure lung volumes and airflow to diagnose pulmonary disease, monitor disease progression, evaluate disability, and evaluate response to bronchodilators.

A male patient with COPD becomes dyspneic at rest. His baseline blood gas results are PaO2 70 mm Hg, PaCO2 52 mm Hg, and pH 7.34. What updated patient assessment requires the nurse's priority intervention? Arterial pH 7.26 PaCO2 50 mm Hg Patient in tripod position Increased sputum expectoration

Arterial pH 7.26 The patient's pH shows acidosis that supports an exacerbation of COPD along with the worsening dyspnea. The PaCO2 has improved from baseline, the tripod position helps the patient's breathing, and the increase in sputum expectoration will improve the patient's ventilation.

During admission of a patient diagnosed with non-small cell lung carcinoma, the nurse questions the patient related to a history of which risk factors for this type of cancer (select all that apply)?

Asbestos exposure Correct Exposure to uranium Correct History of cigarette smoking RATIONALE: Non-small cell carcinoma is associated with cigarette smoking and exposure to environmental carcinogens, including asbestos and uranium. Chronic interstitial fibrosis is associated with the development of adenocarcinoma of the lung. Exposure to cancer-causing substances in the geographic area where the patient has lived for some time may be a risk, but not necessarily where the patient was born.

Which position is most appropriate for the nurse to place a patient experiencing an asthma exacerbation? Supine Lithotomy High Fowler's Reverse Trendelenburg

High Fowler's The patient experiencing an asthma attack should be placed in high Fowler's position and may need to lean forward to allow for optimal chest expansion and enlist the aid of gravity during inspiration. The supine, lithotomy, and reverse Trendelenburg positions will not facilitation ventilation.

After assisting at the bedside with a thoracentesis, the nurse should continue to assess the patient for signs and symptoms of what?

Pneumothorax RATIONALE: Because thoracentesis involves the introduction of a catheter into the pleural space, there is a risk of pneumothorax. Thoracentesis does not carry a significant potential for causing bronchospasm, pulmonary edema, or respiratory acidosis.

The nurse evaluates that a patient is experiencing the expected beneficial effects of ipratropium (Atrovent) after noting which assessment finding? a) Decreased respiratory rate b) Increased respiratory rate c) Increased peak flow readings d) Decreased sputum production

c) Increased peak flow readings Ipratropium is a bronchodilator that should result in increased peak expiratory flow rates (PEFRs).

A patient whose tracheostomy was inserted 30 minutes ago is recovering in the postanesthesia recovery unit when he coughs and expels the tracheostomy tube. How should the nurse respond? A. Suction the tracheostomy opening. B. Maintain the airway with a sterile hemostat. C. Use an Ambu bag and mask to ventilate the patient. D. Insert the tracheostomy tube obturator into the stoma.

B) As long as the patient is not in acute respiratory distress after dislodging the tracheostomy tube, the nurse should use a sterile hemostat to maintain an open airway until a sterile tracheostomy tube can be reinserted into the tracheal opening. The tracheostomy is an open surgical wound that has not had time to mature into a stoma. If the patient is in respiratory distress, the nurse will use an Ambu bag and mask to ventilate the patient temporarily.

Which physical assessment finding in a patient with a lower respiratory problem best supports the nursing diagnosis of ineffective airway clearance?

Basilar crackles RATIONALE: The presence of adventitious breath sounds indicates that there is accumulation of secretions in the lower airways. This would be consistent with a nursing diagnosis of ineffective airway clearance because the patient is retaining secretions. The rapid respiratory rate, low oxygen saturation, and presence of greenish sputum may occur with a lower respiratory problem, but do not definitely support the nursing diagnosis of ineffective airway clearance.

The patient who had idiopathic pulmonary fibrosis had a bilateral lung transplantation. Now he is experiencing airflow obstruction that is progressing over time. It started with a gradual onset of exertional dyspnea, nonproductive cough, and wheezing. What are these manifestations signs of in the lung transplant patient?

Bronchiolitis obliterans (BOS) RATIONALE: Bronchiolitis obliterans (BOS) is a manifestation of chronic rejection and is characterized by airflow obstruction progressing over time with a gradual onset of exertional dyspnea, nonproductive cough, wheezing, and/or low-grade fever. Pulmonary infarction occurs with lack of blood flow to the bronchial tissue or preexisting lung disease. With pulmonary hypertension, the pulmonary pressures are elevated and can be idiopathic or secondarily due to parenchymal lung disease that causes anatomic or vascular changes leading to pulmonary hypertension. CMV pneumonia is the most common opportunistic infection 1 to 4 months after lung transplant.

A diabetic multigravida is scheduled for an amniocentesis at 32 weeks gestation to determine the L/S ratio and phosphatidyl glycerol level. The L/S ratio is 1:1 and the presence of phosphatidylglycerol is noted. The nurse's assessment of this data is: The infant is at low risk for congenital anomalies. The infant is at high risk for intrauterine growth retardation. The infant is at high risk for respiratory distress syndrome. The infant is at high risk for birth trauma.

C: The infant is at high risk for respiratory distress syndrome.

A patient with a history of tonsillitis complains of difficulty breathing. Which patient assessment data warrants emergency interventions by the nurse? A. Bilateral erythema of especially large tonsils B. Temperature 102.2° F, diaphoresis, and chills C. Contraction of neck muscles during inspiration D. β-hemolytic streptococcus in the throat culture

C) Contraction of neck muscles during inspiration indicates that the patient is using accessory muscles for breathing and is in serious respiratory distress. The reddened and enlarged tonsils indicate pharyngitis. The increased temperature, diaphoresis, and chills indicate an infection, which could be β-hemolytic streptococcus or fungal infection, but not an emergency situation for the patient.

What should the nurse inspect when assessing a patient with shortness of breath for evidence of long-standing hypoxemia?

Chest excursion Incorrect Spinal curvatures Incorrect Respiratory pattern Incorrect Fingernails and their base Correct RATIONALE: Clubbing, a sign of long-standing hypoxemia, is evidenced by an increase in the angle between the base of the nail and the fingernail to 180 degrees or more, usually accompanied by an increase in the depth, bulk, and sponginess of the end of the finger.

The patient has been diagnosed with head and neck cancer. Along with the treatment for the cancer, what other treatment should the nurse expect? A. Nasal packing B. Epistaxis balloon C. Gastrostomy tube D. Peripheral skin care

C) Because 50% of patients with head and neck cancer are malnourished before treatment begins, many patients need enteral feeding via a gastrostomy tube because the effects of treatment make it difficult to take in enough nutrients orally, whether surgery, chemotherapy, or radiation is used. Nasal packing could be used with epistaxis or with nasal or sinus problems. Peripheral skin care would not be expected because it is not related to head and neck cancer.

The patient seeks relief from the symptoms of an upper respiratory infection (URI) that has lasted for 5 days. Which patient assessment should the nurse use to help determine if the URI has developed into acute sinusitis? A. Coughing B. Fever, chills C. Dust allergy D. Maxillary pain

D) The nurse should assess the patient for sinus pain or pressure as a clinical indicator of acute sinusitis. Coughing and fever are nonspecific clinical indicators of a URI. A history of an allergy that is likely to affect the upper respiratory tract is supportive of the sinusitis diagnosis but is not specific for sinusitis.

When teaching the patient with chronic obstructive pulmonary disease (COPD) about smoking cessation, what information should be included related to the effects of smoking on the lungs and the increased incidence of pulmonary infections? Smoking causes a hoarse voice. Cough will become nonproductive. Decreased alveolar macrophage function Sense of smell is decreased with smoking.

Decreased alveolar macrophage function The damage to the lungs includes alveolar macrophage dysfunction that increases the incidence of infections and thus increases patient discomfort and cost to treat the infections. Other lung damage that contributes to infections includes cilia paralysis or destruction, increased mucus secretion, and bronchospasms that lead to sputum accumulation and increased cough. The patient may already be aware of respiratory mucosa damage with hoarseness and decreased sense of smell and taste, but these do not increase the incidence of pulmonary infection.

After swallowing, a 73-year-old patient is coughing and has a wet voice. What changes of aging could be contributing to this abnormality?

Decreased respiratory defense mechanisms RATIONALE: These manifestations are associated with aspiration, which more easily occur in the right lung as the right mainstem bronchus is shorter, wider, and straighter than the left mainstem bronchus. Aspiration occurs more easily in the older patient related to decreased respiratory defense mechanisms (e.g., decreases in immunity, ciliary function, cough force, sensation in pharynx). Changes of aging include a decreased response to hypercapnia, decreased number of functional alveoli, and increased calcification of costal cartilage, but these do not increase the risk of aspiration.

The nurse evaluates that nursing interventions to promote airway clearance in a patient admitted with COPD are successful based on which finding? Absence of dyspnea Improved mental status Effective and productive coughing PaO2 within normal range for the patient

Effective and productive coughing Airway clearance is most directly evaluated as successful if the patient can engage in effective and productive coughing. Absence of dyspnea, improved mental status, and PaO2 within normal range for the patient show improved respiratory status but do not evaluate airway clearance.

The nurse is assigned to care for a patient who has anxiety and an exacerbation of asthma. What is the primary reason for the nurse to carefully inspect the chest wall of this patient? Allow time to calm the patient. Observe for signs of diaphoresis. Evaluate the use of intercostal muscles. Monitor the patient for bilateral chest expansion.

Evaluate the use of intercostal muscles. The nurse physically inspects the chest wall to evaluate the use of intercostal (accessory) muscles, which gives an indication of the degree of respiratory distress experienced by the patient. The other options may also occur, but they are not the primary reason for inspecting the chest wall of this patient.

Nursing assessment findings of jugular venous distention and pedal edema would be indicative of what complication of chronic obstructive pulmonary disease (COPD)? Acute respiratory failure Secondary respiratory infection Fluid volume excess resulting from cor pulmonale Pulmonary edema caused by left-sided heart failure

Fluid volume excess resulting from cor pulmonale Cor pulmonale is a right-sided heart failure caused by resistance to right ventricular outflow resulting from lung disease. With failure of the right ventricle, the blood emptying into the right atrium and ventricle would be slowed, leading to jugular venous distention and pedal edema.

When assessing a patient's sleep-rest pattern related to respiratory health, what should the nurse ask the patient about (select all that apply)?

Have trouble falling asleep? Awaken abruptly during the night? Need to sleep with the head elevated? RATIONALE: The patient with sleep apnea may have insomnia and/or abrupt awakenings. Patients with cardiovascular disease (e.g., heart failure that may affect respiratory health) may need to sleep with the head elevated on several pillows (orthopnea). Sleeping more than 8 hours per night or needing to urinate during the night is not indicative of impaired respiratory health. Awarded 3.0 points out of 3.0 possible points.

When planning teaching for the patient with chronic obstructive pulmonary disease (COPD), the nurse understands that what causes the manifestations of the disease? An overproduction of the antiprotease α1-antitrypsin Hyperinflation of alveoli and destruction of alveolar walls Hypertrophy and hyperplasia of goblet cells in the bronchi Collapse and hypoventilation of the terminal respiratory unit

Hyperinflation of alveoli and destruction of alveolar walls In COPD there are structural changes that include hyperinflation of alveoli, destruction of alveolar walls, destruction of alveolar capillary walls, narrowing of small airways, and loss of lung elasticity. An autosomal recessive deficiency of antitrypsin may cause COPD. Not all patients with COPD have excess mucus production by the increased number of goblet cells.

The nurse is caring for a patient admitted to the hospital with pneumonia. Upon assessment, the nurse notes a temperature of 101.4° F, a productive cough with yellow sputum, and a respiratory rate of 20. Which nursing diagnosis is most appropriate based upon this assessment?

Hyperthermia related to infectious illness RATIONALE: Because the patient has spiked a temperature and has a diagnosis of pneumonia, the logical nursing diagnosis is hyperthermia related to infectious illness. There is no evidence of a chill, and her breathing pattern is within normal limits at 20 breaths/minute. There is no evidence of ineffective airway clearance from the information given because the patient is expectorating sputum.

What is the priority nursing intervention in helping a patient expectorate thick lung secretions?

Increase fluid intake to 3 L/day if tolerated. Correct RATIONALE: Although several interventions may help the patient expectorate mucus, the highest priority should be on increasing fluid intake, which will liquefy the secretions so that the patient can expectorate them more easily. Humidifying the oxygen is also helpful but is not the primary intervention. Teaching the patient to splint the affected area may also be helpful in decreasing discomfort but does not assist in expectoration of thick secretions.

The nurse evaluates that a patient is experiencing the expected beneficial effects of ipratropium (Atrovent) after noting which assessment finding? Decreased respiratory rate Increased respiratory rate Increased peak flow readings Decreased sputum production

Increased peak flow readings Ipratropium is a bronchodilator that should result in increased peak expiratory flow rates (PEFRs).

Which clinical manifestation should the nurse expect to find during assessment of a patient admitted with pneumonia?

Increased vocal fremitus on palpation RATIONALE: A typical physical examination finding for a patient with pneumonia is increased vocal fremitus on palpation. Other signs of pulmonary consolidation include bronchial breath sounds, egophony, and crackles in the affected area. With pleural effusion, there may be dullness to percussion over the affected area

When teaching the patient with bronchiectasis about manifestations to report to the health care provider, which manifestation should be included? Increasing dyspnea Temperature below 98.6° F Decreased sputum production Unable to drink 3 L low-sodium fluids

Increasing dyspnea The significant clinical manifestations to report to the health care provider include increasing dyspnea, fever, chills, increased sputum production, bloody sputum, and chest pain. Although drinking at least 3 L of low-sodium fluid will help liquefy secretions to make them easier to expectorate, the health care provider does not need to be notified if the patient cannot do this one day.

While teaching a patient with asthma about the appropriate use of a peak flow meter, what should the nurse instruct the patient to do? Keep a record of the peak flow meter numbers if symptoms of asthma are getting worse. Use the flow meter each morning after taking medications to evaluate their effectiveness. Increase the doses of the long-term control medication if the peak flow numbers decrease. Empty the lungs and then inhale quickly through the mouthpiece to measure how fast air can be inhaled.

Keep a record of the peak flow meter numbers if symptoms of asthma are getting worse. It is important to keep track of peak flow readings daily, especially when the patient's symptoms are getting worse. The patient should have specific directions as to when to call the physician based on personal peak flow numbers. Peak flow is measured by exhaling into the flow meter and should be assessed before and after medications to evaluate their effectiveness.

The nurse is evaluating if a patient understands how to safely determine whether a metered dose inhaler (MDI) is empty. The nurse interprets that the patient understands this important information to prevent medication underdosing when the patient describes which method to check the inhaler? Place it in water to see if it floats. Keep track of the number of inhalations used. Shake the canister while holding it next to the ear Check the indicator line on the side of the canister.

Keep track of the number of inhalations used. It is no longer appropriate to see if a canister floats in water or not since this is not an accurate way to determine the remaining inhaler doses. The best method to determine when to replace an inhaler is by knowing the maximum puffs available per MDI and then replacing it after the number of days when those inhalations have been used. (100 puffs/2 puffs each day = 50 days)

When caring for a patient with chronic obstructive pulmonary disease (COPD), the nurse identifies a nursing diagnosis of imbalanced nutrition: less than body requirements after noting a weight loss of 30 lb. Which intervention should the nurse add to the plan of care for this patient? Order fruits and fruit juices to be offered between meals. Order a high-calorie, high-protein diet with six small meals a day. Teach the patient to use frozen meals at home that can be microwaved. Provide a high-calorie, high-carbohydrate, nonirritating, frequent feeding diet.

Order a high-calorie, high-protein diet with six small meals a day. Because the patient with COPD needs to use greater energy to breathe, there is often decreased oral intake because of dyspnea. A full stomach also impairs the ability of the diaphragm to descend during inspiration, thus interfering with the work of breathing. For these reasons, the patient with COPD should eat six small meals per day taking in a high-calorie, high-protein diet, with non-protein calories divided evenly between fat and carbohydrate. The other interventions will not increase the patient's caloric intake.

The nurse determines that a patient is experiencing common adverse effects from the inhaled corticosteroid beclomethasone (Beclovent) after what occurs? Hypertension and pulmonary edema Oropharyngeal candidiasis and hoarseness Elevation of blood glucose and calcium levels Adrenocortical dysfunction and hyperglycemia

Oropharyngeal candidiasis and hoarseness Oropharyngeal candidiasis and hoarseness are common adverse effects from the use of inhaled corticosteroids because the medication can lead to overgrowth of organisms and local irritation if the patient does not rinse the mouth following each dose.

When teaching the patient with cystic fibrosis about the diet and medications, what is the priority information to be included in the discussion? Fat soluble vitamins and dietary salt should be avoided. Insulin may be needed with a diabetic diet if diabetes mellitus develops. Pancreatic enzymes and adequate fat, calories, protein, and vitamins are needed. Distal intestinal obstruction syndrome (DIOS) can be treated with increased water.

Pancreatic enzymes and adequate fat, calories, protein, and vitamins are needed. The patient must take pancreatic enzymes before each meal and snack and adequate fat, calories, protein, and vitamins should be eaten. Fat-soluble vitamins are needed because they are malabsorbed with the excess mucus in the gastrointestinal system. Insulin may be needed, but there is no longer a diabetic diet, and this is not priority information at this time. DIOS develops in the terminal ileum and is treated with balanced polyethylene glycol electrolyte solution (MiraLAX) to thin bowel contents.

The RN is preparing to give a bed bath to an immobilized pt w/ TB. The RN should wear which item when performing this care? Surgical mask and gloves. Particulate respirator, gown, and gloves Particulate respirator and protective eyewear Surgical mask, gown, and protective eyewear

Particulate respirator, gown, and gloves.

A 71-year-old patient is admitted with acute respiratory distress related to cor pulmonale. Which nursing intervention is most appropriate during admission of this patient?

Perform a physical assessment of the respiratory system and ask specific questions related to this episode of respiratory distress. RATIONALE: Because the patient is having respiratory difficulty, the nurse should ask specific questions about this episode and perform a physical assessment of this system. Further history taking and physical examination of other body systems can proceed once the patient's acute respiratory distress is being managed.

During discharge teaching for a 65-year-old patient with chronic obstructive pulmonary disease (COPD) and pneumonia, which vaccine should the nurse recommend that this patient receive?

Pneumococcal RATIONALE: The pneumococcal vaccine is important for patients with a history of heart or lung disease, recovering from a severe illness, age 65 or over, or living in a long-term care facility. A Staphylococcus aureus vaccine has been researched but not yet been effective. The Haemophilus influenzae vaccine would not be recommended as adults do not need it unless they are immunocompromised. The BCG vaccine is for infants in parts of the world where tuberculosis (TB) is prevalent

After admitting a patient from home to the medical unit with a diagnosis of pneumonia, which physician orders will the nurse verify have been completed before administering a dose of cefuroxime (Ceftin) to the patient?

Sputum culture and sensitivity RATIONALE: The nurse should ensure that the sputum for culture and sensitivity was sent to the laboratory before administering the cefuroxime as this is community-acquired pneumonia. It is important that the organisms are correctly identified (by the culture) before the antibiotic takes effect. The test will also determine whether the proper antibiotic has been ordered (sensitivity testing). Although antibiotic administration should not be unduly delayed while waiting for the patient to expectorate sputum, orthostatic BP, pulmonary function evaluation, and serum laboratory tests will not be affected by the administration of antibiotics.

The nurse is assigned to care for a patient in the emergency department admitted with an exacerbation of asthma. The patient has received a β-adrenergic bronchodilator and supplemental oxygen. If the patient's condition does not improve, the nurse should anticipate what as the most likely next step in treatment? IV fluids Biofeedback therapy Systemic corticosteroids Pulmonary function testing

Systemic corticosteroids Systemic corticosteroids speed the resolution of asthma exacerbations and are indicated if the initial response to the β-adrenergic bronchodilator is insufficient. IV fluids may be used, but not to improve ventilation. Biofeedback therapy and pulmonary function testing may be used after recovery to assist the patient and monitor the asthma.

The physician has prescribed salmeterol (Serevent) for a patient with asthma. In reviewing the use of dry powder inhalers (DPIs) with the patient, what instructions should the nurse provide? a) "Close lips tightly around the mouthpiece and breathe in deeply and quickly." b) "To administer a DPI, you must use a spacer that holds the medicine so that you can inhale it." c) "You will know you have correctly used the DPI when you taste or sense the medicine going into your lungs." d) "Hold the inhaler several inches in front of your mouth and breathe in slowly, holding the medicine as long as possible."

a) "Close lips tightly around the mouthpiece and breathe in deeply and quickly." The patient should be instructed to tightly close the lips around the mouthpiece and breathe in deeply and quickly to ensure the medicine moves down deeply into the lungs. Dry powder inhalers do not require spacer devices. The patient may not taste or sense the medicine going into the lungs.

The patient has an order for each of the following inhalers. Which one should the nurse offer to the patient at the onset of an asthma attack? a) Albuterol (Proventil) b) Salmeterol (Serevent) c) Beclomethasone (Qvar) d) Ipratropium bromide (Atrovent)

a) Albuterol (Proventil) Albuterol is a short-acting bronchodilator that should be given initially when the patient experiences an asthma attack. Salmeterol (Serevent) is a long-acting β2-adrenergic agonist, which is not used for acute asthma attacks. Beclomethasone (Qvar) is a corticosteroid inhaler and not recommended for an acute asthma attack. Ipratropium bromide (Atrovent) is an anticholinergic agent that is less effective than β2-adrenergic agonists. It may be used in an emergency with a patient unable to tolerate short-acting β2-adrenergic agonists (SABAs).

A 45-year-old man with asthma is brought to the emergency department by automobile. He is short of breath and appears frightened. During the initial nursing assessment, which clinical manifestation might be present as an early manifestation during an exacerbation of asthma? a) Anxiety b) Cyanosis c) Bradycardia d) Hypercapnia

a) Anxiety An early manifestation during an asthma attack is anxiety because the patient is acutely aware of the inability to get sufficient air to breathe. He will be hypoxic early on with decreased PaCO2 and increased pH as he is hyperventilating. If cyanosis occurs, it is a later sign. The pulse and blood pressure will be increased.

A male patient with COPD becomes dyspneic at rest. His baseline blood gas results are PaO2 70 mm Hg, PaCO2 52 mm Hg, and pH 7.34. What updated patient assessment requires the nurse's priority intervention? a) Arterial pH 7.26 b) PaCO2 50 mm Hg c) Patient in tripod position d) Increased sputum expectoration

a) Arterial pH 7.26 The patient's pH shows acidosis that supports an exacerbation of COPD along with the worsening dyspnea. The PaCO2 has improved from baseline, the tripod position helps the patient's breathing, and the increase in sputum expectoration will improve the patient's ventilation.

When teaching the patient with bronchiectasis about manifestations to report to the health care provider, which manifestation should be included? a) Increasing dyspnea b) Temperature below 98.6° F c) Decreased sputum production d) Unable to drink 3 L low-sodium fluids

a) Increasing dyspnea The significant clinical manifestations to report to the health care provider include increasing dyspnea, fever, chills, increased sputum production, bloody sputum, and chest pain. Although drinking at least 3 L of low-sodium fluid will help liquefy secretions to make them easier to expectorate, the health care provider does not need to be notified if the patient cannot do this one day.

While teaching a patient with asthma about the appropriate use of a peak flow meter, what should the nurse instruct the patient to do? a) Keep a record of the peak flow meter numbers if symptoms of asthma are getting worse. b) Use the flow meter each morning after taking medications to evaluate their effectiveness. c) Increase the doses of the long-term control medication if the peak flow numbers decrease. d) Empty the lungs and then inhale quickly through the mouthpiece to measure how fast air can be inhaled.

a) Keep a record of the peak flow meter numbers if symptoms of asthma are getting worse. It is important to keep track of peak flow readings daily, especially when the patient's symptoms are getting worse. The patient should have specific directions as to when to call the physician based on personal peak flow numbers. Peak flow is measured by exhaling into the flow meter and should be assessed before and after medications to evaluate their effectiveness.

The nurse determines that the patient is not experiencing adverse effects of albuterol (Proventil) after noting which patient vital sign? a) Pulse rate of 72/minute b) Temperature of 98.4° F c) Oxygen saturation 96% d) Respiratory rate of 18/minute

a) Pulse rate of 72/minute Albuterol is a β2-agonist that can sometimes cause adverse cardiovascular effects. These would include tachycardia and angina. A pulse rate of 72 indicates that the patient did not experience tachycardia as an adverse effect.

The nurse is caring for a patient with an acute exacerbation of asthma. Following initial treatment, what finding indicates to the nurse that the patient's respiratory status is improving? a) Wheezing becomes louder. b) Cough remains nonproductive. c) Vesicular breath sounds decrease. d) Aerosol bronchodilators stimulate coughing.

a) Wheezing becomes louder. The primary problem during an exacerbation of asthma is narrowing of the airway and subsequent diminished air exchange. As the airways begin to dilate, wheezing gets louder because of better air exchange. Vesicular breath sounds will increase with improved respiratory status. After a severe asthma exacerbation, the cough may be productive and stringy. Coughing after aerosol bronchodilators may indicate a problem with the inhaler or its use.

The nurse identifies the nursing diagnosis of activity intolerance for a patient with asthma. In patients with asthma, the nurse assesses for which etiologic factor for this nursing diagnosis? a) Work of breathing b) Fear of suffocation c) Effects of medications d) Anxiety and restlessness

a) Work of breathing When the patient does not have sufficient gas exchange to engage in activity, the etiologic factor is often the work of breathing. When patients with asthma do not have effective respirations, they use all available energy to breathe and have little left over for purposeful activity. Fear of suffocation, effects of medications or anxiety, and restlessness are not etiologies for activity intolerance for a patient with asthma.

When caring for a patient with chronic obstructive pulmonary disease (COPD), the nurse identifies a nursing diagnosis of imbalanced nutrition: less than body requirements after noting a weight loss of 30 lb. Which intervention should the nurse add to the plan of care for this patient? a) Order fruits and fruit juices to be offered between meals. b) Order a high-calorie, high-protein diet with six small meals a day. c) Teach the patient to use frozen meals at home that can be microwaved. d) Provide a high-calorie, high-carbohydrate, nonirritating, frequent feeding diet.

b) Order a high-calorie, high-protein diet with six small meals a day. Because the patient with COPD needs to use greater energy to breathe, there is often decreased oral intake because of dyspnea. A full stomach also impairs the ability of the diaphragm to descend during inspiration, thus interfering with the work of breathing. For these reasons, the patient with COPD should eat six small meals per day taking in a high-calorie, high-protein diet, with non-protein calories divided evenly between fat and carbohydrate. The other interventions will not increase the patient's caloric intake.

The nurse determines that a patient is experiencing common adverse effects from the inhaled corticosteroid beclomethasone (Beclovent) after what occurs? a) Hypertension and pulmonary edema b) Oropharyngeal candidiasis and hoarseness c) Elevation of blood glucose and calcium levels d) Adrenocortical dysfunction and hyperglycemia

b) Oropharyngeal candidiasis and hoarseness Oropharyngeal candidiasis and hoarseness are common adverse effects from the use of inhaled corticosteroids because the medication can lead to overgrowth of organisms and local irritation if the patient does not rinse the mouth following each dose.

A patient with an acute exacerbation of chronic obstructive pulmonary disease (COPD) needs to receive precise amounts of oxygen. Which equipment should the nurse prepare to use? a) Oxygen tent b) Venturi mask c) Nasal cannula d Oxygen-conserving cannula

b) Venturi mask The Venturi mask delivers precise concentrations of oxygen and should be selected whenever this is a priority concern. The other methods are less precise in terms of amount of oxygen delivered.

A patient is being discharged from the emergency department after being treated for epistaxis. In teaching the family first aid measures in the event the epistaxis would recur, which of the following measures would the nurse suggest (select all that apply)? A) Tilt patient's head backwards. B) Apply ice compresses to the nose. C) Tilt head forward while lying down. D) Pinch the entire soft lower portion of the nose. E) Partially insert a small gauze pad into the bleeding nostril.

b, d, e

Which of the following symptoms is seen in a child with bronchopulmonary dysplasia? a. Minimal work of breathing b. Tachypnea and dyspnea c. Easily consolable d. Hypotension

b. Tachypnea and dyspnea

The effects of cigarette smoking on the respiratory system include: a. hypertrophy of capillaries causing hemoptysis b. hyperplasia of goblet cells and increased production of mucus c. increased proliferationof cilia and decreased clearance of mucus d. proliferation of alveolar macrophages to decrease the risk for infection

b. hyperplasia of goblet cells and increased production of mucus

a type of tracheostomy tube that prevents speech is a. a cuffless tracheostomy tube b. a fenestrated tracheostomy tube c. a tube with an inflated foam cuff d. a cuffed tube with the cuff deflated

c

When teaching the patient with chronic obstructive pulmonary disease (COPD) about smoking cessation, what information should be included related to the effects of smoking on the lungs and the increased incidence of pulmonary infections? a) Smoking causes a hoarse voice. b) Cough will become nonproductive. c) Decreased alveolar macrophage function d) Sense of smell is decreased with smoking.

c) Decreased alveolar macrophage function The damage to the lungs includes alveolar macrophage dysfunction that increases the incidence of infections and thus increases patient discomfort and cost to treat the infections. Other lung damage that contributes to infections includes cilia paralysis or destruction, increased mucus secretion, and bronchospasms that lead to sputum accumulation and increased cough. The patient may already be aware of respiratory mucosa damage with hoarseness and decreased sense of smell and taste, but these do not increase the incidence of pulmonary infection.

The nurse evaluates that nursing interventions to promote airway clearance in a patient admitted with COPD are successful based on which finding? a) Absence of dyspnea b) Improved mental status c) Effective and productive coughing d) PaO2 within normal range for the patient

c) Effective and productive coughing Airway clearance is most directly evaluated as successful if the patient can engage in effective and productive coughing. Absence of dyspnea, improved mental status, and PaO2 within normal range for the patient show improved respiratory status but do not evaluate airway clearance.

The nurse is assigned to care for a patient who has anxiety and an exacerbation of asthma. What is the primary reason for the nurse to carefully inspect the chest wall of this patient? a) Allow time to calm the patient. b) Observe for signs of diaphoresis. c) Evaluate the use of intercostal muscles. d) Monitor the patient for bilateral chest expansion.

c) Evaluate the use of intercostal muscles. The nurse physically inspects the chest wall to evaluate the use of intercostal (accessory) muscles, which gives an indication of the degree of respiratory distress experienced by the patient. The other options may also occur, but they are not the primary reason for inspecting the chest wall of this patient.

Nursing assessment findings of jugular venous distention and pedal edema would be indicative of what complication of chronic obstructive pulmonary disease (COPD)? a) Acute respiratory failure b) Secondary respiratory infection c) Fluid volume excess resulting from cor pulmonale d) Pulmonary edema caused by left-sided heart failure

c) Fluid volume excess resulting from cor pulmonale Cor pulmonale is a right-sided heart failure caused by resistance to right ventricular outflow resulting from lung disease. With failure of the right ventricle, the blood emptying into the right atrium and ventricle would be slowed, leading to jugular venous distention and pedal edema.

Which position is most appropriate for the nurse to place a patient experiencing an asthma exacerbation? a) Supine b) Lithotomy c) High Fowler's d) Reverse Trendelenburg

c) High Fowler's The patient experiencing an asthma attack should be placed in high Fowler's position and may need to lean forward to allow for optimal chest expansion and enlist the aid of gravity during inspiration. The supine, lithotomy, and reverse Trendelenburg positions will not facilitation ventilation.

When teaching the patient with cystic fibrosis about the diet and medications, what is the priority information to be included in the discussion? a) Fat soluble vitamins and dietary salt should be avoided. b) Insulin may be needed with a diabetic diet if diabetes mellitus develops. c) Pancreatic enzymes and adequate fat, calories, protein, and vitamins are needed. d) Distal intestinal obstruction syndrome (DIOS) can be treated with increased water.

c) Pancreatic enzymes and adequate fat, calories, protein, and vitamins are needed. The patient must take pancreatic enzymes before each meal and snack and adequate fat, calories, protein, and vitamins should be eaten. Fat-soluble vitamins are needed because they are malabsorbed with the excess mucus in the gastrointestinal system. Insulin may be needed, but there is no longer a diabetic diet, and this is not priority information at this time. DIOS develops in the terminal ileum and is treated with balanced polyethylene glycol electrolyte solution (MiraLAX) to thin bowel contents.

The nurse teaches pursed lip breathing to a patient who is newly diagnosed with chronic obstructive pulmonary disease (COPD). The nurse reinforces that this technique will assist respiration by which mechanism? a) Loosening secretions so that they may be coughed up more easily b) Promoting maximal inhalation for better oxygenation of the lungs c) Preventing bronchial collapse and air trapping in the lungs during exhalation d) Increasing the respiratory rate and giving the patient control of respiratory patterns

c) Preventing bronchial collapse and air trapping in the lungs during exhalation The purpose of pursed lip breathing is to slow down the exhalation phase of respiration, which decreases bronchial collapse and subsequent air trapping in the lungs during exhalation. It does not affect secretions, inhalation, or increase the rate of breathing.

The nurse is assigned to care for a patient in the emergency department admitted with an exacerbation of asthma. The patient has received a β-adrenergic bronchodilator and supplemental oxygen. If the patient's condition does not improve, the nurse should anticipate what as the most likely next step in treatment? a) IV fluids b) Biofeedback therapy c) Systemic corticosteroids d) Pulmonary function testing

c) Systemic corticosteroids Systemic corticosteroids speed the resolution of asthma exacerbations and are indicated if the initial response to the β-adrenergic bronchodilator is insufficient. IV fluids may be used, but not to improve ventilation. Biofeedback therapy and pulmonary function testing may be used after recovery to assist the patient and monitor the asthma.

A patient is seen at the clinic with fever, muscle aches, sore throat with yellowish exudate, and headache. The nurse anticipates that the collaborative management will include (select all that apply) a. antiviral agents to treat influenza b. treatment with antibiotics starting ASAP c. a throat culture or rapid strep antigen test d. supportive care including cool, bland liquids e. comprehensive history to determine possible etiology

c, d, e

Infants with bronchopulmonary dysplasia are commonly treated with bronchodilators such as theophylline. Which of the following adverse effects is common with this drug? a. Lethargy b. Decreased calcium level c. Increased heart rate d. Decreased serum potassium level

c. Increased heart rate

The major advantage of a Venturi mask is that it can: a. deliver up to 80% O2. b. provide continuous 100% humidity c. deliver a precise concentration of O2. d. be used while a patient eats and sleeps

c. deliver a precise concentration of O2.

A patient was seen in the clinic for an episode of epistaxis, which was controlled by placement of anterior nasal packing. During discharge teaching, the nurse instructs the patient to a. use asprin for pain relief b. remove the packing later that day c. skip the next dose of antihypertensive medications d. avoid vigorous nose blowing and strenuous activity

d

Which of the following is a late symptom of head and neck cancer? a. hoarseness b. change in fit of dentures c. mouth ulcers that do not heal d. decreased mobility of the tongue

d

a patient with allergic rhinitis reports severe nasal congestion, sneezing, and watery, itchy eyes and nose at various times of the year. To teach the patient to control these symptoms, the nurse advises the patient to a. avoid all intranasal spray and oral antihystamines b. limit the duration of use of nasal decongestant spray for 10 days c. use oral decongestants at bedime to prevent symptoms during the night d. keep a diary of when the allergic reaction occurs and what precipitates it

d

Which statement made by the patient with chronic obstructive pulmonary disease (COPD) indicates a need for further teaching regarding the use of an ipratropium inhaler? a) "I can rinse my mouth following the two puffs to get rid of the bad taste." b) "I should wait at least 1 to 2 minutes between each puff of the inhaler." c) "Because this medication is not fast-acting, I cannot use it in an emergency if my breathing gets worse." d) "If my breathing gets worse, I should keep taking extra puffs of the inhaler until I can breathe more easily."

d) "If my breathing gets worse, I should keep taking extra puffs of the inhaler until I can breathe more easily." The patient should not just keep taking extra puffs of the inhaler to make breathing easier. Excessive treatment could trigger paradoxical bronchospasm, which would worsen the patient's respiratory status. Rinsing the mouth after the puffs will eliminate a bad taste. Waiting 1 to 2 minutes between each puff will facilitate the effectiveness of the administration. Ipratropium is not used in an emergency for COPD.

A patient has been receiving oxygen per nasal cannula while hospitalized for COPD. The patient asks the nurse whether oxygen use will be needed at home. What is the most appropriate response by the nurse? a) "Long-term home oxygen therapy should be used to prevent respiratory failure." b) "Oxygen will not be needed until or unless you are in the terminal stages of this disease." c) "Long-term home oxygen therapy should be used to prevent heart problems related to COPD." d) "You will not need oxygen until your oxygen saturation drops to 88% and you have symptoms of hypoxia."

d) "You will not need oxygen until your oxygen saturation drops to 88% and you have symptoms of hypoxia." Long-term oxygen therapy in the home will not be considered until the oxygen saturation is less than or equal to 88% and the patient has signs of tissue hypoxia, such as cor pulmonale, erythrocytosis, or impaired mental status. PaO2 less than 55 mm Hg will also allow home oxygen therapy to be considered.

while in the recover room, a patient with a total laryngectomy is suctioned and has blood mucus with some clots. Which of the following nursing interventions would apply? a. notify the physician immediately b. place the patient in the prone position to facilitate drainage c. install 3 mL of normal saline into the tracheotomy tube to loosen secretions d. continue your assessment of the patient, including O2 saturation, respiratory rate, and breath sounds

d) Immediately after surgery, the patient with a laryngectomy requires frequent suctioning by means of the laryngectomy tube. Secretions typically change in amount and consistency over time. The patient may initially have copious blood-tinged secretions that diminish and thicken. Normal saline bolus through the tracheostomy tube is not recommended to assist with removal of thickened secretions because it causes hypoxia and damage to the epithelial cells.

The nurse is assisting a patient to learn self-administration of beclomethasone, two puffs inhaled every 6 hours. What should the nurse explain as the best way to prevent oral infection while taking this medication? a) Chew a hard candy before the first puff of medication. b) Rinse the mouth with water before each puff of medication. c) Ask for a breath mint following the second puff of medication. d) Rinse the mouth with water following the second puff of medication.

d) Rinse the mouth with water following the second puff of medication. Because beclamethosone is a corticosteroid, the patient should rinse the mouth with water following the second puff of medication to reduce the risk of fungal overgrowth and oral infection.

When nursing a voice prosthesis, the patient a. swallows air using a Valsalva maneuver b. place a vibrating device in the mouth c. places a speaking valve over the stoma d. blocks the stoma entrance with a finger

d) To speak with a voice prosthesisthe patient manually blocks the stoma with the finger. Air moves from the lungs, through the prosthesis, into the esophagus, and out the mouth. Speech is produced by the air vibrating against the esophagus and is formed into words by moving the tongue and lips.

Before discharge, the nurse discusses activity levels with a 61-year-old patient with chronic obstructive pulmonary disease (COPD) and pneumonia. Which exercise goal is most appropriate once the patient is fully recovered from this episode of illness? a) Slightly increase activity over the current level. b) Swim for 10 min/day, gradually increasing to 30 min/day. c) Limit exercise to activities of daily living to conserve energy. d) Walk for 20 min/day, keeping the pulse rate less than 130 beats/min.

d) Walk for 20 min/day, keeping the pulse rate less than 130 beats/min. The patient will benefit from mild aerobic exercise that does not stress the cardiorespiratory system. The patient should be encouraged to walk for 20 min/day, keeping the pulse rate less than 75% to 80% of maximum heart rate (220 - patient's age).

Which guideline would be a part of teaching patients how to use a metered-dose inhaler (MDI)? a. After activating the MDI, breathe in as quickly as you can b. Estimate the amount of remaining medicine in the MDI by floating the canister in water. c. Disassemble the plastic canister from the inhaler and rinse both pieces under running water every week d. To determine how long the canister will last, divide the total number of puffs in the canister by puffs needed per day

d. To determine how long the canister will last, divide the total number of puffs in the canister by puffs needed per day

Of all the signs seen in infants with respiratory distress syndrome, which sign is especially indicative of the syndrome?

grunting

The nurse needs to give an injection in the deltoid to a 4-year-old child. The BEST approach to use is to: a. smile while giving the injection to help child relax. b. tell the child that you will be so quick that the injection will not even hurt. c. explain that the child will experience a little stick in the arm. d. explain with concrete terms, such as putting medicine under the skin.

his is too abstract. The young child will not correlate a smile with relaxation. Distraction techniques are more appropriate. The nurse does not know that the injection will not hurt the child. Lying or distorting the truth is never appropriate. This response will block trust, especially if the injection does hurt the child. The child may visualize an actual stick being placed in the arm. Children at this age are very literal. By using concrete terms the nurse helps the child understand what the nurse is going to do.

The most important reason to protect the preterm infant from cold stress is that:

it could make respiratory distress syndrome worse.

When planning appropriate nursing interventions for a patient with metastatic lung cancer and a 60-pack-per-year history of cigarette smoking, the nurse recognizes that the smoking has most likely decreased the patient's underlying respiratory defenses because of impairment of

mucociliary clearance. RATIONALE: Smoking decreases the ciliary action in the tracheobronchial tree, resulting in impaired clearance of respiratory secretions and particles, chronic cough, and frequent respiratory infections.

The nurse provides dietary teaching for a patient with chronic obstructive pulmonary disease (COPD) who has a low body mass index (BMI). Which patient statement indicates that the teaching has been effective? "I can have ice cream as a snack every day." "I will drink lots of fluids with my meals." "I will decrease my intake of meat and poultry." "I will exercise for 15 minutes before meals."

"I can have ice cream as a snack every day."

The nurse provides discharge teaching for a patient who has two fractured ribs from an automobile accident. Which statement, if made by the patient, would indicate that teaching has been effective? "I will use the incentive spirometer every hour or two during the day." "I should plan on taking the pain pills only at bedtime so I can sleep." "I am going to buy a rib binder to wear during the day." "I can take shallow breaths to prevent my chest from hurting."

"I will use the incentive spirometer every hour or two during the day."

The nurse teaches a patient about the transmission of pulmonary tuberculosis (TB). Which statement, if made by the patient, indicates that teaching was effective? "I will take the bus instead of driving to visit my friends." "I will keep the windows closed at home to contain the germs." "My husband will be sleeping in the guest bedroom." "I will avoid being outdoors whenever possible."

"My husband will be sleeping in the guest bedroom."

A lobectomy is scheduled for a patient with stage I non-small cell lung cancer. The patient tells the nurse, "I would rather have chemotherapy than surgery." Which response by the nurse is most appropriate? "Are you afraid that the surgery will be very painful?" "Surgery is the treatment of choice for stage I lung cancer." "Did you have bad experiences with previous surgeries?" "Tell me what you know about the various treatments available."

"Tell me what you know about the various treatments available."

Which instruction should the nurse include in an exercise teaching plan for a patient with chronic obstructive pulmonary disease (COPD)? "Upper body exercise should be avoided to prevent dyspnea." "Stop exercising if you start to feel short of breath." "Breathe in and out through the mouth while you exercise." "Use the bronchodilator before you start to exercise."

"Use the bronchodilator before you start to exercise."

A patient has been receiving high-dose corticosteroids and broad- spectrum antibiotics for treatment secondary to a traumatic injury and infection. The nurse plans care for the patient knowing that the patient is most susceptible to a. candidiasis. b. aspergillosis. c. histoplasmosis. d. coccidioidomycosis.

A

Following assessment of a patient with pneumonia, the nurse identifies a nursing diagnosis of ineffective airway clearance. Which assessment data best supports this diagnosis? A. Weak, nonproductive cough effort B. Large amounts of greenish sputum C. Respiratory rate of 28 breaths/minute D. Resting pulse oximetry (SpO2) of 85%

A

The nurse is preparing the patient for a diagnostic procedure to remove pleural fluid for analysis. The nurse would prepare the patient for which test? a. Thoracentesis b. Bronchoscopy c. Pulmonary angiography d. Sputum culture and sensitivity

A

The nurse receives a change-of-shift report on the following patients with chronic obstructive pulmonary disease (COPD). Which patient should the nurse assess first? A patient with a respiratory rate of 38/minute A patient with jugular venous distention and peripheral edema A patient who has a cough productive of thick, green mucus A patient with loud expiratory wheezes

A patient with a respiratory rate of 38/minute

The nurse receives change-of-shift report on the following four patients. Which patient should the nurse assess first? A 23-year-old patient with cystic fibrosis who has pulmonary function testing scheduled A 35-year-old patient who was admitted the previous day with pneumonia and has a temperature of 100.2° F (37.8° C) A 77-year-old patient with tuberculosis (TB) who has four antitubercular medications due in 15 minutes A 46-year-old patient on bed rest who is complaining of sudden onset of shortness of breath

A 46-year-old patient on bed rest who is complaining of sudden onset of shortness of breath

Guidelines for a nurse using an interpreter in developing a care plan for an 8-year-old admitted to rule out epilepsy include: a. explaining to the interpreter what information is necessary to obtain from the patient and family. b. encouraging the interpreter to ask several questions at a time to make the best use of time. c. not giving the interpreter too much information so the interview evolves. d. discouraging the interpreter and client from discussing topics that are deemed irrelevant to the original intent of the interview.

ANSWER: A The interpreter should be given guidance about what information is necessary to obtain during the interview. One question should be asked at a time, leaving sufficient time for the family to answer. The interpreter should not have to guess what to ask and what information to obtain during the interview. The interpreter should gain as much information from the family as they are willing to share based on the questions posed. Limits should not be placed on the interview.

The nurse is ready to begin a physical examination on an 8-month-old infant. The child is sitting contentedly on his mother's lap, chewing on a toy. What should the nurse do FIRST? a. Elicit reflexes b. Auscultate heart and lungs c. Examine eyes, ears, and mouth d. Examine head, systematically moving toward feet

ANSWER: B This may disturb or upset the child, making auscultation and the remainder of the physical examination difficult. Auscultation should be performed while the child is quiet. This may disturb or upset the child, making auscultation and the remainder of the physical examination difficult. Although this is the way most physical examinations proceed, the nurse should perform the assessment for a child in an order that moves from least disturbing to most disturbing from the child's perspective.

Which statement is true concerning the increased use of telephone triage by nurses? a. Telephone triage has led to an increase in health care costs. b. Emergency department visits are not recommended by nurses and thus are not a Perry component of telephone triage. c. Access to high-quality health care services has increased through telephone triage. d. Home care is often recommended when it is not appropriate.

ANSWER: C Health care costs have decreased because of fewer visits to emergency departments. Based on the response to screening questions, the triage nurse determines whether the child needs to be referred to emergency medical services. The nurse can then initiate the call if needed. The judicious use of telephone triage has decreased the number of unnecessary visits, allowing time for improved care. Home care is recommended only when indicated on the basis of the screening questions.

The nurse is interviewing the mother of Adam, 9 years old. As the nurse begins to assess Adam's school performance, the MOST appropriate question to ask is: a. "Did Adam go to preschool?" b. "Does Adam have problems at school?" c. "How is Adam doing in school?" d. "How well does Adam seem to be doing in school?"

ANSWER: C a. This is a close-ended question, which will elicit a yes or no answer. b. This is a close-ended question that implies that Adam is not doing well. c. This is an open-ended question without any descriptive terms that may limit the mother's responses. d. This is a close-ended question that will have a short answer and assumes that Adam is doing well.

The most accurate method of determining the length of a child less than 12 months of age is: a. standing height. b. estimation of length to the nearest centimeter or ½ inch. c. recumbent length measured in the prone position. d. recumbent length measured in the supine position.

ANSWER: D Infants are generally unable to stand to obtain a height measurement. Measurement should not be estimated since an accurate measurement is required to determine growth. The infant should be measured in the supine, not the prone, position. The crown-heel length measurement is the most accurate measurement in infants.

Which statement explains why it can be difficult to assess a child's dietary intake? a. No systematic assessment tool has been developed for this purpose. b. Biochemical analysis for assessing nutrition is expensive. c. Families usually do not understand much about nutrition. d. Recall of children's food consumption is frequently unreliable.

ANSWER: D Systematic tools have been developed and are available. Nutrients for different foods are known; it is the quantity and type of food consumed that are difficult to ascertain. The family does not need nutrition knowledge to describe what the child has eaten. It is difficult for parents to recall exactly what their child has eaten. Concurrent food diaries are somewhat more reliable.

An 81-year-old patient who has been in the intensive care unit (ICU) for a week is now stable and transfer to the progressive care unit is planned. On rounds, the nurse notices that the patient has new onset confusion. The nurse will plan to notify the health care provider and postpone the transfer. obtain an order for restraints as needed and transfer the patient. inform the receiving nurse and then transfer the patient. give PRN lorazepam (Ativan) and cancel the transfe

inform the receiving nurse and then transfer the patient.

To verify the correct placement of an oral endotracheal tube (ET) after insertion, the best initial action by the nurse is to auscultate for the presence of bilateral breath sounds. use an end-tidal CO2 monitor to check for placement in the trachea. observe the chest for symmetric chest movement with ventilation. obtain a portable chest x-ray to check tube placement.

use an end-tidal CO2 monitor to check for placement in the trachea.


Ensembles d'études connexes

Project + Practice Qs (CompTIA Certmaster)

View Set

Intro to Criminal Justice Chapter 3

View Set

Chapter 8: Cognition and Perception

View Set

HESI/Saunders Module 7: Basic Care and Comfort

View Set